Você está na página 1de 150

Oi::

o?-
ô
\J)
+

ª
N
o

f
l•AUL0

PROBLE MAS
DE
MATEMÁTICA
PARA O ADMISSÃO

AO PAULO IOZON+EDlTOR 1''0RTALEZA


1 unru fio Pni ~sandu, 51 Rua Pedro Pereira:, 313
l\pl" l fiO J e 1502 R O Gtupo 2 - CP. 1236
-r.,1. ' 30 8670 - 35-8815 Av, Mal. Floriano, 22. ,1 .0 Te 1 e fone : 21-9357
N 'I' 1.; Jt ó Tels .: 223-3943/243-6064 B E L É M
Av . l\'ntmil P •l xo lo, 37 0 R. Barão Gna·l'atlbn, 29/3 1 Rua Mundu cmus, 1567
/ 721 Tclcf.: 2 - 6433 T e 1 e f o n e : 245·7126 T e 1 e f o n e : 5141
Aos MEUS NETOS

MÓNICA,
SOLANGE,
ANDRÉ
E
FERNANDA
OOM A BÊNÇÃO DO VOVÔ

PAULO
PREFÁCIO

Não estava em nossas cogitações escrever êste livro e


só resolvemos organizá- lo ao termos conhecimento de que
"Problemas de Aritmetica" de nossa autoria, está sendo uti-
lizado no preparo de candidatos aos exames de admissão ao
curso ginasial. Tendo sido aquêle livro elaborado com o fim
específico de atender às necessidades dos alunos do curso
ginasial; dos candidatos às escolas preparatórias e de todos
aquêles que pr.e tendem ingressar nos Institutos de Educação
para cursarem o Normal, seu nÍV'el ér como não podia dei-
xar de s er, bem mais elevado do que o exigido para o exame
d admissão ao 1. 0 ano do primeiro ciclo escolar.

Para ovílar os inc:onv. ni nte:; do uso de um livr.o 1nndt1-


11umlo ao objetivo a atingir, resolvemos lançar "ProJ:;fomws
de Ma temática para o Admissão", esperando que êlc possa
1cw;ino1. aos jovens a dar os prímeitos passos no es tudo d r:1
111ul má li ca .

AUT lt
NUMERAÇÃO

Aritmetica é a c1encia dos números. Ela nos ensina o


medir, contar e calcular as grandezas.
Grandeza é tudo ,q ue pode ser medido, contado, compa-
rado, pesado, etc. Sua noçÇío é proveniente da comparação
de dois objetos ou de duas quantidades da mesma espécie.
Assim é que ao depararmos com dois postes podemos
compará-los no que diz respeito às suas alturas ou às suas
grossuras, por exemplo. A altura de um poste; de uma casa;
de uma pessoa ou de uma arvore, etc., é uma grandeza.
Eias podem ser contínuas e descontÍlmas .
Dizem-se contínuas, quando podem ser aumentada s ou
diminuídas de uma quantidade qualquer.
São descontínuas quando só podem ser aumentadas ou
diminuídas de urna quantidade determinada e no mínimo
igual a ela.
Podem ser ainda: mensuráveis e imensuráveis.
As primeiras são as que podem ser medidas, como por
iexemplo: o comprimento de um fio; o pêso de um corpo; a
superfície de um terreno; o volume de um sólido; etc.
As imensuráveis são as que não podem ser medidas e
em outro estágio da vida escolar serão mencionadas.
Quantidade é a grandeza medida. Medir uma grandeza
é compará-la com outra da mesma espécie chamada unidade.
Por exemplo: escolhido o metro para medida de compri-
m ento, comparar com êle o comprimento de uma mesa para
fi car conhecendo quantos metros ela tem . A unidade é pois
uma grandeza conhecida, com a qual se comparam as gran-
d zas da mesma espécie que se pretende medir.

Problemas de Ma lemálica para o Admis são 11


-- --- - -- - ,,
Dígitos - Os que podem ser contados pelos dedos das
No caso das grandezas descontinuas, tais como homens , mãos. São pois os 10 primeiros números.
pássaros, bolas, etc., um homem; um pássaro, uma bola Os nove primeiros números são representados por ca-
~ão as unidades respectivas.
racteres ou símbolos, denominados algarismos. Com êles e
Número é o resultado da comParaçõ:o de uma grar d!:!- .m ais o algarismo zero, pode-se representar qualquer
za com a sua respectiva unidade. A unidade tem por medi- número.
da o número um. Numeração - É o conjunto de princípios e leis empre-
Podem Sf',r inteiros (os números), quando contêm a uni- gados para exprimir os números com o auxílio dos alga-
tiade um número exato de vêzes: Ex.: S, 7, 11. etc. rismos. Ela compreende duas Partes: a numeração talada
Nat urais, assim chamados todos os números inteiros a e a numeração escrita.
partir de um, exceto o zero que é um número inteiro mas A numeração falada tem por fim dar nomes aos nú-
não é u m número natural. meros, por meio de pequeno número de palavras, combi-
Fracionários - os que contêm uma ou várias partes da nadas entre si, de acôrdo com regras simples.
1 3 O princípio fundamental da numeração falada se ba-
unid d sem contudo alcançá-la . Ex.: - - -, - -- , seia no fato de certo número de unidades de uma ordem
3 4 qualquer for marem uma unidade de ordem imediatamente
2 superior.
- - -.etc. A numeração falada procura reunir os nú,meros em
s f;éries , chamadas ordens e as ordens em classes. Há três
Mixtos - Os que são compostos de uma parte inteira ordens de unidades: unidades, dezenas e centenas. Dez
e outra fracionária, isto é, constituídos de inteiros e fracio - unidades valem uma dezena e dez dezenas valem uma cen-
2 4 2 tena. Para não haver necessidade de usar novas ordens de
nários. Ex. : l + 5 +-- --; 8 + --- que tam- unidades, elas são grupadas em classes. Assim dez cente·
3 7 5
nas formam uma unidade de classe superior, isto é, a clas-
2 4 2
1se dos milhares.
bém podem ser escritos: 1 - - - ; 5 - - - ; 8 - --
Ao preparar-se para o exame de admissão ao ginásio,
3 7 s o aluno já aprendeu a ler e escrever números, motivo pelo
qual não iremos insistir ·em tais assuntos.
Concretos - São os que designam a espécie da unidade.
A numeração escrita é aquela que, empregando os al-
Ex. : 9 alunos; 7 laranjas; 8 metros.
garismos combinados entre si, segundo regras simples
, Abstratos - São os que não designam a espécie de uni-
apresenta os números por escrito.
aade. Ex.: 5; 9; 2 .
. Ordinais - Os que indicam a ordem ocupada por um Princípio fundam ental de numeração escrita
obieto ou pessoa. Ex.: 5.0 livro; 3.0 filho, etc.
Cardinais - Os que indicam apenas o total de objetos Todo algarismo escrito à esquerda de outro representa
ou pessoas. Ex.: 45 lápis; 28 alunos. unidades dez vêzes maiores do que representaria se estivese
Simples - Os que têm só um algarismc escrito no lugar desse outro.
Compostos - Os que têm mais de um algarismo. A partir da direita, o primeiro algarismo representa uni-
Pares - Os que terminam em zero, 2, 4, 6 e 8. dc1dcs simples; o segundo dezenas; o terceiro centenas.
Impares -- Os que terminam em 1. 3, 5, 7 e 9.
l,rolll~111a~ de M.atemi\tictt :para o A<lmissiio 13
)2, Paulo Pessoa
Valor dos algarismos Um algarismo colocado à direita de outro de maior
valor é somado a êste. Assim: XV = 15; XIII = 13; XVIII
. . o~ algarismos de l até 9 são chamados algarismos sig- = 18.
mt1cat1vos e o zero, algarismo insignificativo. Um algarismo colocado à esquerda do outro de maior
Como conseqüência do que foi dito quando tratamos valor é subtraído dêste. Assim: IV = ! 4; CM = 900; IX = 9.
do princípio fundamental da numeração escrita, os alga-
Um traço horizontal coiceado sôbre um algarismo ou
rismos significativos possuem dois valôres, isto é, o valor
um grupo de algarismos torna-o ou torna-os mil vêzes maior.
absoluto, que êle tem quando está isolado e o valor relativo
que ~ o que possui de acôrdo com a posição que ocupa Assim: l = l . 000: Se o número de traços fôr dois o
no numero. algarismo ou grupo de àlgaris mos fica multiplicado por um
Assim, no númêro 1968 o algarismo nove tem o valor milhão . Assim: V = 5 . 000 . 000.
relativo 900, o algarismo. 1 tem o valor relativo 1. 000; o
algarismo 6 tem o · valor relativo 60- e o algarismo 8 tem De modo semelhante
o valor relativo igual ao absoluto, isto é, 8.

Sistema de numeração
I == 1.000.000.000
É um conjunto de palavras, sinais e regras com os quais
apresentamos os números. Base de um sistema de numera- Depois do que foi dito
ção é o número de unidades de uma ordem necessário para
formar uma unidade de valor imediatamente superior. 1968 MCMLXVIII
O sistema de numeração usado por nós é o decimal 2000 = ll ou MM
porque 10 unidades de uma ordem valem uma de ordem 1500 MD
imediatamente superior. O número de algarismos de um
'Sistema é igual à base. No sistema decimal há 10 algaris-
mos. EXERCíCIOS RESOLVIDOS
Há duas espécies de algarismos. Os arábicos comu-
mente usados e os r.omanos. 1) Qual o menor número de três algarismos expresso
pelos algarismos O, 1 e 5?
Os caracteres representativos dos algarismos romanos
são: 1 (um); V (cinco); X (dez); L (cincoenta); C (cem); D O número não pode começar por zero porque ficando
(quinhentos) e M (mil). êle à esquerda, o número só teria dois algarismos.

Quando vamos escrever números com algarismos ro- Tendo três algarismos a ordem mais elevada é a das
manos, qualquer dos algarismos escritos (no máximo três centenas que por isso deve ser preenchida pelo menor dos
vêzes) um ao lado do outro, representa três vêzes o seu algarismos significativos dados e por isso só pode ser 1.
valor. Essa regra: só é empregada com os algarismos I. X, Se o segundo algarismo, o das dezenas fôsse o 5 o das
CeM unidades teria que ser o zero e o valor relativo do 5 no nú-
Assim XX = ZO; llI = 3; CCC = 300. mero seria 50. Concluímos então que o número menor é 105.

14 l'aulo Pessoa Problemas de Matemática: para . o Admlssüo 15


~) O algarismo 8 ocupa em um número inteiro a. qucirl 8) Quantas dezenas existem em 1968?
cerno. Qual o seu valor relativo? Como das vêzes anteriores basla dividir por 10, parcr
A quar ta casa é das unidad s de milhar; enlão seu en conlrar 196.
valor relativo é 8. 000. 9) Quais são as unidades dez vêzes maiores que as
3) De que ordem são as centenas de milhar? centenasº?
A existê ncia de centenas de milhar em um número, sig- O número dez vêzes maior que as centena.s é obtido
nifica que o número tem, pelo menos, duas classes e como multiplicando-se 100 por 10, cujo resultado é 1. 000. O al-
cada classe tem 3 ordens conclui~se que a centena de mi- garismo de quarta é: unidade de milhar.
lhar é um algarismo da 6.ª ordem .
10) Escrever em algarismos romanos: 3.005; 12.729;
4) Quando se escrevem todos os números de 10 o 100, 5.329.803; 7.345.129.200.
quantas vêzes se escreve o algarismo l?
Em intervalos pequenos como o do problema é fácil fa·· III V; XII DCC XXIX;
zer-se u verificação.
),
Assim é que de 10 até 19, como algarismo das unida-
des aparece apenas uma vez, no número 11.
Como algarismo das dezenas aparece: 19 -- 10 l + VII CCCXXIX DCCCIII VII CCCXL V CXXIX CC
10 vêzes.
Aparece ainda em 21, 31, 41, 51, 61, 71, 81 e 91 isto é,
8 vêzes e finalmenie no número 100, uma vez coi:no alga-
EXERCíCIOS PARA RESOLVER
rismo das centenas.
O total de vezes é pois: Q Qual o menor e o maior número de quatro alga-
rismos que se pode· escrever com os algarismos 8, 3, 5 e 2 .
1 + 10 +8+ 1 = 20 vêzes
Resposta: 2. 358 e 8 . 532
5) Quantos números existem de 4 algarismos?
O menor número de 4 algarismos é 1.000 e o maior é
9.999. Entre êles, contando com êles, existem
o o algarismos
qu~;~casa. Qual o
4 ocupa em um número inteiro a
seu valor relativo?
9 . 999 - 1 . 000 + 1 = 9. 000 números. Resposta: 40. 000
6) Quantos milhares há em 3.300?
Basta calcularmos quantas vêzes 1 . 000 está contido em
r;)o algarismo 3 ocupa em um número inteiro a ter-
ceira casa. Qual o seu valor relativo?
3.300 para concluirmos que existem 3 milhares em 3.300.
Resposta: 300
7) Quntas centenas há em 50.000? 4) Quando se escrevem todos os números desde 1 até
Por um raciocínio idêntico ao apresentado no problema 80, quantas vêzes escrevemos o algmismo 6?
anterior, é bostante dividir 50.000 por 100 para achar 500
para resposta. Resposta: 18 vêzes

16 P n.u lo Pe s soa Probler~as de Matemittic" parn o Admi seão 17


\
5) Quando se escrevem todos os números desde 1 até
100, quantas vêzes se escreve o algarismo l?
W No número 5 . 974 a soma do valor relativo do
a lgarismo 9 com o valor absoluto de . . . . é de CMIX .
Resposta: 21 vêzes Resposta: 19. C. Mili tar, 1968
® Qual é o maior número d~ 6 algarismos? rífi)) Escreva o menor número de 3 algarismos que s e
Resposta: 999 . 999 p~screver com os algarismos 7, 2 e 5, ocupando o 7 a

PI> Qual é o menor número de quatro algarismos?


ordem das dezenas.
Resposta: 275. Ginasial, 19,68
Resposta: 1.000.
@ Quantos números existem de 6 algarislos? screva em algarismos arábicos o número quatro
quarenta mil e quatro.
Resposta: 900.00. Resposta: 4 040 004. Ginasial, 19138
@ Quantas dezenas de milhar ha"' em 439 . 852?
~ Com os algarismos 2, 7, O, 4 e 1 escrever:
Resposta: 4.
a) o maior número de 5 algarismos.
~ Quantas centenas há em 972. 345 e 7. 893. 459?
b) o menor número de 5 algarismos.
Resposta: 9.723 e 78.934.
0 Quantas unidades há em 1968?
Cl e m algarismos romanos o número de centenas do
primeiro.
Resposta: a) 74.210
Resposta: 1.968.
~ Escrever em algarismos romanos 1.889; 32.915; b) 10.247
6 . 183. ~2 5 c) DC CXLII. C. Pe dro 11, 1968

Resposta: MDCCCLXXXIX; XXXIICMXV:


(iro) Qual o número de dois algarismos, menor do que
20, que tenha a soma dos valôres absolutos de seus algo-
VI CLXXXIII CMXXV rismos igual a 9.

.@ Escreva em algarismos romanos a diferença e a


Resposta: 18. Liceu Nilo Peçanha, 1968
soma do maior número de três algarismos diferentes e do ~ual o maior número de 5 algarismos distintos ?
m enor número d 9 três algarismos diferentes.
Resposta: 98.765. Liceu Nilo Peçanha, 1968
Resposta: DCCCLXXXV e MLXXXIX. C. Pedro II 1968
14~ 3 e meia unidades de 5.ª ordem a quanto equi- ~ Considere o número s etenta e cinco mil e vinto
va em? cinco. Es ·eva êsse número cm a lga rismos arábicos.

Resposta: 3,500. C. Militar 1968 Resposta: 75.02 5. Liceu Nilo Peçanlla, 196&

18 Pft.11!0 Pessoa Prob le ma" de l11<1tem(I Li c>< para o Adni ii;si"io 19


22) iga o nome da classe mais elevada do número
da questão acima.
Resposta: Milhar. Liceu Nilo Peçanha, 1968

'3 3) No mesmo número, diga o nome da ordem mais


elevada.
OPERAÇÕES FUNDAMENTAIS
Resposta: Dezena de milhar.
Liceu Nilo Peçonha, 1968
Operações fundamentais são:
" 24) No mesmo número dê o valor relativo do algaris-
Soma ou adição; subtração; multiplicação e divisão.
mo ae 4.ª ordem.
Veja mos o que significa cada uma deia.3.
Resposta: 5.000. Liceu Nilo Peçanha, 1968 Somar é reunir duas ou mais quanêidades de: mesma
espécie, numa só.
-25)1 Dado o número 91.742.148, qual a maior classe e Quantldades da m esma espécie são chamadas homo-
a maiÓr ordem? gêneas. Só podemos somar, portmlto, quantidades homo -
gêneas.
Resposta: Milhões e dezenas de milhões .
Quantidades de espécies diferentes são chamadas he-
Liceu Nilo Péçanl10, 1967 'lerog ên eas e não podem ser somadas. As quantidades a
somar chamam-se parcelas e o resultado da -apuração,
soma.
26) Na adição IV CLXVIII + ... MCDXXXI es· Uma soma é indicgda pelo sinal mais ( \- ) colocado
crever a parcela que falta, em romanos. entre as parcelas (soma indicada).
Resposta: CMXCVICCLXIII. Assim:
Liceu Nilo Peçanha, 1968 1. 967 + l. 397 + 105
:indica uma soma cujas parcelas são: 1.987; 1.397 e 105.
Para obtermos o seu resultado costumamos dispô-las como
\Se segue:

1.967 }
+ 1.397 parcelas
105

3.469 Soma efetuada


ou valor d a soma ou resultado da adição ou, simples-
mente, som:a .

Problema• ele Matemática parn o Admis s iio . 21


-- --- '"

Propriedades da sômu O minuendo, o s ubtraendo e o resto, chamam-se têrmos


da subtração.
l.ª) A soma de várias parcelas é independente de Não podemos s ubtrair quantidades heterogêneas., isto é,
flua ord em (propriedade comutativa). d e espécies diferentes .
Assim Indica-se uma subtração colocando-se o sinal menos
(- ) entre o minuendo e o subtraendo. Assim:
1.967 ·i · 1.387 ·1- 105 é o mesmo que
l. 397 +1. 967 + 105 ou 3 . 547 - 132 é uma subtração indicada.
105 +1.967 + 1.367 . A subtração não é uma operação comutativa pois se
podemos subtrair de 3. 547, 132, não é possível de 132 sub-
2.ª) Numa sorna indicada de vanas parcelas pode-
trair 3.547.
mos substituir várias de suas parcelas pela sua soma (pro-
priedade associativa). 3. 547 é o minuendo e 132 o subtraendo.
1 . 967 + 1 . 397 + 105 é o mesmo ,q ue Para obtermos o seu resultado (resto; excesso ou di-
1-967 -~ 1. 397 + 105 ferença) costumamos dispô-los como se segue:
3 . 3;64 .+ 105 ou 3. 547 (minuendo)
1.967 + 1.397 + 105 ou 132 (subtraendo)
1. 967 + l. 502 3 . 415 (resto)
Numa soma indicada de vanas parcelas, pode-
3.ª)
mos decompor qualquer das parcelas e várias outras que Propriedades da subtração
somadas tenham o 8eu valor (propriedade dissociativa).
l.ª) Uma subtração não se altera quando se soma ou
Assim: subtrai ao minuendo e ao subtraendo, a mesma quantidade.
1 . 967 + 1 . 502 é o mesmo que
Assim:
1.967 + 1. 397 + 105 ou Minuendo : 1968
1 . 000 +· 900 + 67 +1 . 502 Subtraendo: 347
Subtrair é achar quantas unidades de uma das quan-
tidades, chamada minuendo, excede às de outra da mesmct Resto 1621
espécie, denominada subtraendo . O resultado da opera-
ção chama.se resto, excesso ou dife:;.;ença. Esta definição Se somarmos ao minuendo e ao subtraendo 439, por
permite-nos escrever: exemplo, teremos:

Minuendo = Subtraendo Resto + Nôvo Minuendo: 1968 ·+ 439


347 + 439
::=: 2407
Subtraendo =
Minuendo - Resto Nôvo -Subtraendo: ::=: 786

= Minuendo - Subtraendo Resto (mesmo) 1621


Resto

22 Pi.ulo Pessoa,
Prob]en1as do Matemút ica. para o Ad 111i:-.: " Ctu 2 '.)
,,_)
O minuendo e o subtraendo foram a umentados de 439 Façamos o mesmo subtraindo-se do minuendo 35.
e o resto não se a lterou. Vem:
O mesmo acontecerá se subtrairmos do minuendo e
do subtraendo, a mesma quantidade. Nôvo minuendo 1968 - 35 = 1933
mesmo subtraendo 347 -
Vejamos:
Minuendo : 1968 Nôvo resto 1586 =
Subtraendo: 347 = (resto anterior) 1621 - 35 -\- 1586

Resto: 1621 3.ª propriedade


Se subtrairmos do minuendo e do subtraendo 29, por
e xemplo, virão: Somando-se ou subtraindo-se ao subtraendo uma ce rta
quantidade, o resto aparece diminuído ou aumentado da
Nôvo Minuendo 1968 29 1939 mesma quantidade.
Nôvo Subtraendo: 347 29 318
Consideremos ainda os mesmos minuendo e subtraen-
Resto (mesmo) 1621 do dados como exemplo e cujo resto é 1621 .

2.ª propriedade Aplicada a propriedade vem se somarmos 43 ao sub-


traendo:
Somando-se ou subtraindo-se ao minuendo uma certa
quantidade, o resto a p arece aumentado ou diminuído da Mesmo minuendo: 1968
m esma quantidade. Nôvo subtraendo: 347 + 43 390
Se ja m 1968 e 347 o minuendo e o subtraendo respecti- Nôvo resto: 1578 =
vamente.
(resto antigo) 1621 - 43 = 1578
O res to como vemos ó:
Minueudo: 1968
Se s ubtra irmos 43 ao subtraendo, vem:
Subtraendo : 34·7
Mesmo minuendo: 1968 -
He ·to: 1621 Nôvo subtraendo: 347 - 43 = 364
Nôvo re sto: 1664 =
Somemos ao minuendo 35, por ~x~mplo, s cousorvsruoa
o s ubtraendo. Virá: _ . . __ . . . . . .
(resto antigo) 1621 + 43 1664
Nóvo minuendo: 1968 + 35 = 2003
Mesmo subtraendo: 347 4.ª propriedade

Nôvo res to: 1656 A soma dos têrmos de uma s ubtração, isto é, minuen -
..::::. (resto anterior) 1621 +-35 _:_ 1656 do, subtraendo e resto é igual ao dôbro do minuendo.

l 'a. u lv J"<..' SS OH Problema s de Matemá Uca para o Admis sã o 25


Assim Affaim
Minuend o
Subtraendo:
1968
347
3 x A X 1 4 o 111esnio qus
4 X 3 X 7 ou 7 X 3 X 4
Resto: 1621 2 ....) Nwn produto indicado de vários fatôres, podemos
:.iub:;tituir vários fotôres pelo seu p:roduto (propriedade
1968 + 347 + 1621
associativa).
~-y---1

1968 + 1968 que é duas vêzes o minuendo. Assim:


Multiplicar é repetir um número chamado multiplican- 3 X 4 X 7 é o mesmo que
do, tantas vêzes quantas forem as unidades de outro de- 12 X 7 ou 3 X 28 ou
nominado multiplicador. O resultado da operação chama-se
4 X 21
produto. O multiplicando e o multiplicador são os fatôres
do produto 3. ª) Para multiplicar uma soma de várias parcelas por
Indica-se uma multiplicação empregando-se o sinal um nú,mero, é preciso multiplicar cada parcela ua &oma,
vêzes (X) entre os fatôres. por êsse número e soma r os resultados.
Assim 4 X 3 é uma multiplicação indicada; 4 é o mul-
tiplicando e 3 o multiplicador . AsiSim:
Pela definição dada acima, 4 X 3 é o mesmo que 4 + + 3 + 7) X 5 é o mesmo que
4 + 4, isto é, a repetição do multiplicando (4) tantas vêzes
(2
:.: X S -t- J X 5 -t- '/ X 6, i:;to é:
quantas são as unidades do multiplicador (3).
O resultado 12 é o produto. lU + 15 + 3::>
3 e 4 são os iatôres do prod"U:to. 4.u) O produto de d uas somas é igual à soma dos
Seja efetuar o produto indicado 132 X 15 . produtos ob.idos, multiplicando-se sucess•vamente as par·
Para obtermos o seu resultado (produto) costumamos ce!as aa primeira soma pelas da segunda.
dispô-los como se segue:
Assim:
- ·-
132 Multiplicando
X 15 Multiplicador
(2 + 3) (5 + 7) é o mesmo que
2 X 5 -t- 2 X7 + 3 X 5 + 3 X 7 = 60
- ·--aso 5.ª) Num produto indicado de vários fatôres, po~emos
132 substituir qualquer dos fatôres por outros que mulhphcados
tenham o seu valor (propriedade dissociativa).
1. 980 Produto
Assim:
Propriedades da multiplicação
24 X 35 é o mesmo que
l.ª) A ordem dos . fatôr_e.s não altera o produto (pro- ,..-A-, ,..-A-,
priedade comutativa). 3 X 8 5 X 7

i26 Paulo Pessoa Problemas de Matemática pura o Adm issão 27


- - ---- /\ -
6.u) O produto é da mesma espécie do multiplicador.
Assim em urna divisão cujo divisor seja 5, o maior
Assim, se o dia de trabalho de um operário é p ago à razã o
resto que se pode achar é 4, isto é, 5 - 1 = 4.
de NCr$ 2,00 por dia, no fim do mês o operário receberá
30 dias X NCr$ 2.00 = NCr$ 60,00 39 5
Dividir é verificar quantas vêzes as unidades de uma 4 7
das quantidades, chamada divisor, estão contidas na outra
quantidade conhecida pelo nome de dividendo.
Q uando as unidades do divisor se contém em nú- 3.ª) Multiplicando-se ou dividindo-se o dividendo e o
:mero exato de vêzes no dividendo, dizemos que a divisão divisor por um número, o quociente não se altera m as o
é exata, isto é, não há resto. resto fica multiplicado ou dividido por êsse número
O número de vêzes que o divisor se contém no divi-
dendo, chama-se quociente. Assim
Numa divisão há, portanto, dividendo, divisor, quociente 15 /_7
e resto .
Uma divisão é indicada pelo sinal de divisão h-) se- 1 2
parando o dividendo e o divisor. Se multiplicarmos 15 e 7 por 3, respectivamente teremos
Assim: 45 1 21
1 )~
15 -;.... 3; 15 -;.... 7. 3 2
(Dividendo) 15 J 3 (divisor) Como vemos o quociente continuou sendo o mesmo,
·isto é, 2, mas o resto que antes era um, passou a ser três,
(Resto) o 5 (quociente) isto é, apareceu também multiplicado por 3. No caso tere-
e mos 27 para dividir por 6, o seu resto será
(Dividendo) 15 7 (divisor) 27 1 6
1-
(Resto) 2 (quociente) 3 4
Pmpriedades da divisão Se dividirmos 27 e 6 por 3 passaremos a ter 9 e 2 e
n a divisão que fôr efetuada, o quociente continuará sen-
l.ª) O dividendo é igual ao produto do divisor pelo do 4 mas o resto será um, isto é 3 -;.... 3 --: 1, como
quociente, mas o resto (se. houver).
9 2
Assim no segundo exemplo dado, isto é, 15' -;.... 7
15 = 7 X 2 + 1 1 4

2.ª) O maior resto que se pode achar em uma divisão Nã o vamos, neste trabalho, por falta e cabimento, en-
é igual ao divisor menos uma unidade. sinar as quatro operações; entre:tanto aproveitaremos as
operações efetuadas para tirar suas provas reais.
.28 Paulo Pessoa
P rnl:!le nrno de Mut em;ítica par« o Ad mio"ilo 29
Então (soma do subtraendo e do resto) subtração certa.
Assim é que na soma
Na multiplicação:
1. 967
1.397 + 132 X 15 = 1. 980
105
Divide-se o produto 1. 980 por 132 ou 15 Se os resulta-
3.469 P,os forem 15 e 132, respectivamente, é sinal de que o
produto est ácerto.
A prova real é feita separando-se uma das parcelas e
subtraindo-se da soma total, a soma das demais. Se o Assim:
resultado encontrado fôr igual à parcela separada é sinal 1980 15 132 ou
de que a soma está certa. 1980 132 15.
Assim, separando-se por exemplo a parcela 1 . 967 e so- Na divisão:
mando-se as outras duas· encontraremos
Em virtude da 2.ª propriedade
1.397 (dividendo) 15 ·3 (divisor) X 5 (quociente) + O (resto)
105 (dividendo) 15 = 7 (divisor) X 2 (quociente) + 1 (resto)
Indica estarem certas as divisões efetuadas.
1.502

que subtraídos da soma total 3. 469, dará l . 967 (parcela EXERCfCIOS RESOLVIDOS
separada) se a operação estiver certa.
1) Numa adição de quatro parcelas a primeira é 3. 046;
Então a segunda é 810 e a terceira é 90. A soma de tôdas é 4. 292.
Calcular a quarta parcela.
3.469
1.502 A soma das três parcelas conhecidas é:

1967 (operração certa). Na subtração, se a 3.046


soma do subtraendo e do resto fôr igual ao minuendo é + 810
90
indicação de que a subtração está certa
3.946
Assim
Como a soma de tôdas é 4 . 292 e a soma das três é
3 .547 3 . 946, menor
- 132 1
----~ + -
4 .292
3.946
3.415 J
346
3. 547 (soma do subtwendo e do rasto)

30
Problemas de Matemática: para o Admissão 31
Paulo Pessoa
~ - - - --- - . !\
do que a soma total conclui-se que o vaior da última par- Houve, porém um êrro de 394 unidades para mais no
cela, isto é, a quarta, é 346. minuendo e no subtraendo, um êrro, também para mais
igual a 27 .
2) Um aluno ao escrever as duas parcelas de uma
soma enganou-se e escreveu a primeira com um êrro de Qual seria o resultado certo se não houvesse os êrros
347 para mais e a segunda também com êrro de 129 uni- apontados?
dades para mais.
Como o êrro, para mais, de 394 no minuendo, a dife-
Qual foi o êrro total cometido pelo aluno? rença encontrada, 708 . 356 ficam aumentada de 394. Se o
Tendo acrescido a primeira parcela de 347 unidades, foi êrro fôsse só êste o resultado certo seria:
como se houvesse introduzido nas quantidades a somar
708.356
uma parcela igual a 347. -- 394
Com a segunda parcela aconteceu o mesmo, acarre-
tando assim a criação de uma outra parcela igual a 129. 707.962
Assim sendo, a soma que tinha de início duas parcelas Tendo havido também um êrro para mais igual a 27,
acabou como se tivesse quatro, isto é as duas primeiras e no subtraendo significa que foi subtraído um número maior
mais outras duas iguais a 347 e 129, respectivamente . 27 unidades do que devia do minuendo e por isso o que
sobrou, depois da primeira correção, deve ser acrescido
Verifica-se então que a soma final foi acrescida de 'do que foi subtraído a mais.
347
+ 129 O resultado certo será pois:
476, que foi o êrro cometido na soma, para 707. 962
mais, pelo aluno. + 27
3) No mesmo problema os êrros cometidos foram na 707 .989
primeira parcela 347 para mais e na segunda 129 para me-
nos. Qual foi o êrro total na soma? O que foi dito pode ser esquematizado como se segue:

Pelo fato de ter errado para mais de 347 na primeira r.;Iinuendo + 394
parcela, segue-se que a soma sofrerá um aumento de 347. Subtraendo + 27

Em virtude de ter errado, na segunda, para menos 129 Resto 708. 356 (errado)
unidades, a soma ficou diminuída de 129 unidades . Então o
êrro total foi: Resto certo: 708. 356 - 394 + 27 = 707. 989

347 (para mais) 5) No mesmo problema anterior o serres cometidos


129 (para menos) foram: 1 . 394 unidades para menos no minuendo e, 327 uni-
dades também para menos no subtraendo. O resto encon-
218 (êrro final para mais) trado foi 345 . 901.
4) A diferença de dois 1núrneros foi 708. 356. Qual o resto certo?

32 Paulo Pessoa l'roblcma.s de Matemálica para o Admi ss ão 33


Errar para menos in o minuendo, implica em sobrar
menos no resto. Para compensar o resto torna-se necessá- Diminuir 15 ao minuendo implica em diminuir o resto
rio somar a ele o êrro cometido no minuendo, isto é l . 394. de 15.
Somar 7 ao subtraendo acarreta sobrar menos no resto.
Teremos então:
Como resultado final o resto ficará diminuído de:
345.901
+ 1.394 15 + 7 = ' 22.
:347.295 8) O minuendo foi aumentado de 13. 005 unidades,
que devemos fazer no subtraendo a fim de que o acrésci-
Por outro lado errar para menos no subtraendo im- mo do resto seja apenas de 36 unidades.
plica em subtrair menos do que devia, e portanto, no resto Aumentar o minuendo de 13. 005 unidades implica em
sobrará mais. aumentar o resto de 13. 005.
Para compensar é preciso retirar do resto o que foi Para diminuir o resto é preciso aumentar o subtraendo.
somado a me nos , isto é 327 e teremos: Como queremos que o resto fique aumentado apenas
347 . 295 de 36 unidades é necessário que o subtraendo seja au-
327 mentado apenas de:
13.005
346.968 36
6) Que alteração sofre o resultado de uma: subtração
(resto) quando aumentamos o minuendo de 19 e diminuí- 12.969
mos o subtraendo de 15? 9) Quantas unidades ficam somadas ao número 31
Aumentar o minuendo de 19 é o mesmo que concorrer quando o multiplicamos por 12?
para que no resto sobre mais 19 unidades. Mutiplicar o número 31 por 12 é repeti-lo 12 vêzes.
Diminuir o subtraendo de 15 é o mesmo que subtrair
um número menor 15 unidades, concorrendo para que Como antes da repetição o número 31 já existia, se-
sobrem no resto mais 15 unidades. gue-se que é necessário escrevê-lo mais 11 vêzes o que é
Considerando··se os dois efeitos, conclue-se que o resto o mesmo que multiplicá-lo por 11 e teremos:
ficará aumentado de: 31 X 11 = 341.
19 10) Que altteração sofrerá a soma 100 -\--90 = 190 se
+ 15 multiplicarmos a primeira parcela por 19.
34 Depois do que foi dito no problema anterior multipli-
C'Or 100 por 19 é repetir 100 dezenove vezes; ou seja somar
7) Que alterar-ão sofre o resto de uma subtração n 100 mais dezoito parcelas iguais a ela, isto é, multiplicar
quando subtraímos 15 unidades ao minuendo e sarnamos l 00 or 18, que é igual a 1. 800.
5 unidades ao subtraendo? Êsse resultado será a alteração .sofrida pela soma
100.
34 Pa.ulo Pessoa
11) Sabendo que 15 X 30 = 450, dizer quantas uni- 36. Tendo achado um resultado menor 34. 812. por não
dades há a mais no produto 15 X 32, sem efetuá-lo. ter multiplicado por 36, segue-se que o número que não
foi multiplicado é:
Podemos escre ver: 34.812--;- 36 = 967
15 X 32 = 15 X (30 + 2)
14) Se o produto de dois números é 1.113. qual será
Como foi explicado anteriormentE o produto de um nÚ."llero 5 vêzes maior que o primeiro
fotor, por outro 7 vêzes maior que o segundo?
15 X (30 + 2) = 15 X 30 + 15 >< 2 (1)
Se apenas o primeiro fator fôsse 5 vêzes maior que no
Considerando q ue o produto inicial era 15 X 30 = 450, ~nício, o resultado seria S vêzes maior.
na igualdade (1) êsse produto aparece acrescido de 15 X 2 Como o segundo também tornou-se 7 vêzes maior o
= 30, que é a alteração sofrida pelo produto a calcular produto será sete vêzes maior.
sem efetuá-lo. Ficando 5 vêzes maior em virtude da alteraçã o no pri-
meiro fator e 7 vêzes maior em razão de alteração do se-
12) Sem efetuar a multiplicação 15 X 14, calcular gundo, o produto ficará 5 X 7 t= 35 vêzes maior do que
quantas unidades êsse produto excede 14 )( 13. era, ou seja:
Como no exemplo a nterior, podemos escrever: 1.113 X 35 == 38 .955
14 X 13 = 05 - 1) (14 - l)
15) Um aluno efetuou a multiplicação de 213 X 105 e
e como já foi mostrado seu resultado é 1escreveu o segundo produto parcial sob o primeiro deslo-
~ando-o para a esquerda uma única ordem . Determinar o
15 X 14 - 15 X 1 - 1 X 14 + êrro sem refazer a operação.
O que o aluno fêz foi multiplicar o número por 15 em
A
lugar de multiplicá-lo por 105. Deixou portanto de multi-
Ve-se assim qi.:e o produto primitivo 15 X 14 ficará plicar 231 por 105 - 15 =
90.
alterado em virtude da parte assinalada com a letra A, Para refazer o êrro é bastante multiplicar 231 por 90
isto é: e somar o resultado ao encontrado primitivamente.

- 15 - 14 + 1= - . 28 O êrro comelido foi de:

13) Um aluno ao multiplicar um número por 40 mul- 231 X 90 = 20.790


tiplicou-o por 4 e esqueceu-se de colocar o zero à direita
do produto, cometendo um êrro no resultado de 34. 812, lG) O menor de dois números é: 2. 082. A diferença
~ntre êles é o quintuplo do menor. Caicular o maior.
para menos. Que número o aluno deveréc ter multiplicado
por 40? Se a diferença entre os dois números 'Cujo menor é
Não colocando o z 1 à direita do produto foi como ~. 082, é o quíntuplo do menor, é fácil concluírmos que o
se houvese multiplicado número penas por 4. Como menor somado com a diferença dá um resultado igual a seis
devia multi} Ecá-lo por 40 deixou de fazê -lo por 40 - 4 = vezes o menor .

l'1"oblemas de Matemática para o Admis"ão 37


Vimos tcxm.bém que a soma do menor (subtraendo' co.trl Se os dois fôssem iguais ao menor a soma dêles seria:
u diferença (resto) é igual ao minu ndo.
3 . 935 - 1 !=, 3.934
EntÕ<; o mo.ivr du · d0 1·· r.úmoros (mwut.hd
vêzes o menor, h to e: e conseqüentemente um dêles teria o valor:
2.082 X 6 = 12.492 3. 934 -7- 2 = 1. 967.
17) Paulo e Pedro tem quantias iguais. Paulo dá a
Conhecido o menor e sabendo que são consecutivos
Pedro NCr$ 4,üU e recebe d e .h~d.o NCr'.ai 9.UO.
é fácil concluir que o maior será 19G7 1 1968. + =
Dizer quem ficou com a maior quantia e quan ~o mais'?
20) A soma de três números ímpa..res consecu tivos é 63.
Quando Paulo daa l~Gc.,i: 4, Uu a Pedro, ficou com me-
Achar os três números.
nos NCr$ 8,00 do que aquêie.
Por outro lado, quando recebeu NCr$ 9,00 de Pedro fi- Dois números ímpares e consecutivos diferem de duas
cou com NCr$ 18,00 mais do que êle. unidades.
Sendo três números, a diferença do menor para o se-
A diferença entre as dua s situações:
gundo é 2 e a diferença para o terceiro, 4.
NCIS 18, úO - NCr$ 8, 00 .= NCr~ 10, 00
Para que todos se tornem iguais ao menor é necessá··
mostra que Paulo ficou com rnais NCr$ 10,00 do ,q ue Pedro. iio tirar 2 ao segundo e 4 a o terceiro.
18) Num ta que entram 36 litros dágua por minuto Conseqüentemente a soma dos três números diferen-
e se escoam 14 litios no mesmo tempo. No fim de 5 hcta:> tes (63) passará a ser a soma de três números iguais ao 1
quantos litros haverá no tanque'? menor; não será mais 63 e sim:
Se entram S6 iitros e saem 14 litros no mesmo tempo, 63 - 6 = 57
ticam no reservatório:
Os 1 úmeros iguais valerão:
36 - 14 = 22 litros por minuto.
57 + 3 = 19, que é o menor.
Como cada hora tem 60 minutos, em 5 horas existirão
5 X 60 minutos = 300 minu·os. Fácil será determinar os outros dois, pois êles são ím-
pares sucessivo.s e p or isso serão:
Então em 5 horas ou 300 minutos estavam contidos no
tanque: 19, 21 e 23.
22 litros X 300 minutos = 6. 600 litros.
21) João emprestou NCr$ 1. 578,80 à Pedro.
19) Achar dois números inteiros e consecutivos, cuja
soma é 3.935. Se houvesse emprestado mais NCr$ 411. 00 teria ficado
Dois números inteiros e consecutivos diferem de uma r·om NCr$ 20.000,00. Quanto tinha João antes do emprés-
unidade. timo?

33 Pa.ulo Peosoa l'roulcma" uc MaLcmáLica· l'""" o A<l11dtitiÚo 39


O empréstimo inicial foi de NCr$ 1. 578,80. Se houvesse 24) A soma de dois números é 867 e a diferença en-
emprestado mais NCr$ 411.00, o empréstimo total seria de: tre êles é 253. Quais são os números?
NCr$ 1. 578. 80 Em problemas dêsse tipo convém ter em mente as se-
NCr$ 411. 00 guinte regra:
NCr$ 1 . 989. 80 A soma da soma com a difnença dos dois números
é igual ao dôbro do maior número.
Depois disso João ainda teria NCr$ 20. 000,00.
A diferença entre a soma dos dois números e sua dife-
t fácil concluir-se que se não houvese efetuado o em- rença é igual ao dôbro do menor dos números.
préstimo teria:
Então 867 (soma dos dois números) +
253 (diferença
NCr$ 20 . 000,00 + NCr$ 1 . 989,80 dos dois números) é igual ao dôbro do maior dos números.
ou seja: Assim:
NCr$ 21. 989,80.
22) Pagou-se uma dívida de NCr$ 310, 00. Se houvés- 867 + 253 = 1.120 (dôbro do maior número) e o maior
númer.o será:
semos pago menos NCr$ 64.00 teríamos ficado com ... .
NCr$ 15. 000,00. 1. 120 --:-- 2 = 560 .
Quanto tinha o devedor? Do mesmo modo:
Se em lugar de pagar NCr$ 310. 00 houvesse pago 867 (soma dos dois números) - 253 (diferença dos dois
menos NCr$ 64,00, o pagamento teria sido de NCr$ 310,00 11úmeros) é igual ao dôbro do menor dos números.
- NCr$ 64,00 = NCr$ 246,00.
Assin
Se depois de efetuar o pagamento ainda ficou .com
NCr$ 15. 000,00 é sinal de que antes do pagamento tmha 867 - 253 = 614 (dôbro do menor dos números) e o
NCr$ 15 .000,00 +
NCr$ 246,00 = 1NCr$15.246, 00. menor número será:

23) Paulo efetuou um pagamento de NCr$ 4.000,00. 614 --:-- 2 = 307.


Se em vez de efetuá-lo todo ficasse devendo a metade po- 25) A soma de dois ·números é 28 e um dêles é o tri-
deria adquirir um objeto de NCr$ 8. 745.00. plo do outro. Achar os dois números.
Com quanto ficou Paulo depois de efetuar o pag_?-
mento? Se o maior dos números é três vêzes o menor, êle
(maior) somado com o menor corresponde a qua1ro vêzes
A metade do pagamento corresponde a NCr$ 2. 000,00. o menor, que o problema diz ser 28 (a soma).
Se fôssem pagos só NCr$ 2 . 000 00 restariam:
Então para acharmos o menor é bastante dividir a so-
NCr$ 8 . 745,00 rna 28 por 4 e acharemos que o menor vale 7.
Considerando que o maior é o triplo do menor conclui-
Então tendo sido pagos os NCr$i 4. 000,00 sobraram: oe 1ue o maior. será 3 X 7 = 21.
NCr$ 8.745,00 - NCr$ 2.000,00 = NCr$ 6.745,00. Os números são pois: 7 e 21.

4.0 ·ri a.u lo Pesi:; oa. Prohlcmn.s de Matcmúti ca· pnrn o J\dmiR~iío <J!l
26) A diferença de dois números é 288 e o menor é 5 28) A soma de dois números é 66. O menor dos núnlé•
vêzes o maior. Achar os dois números. ~os é igual a quarta parte do maior mais 16 . Quais são
Se o maior é 5 vêzes o menor, a diferença entre 5 vêzes os números?
o menor e o menor, é 4 vêzes o menor. Essa diferença, diz Se o menor dos números fôsse apenas a quarta parte
=
o problema é 288. Então o menor será 288 ...;-.- 4 72. d.o maior, a soma dos dois números não seria 66 e sim:
O maior, que é 5 vêzes o menor, será: 66 - 16 = 50.
72 X 5 = 360. Se o menor é a quarta parte do maior é porque o maior
Os números são pois: 72 e 360. é ,q uatro vezes o menor.
A soma dos dois, isto é: quatro vêzes o menor mais
27) Um pai tem 31 anos e seu filho 13. Há quantos o menor dá 5 vêzes o menor cuja soma é 50. Então o menor
anos a idade do pai foi o quádruplo da do filho?
Se o filho tem 13 anos o acontecimento não está se
seria 50 -;- 5 = 10.
realizando, pois para isso era necessário que o pai tivesse Acontece porém que o menor é a quarta parte do maio1
4 X 13 =52 anos . Considerando que êle tem 31 para (10) mais 16, isto é
que o fa to estivesse acontecendo era necessário que o pai w +
16 = 26.
tivesse 52 - 31 ·= 21 anos mais do que tem. Como temos a soma de dois números igual a 66 e um
Considerando que a idade do pai é 4 vêzes a do filho dêles 26, conclui-se que o outro será:
(no instante que se procura), a diferença entre suas idades
1será 3 (4 -- 1) vêzes a idade do filho e que no caso do 56 - 26 = 40.
problema seria 21 anos, se o instante fôsse o atual. Os números são pois: 26 e 40.
Então: 29) A soma de dois números é 182. O maior é o quá-
21 + 3 7 anos (ocasião em que se deu o fato da druplo do menor, menos 8. Quais são os números?
idade do pai ser o quádruplo da do filho). Se o maior fôsse exatamente o quádruplo do menor
(néio faítasse 8), a soma dos dois números seria 5 vêzes o
Outra solução menor dêles e valeria:
No momento a diferença de idades entre pai e filho é 182 +8= 190.
31 - 13 = 18 anos. Então o menor dêles seria:
A iàade do pai sendo quatro vêzes a do filho, a dife-
rença entre elas é igual a 3 vêzes a do filho.
190 + 5 = 38.
O maior seria:
Então: 4 X 38 = 152.
18 + 3 =
6 anos que será a idade que o filho terá. se não faltasse 8 para ser 4 vêzes o menor. Então o maior
Como êle tem 13 anos e o fato se deu quando êle tinha será:
6 anos, conclui-se que tudo se passou quando êle tinha: 152 ~ 8 = 144
13 - 6 = 7 anos. Os números são pois 38 e 144.

Problem>ts de Matemá tico: para o Admissão 43


30) Um pai tem 60 anos e seus filhos 14, 16 e 18 anos. Se ao terminar a série de tiros os dois tinham quantias
No fim de quanto tempo a ídade do pai será igual à soma iguo:1s, cada um tinha:
das idades dos filhos? NCr$ 48, 00 -7 2 = NCr$ 24, 00
A diferença entre as idades do pai e a soma das ida-
des dos filhos é: O primeiro tinha perdido:
60 - (14 + 13 + 18) 1 1 60 - 48 = 12 NCr$ 36, 000 - NCr$ 24, 00 = NCr$ 12. 00
Cada ano que se passa a idade do pai aumenta um Cada acêrto valendo NCr$ O, 20, o número de tiros que
ano e a soma das idades dos filhos aumenta 3. Em cada o segundo acertou mais que o primeiro, foi:
ano, portanto, a diferença de 12 anos diminui de:
NCr$ 12, 00 -7 NCr$ O, 20 =:60
3 - 1 = 2.
Para que a diferença de 12 anos desapareça serão ne- 33) As idades de duas pessoas somam 80 anos. Sub-
cessários: traindo-se 15 anos da idade da mais velha e acrescentan-
12 -7 2 = 6 anos . do-os à da mais nova, as idades tornam-se iguais. Qual a
idade de cada uma?
31) Um avicultor leva ovos para vender por NCr$ 0,10
cada um. Tendo quebrado na viagem 15 ovos vendeu en- Se as idades de ambas somam 80 anos e tornam-se
tretanto, os restantes por NCr$ O, 12 cada um, obte11do a.ssim iguais com a passagem de 15 anos de uma para outra,
uma va11tagem de NCr$ 4,20 em relação à venda de todos segue-se que cada uma ficou com 40 anos.
que levCl\vja por NCr$ p,w,., se não quebrasse nenhum.
Para que a mais môça, depois de receber 15 anos fi-
Quantos ovos levava o avicultor?
casse com a idade de 40 anos, era preciso que tivesse
O a créscimo de NCr$ 0,12 - NCr$ 0,10 = NCr$ 0,02 antes:
no prêço de cada ôvo, acarretou na venda d os restantes
um lucro de NCr$ 4,20. Se não houvese quebrado nenhum 40 - 15 = 25 anos.
e resolvesse vendê-los a NCr$ O, 12 o lucro seria :
Por outro lado, e depois de dar 15 anos, a mais velha
NCr$ 4,20 +
15 X NCr$ 0,12 = • NCr$ 6,00, que teria ficou com 40 anos é porque antes da doação tinha:
sido conseguido com a venda de NCr$ 6,00 -7 NCr$ 0,02
= 300 ovos. 40 + 15 = 55 anos.
32) Dois atiradores vão fazer tiro ao alvo valendo ca- 34) Como pagamento de uma dívida ofereceram dois
rla acêrto NCr$ 0,20. De início um tinha NCr$ 36,00 e 0 1 outro objetos a escolher, que valiam juntos NCr$ 20,00. Caso acei-
NCr$ 12,00, mas ao terminar a série de tiros, o primeiro tassem o primeiro dos· objetos, o devedor ficaria a inda. com
tinha a mesma importância que o segundo. d ébito de NCr$ 2, 00 . Se o segundo fôsse preferido, o credor
Quantos acertos o segundo obteve mais que o pri- teria que devolver NCr$ 2, 00 ao devedor . Qual o valor de
meiro? r-u a objeto e qual a importôncia da dívida?
A importância dos dois era: F~vid nt01nente, o seg undo oi joto é mais valio:;o que
NCr$ 36,00 +
NCr$ 12,00 NCr$ 48,00. u pllmeiro.

44i Paulo Pessoii


A diferença de valôres entre um e outro é fácil de ver, A importância a dividir deverá ser dividida em 5 par-
corresponde a: 1es (1 da primeira e 4 da segunda) e leremos então para
a primeira:
NCr$ 2,00 + NCr$ 2,00 = NCr$ 4,00,
NCr$ 2 .000, 00 --:- 5 = NCr$ 400. 00 .
pois com a entrega do segundo objeto em vez do primeiro,
ps NCr$ 2, 00 que ficariam faltando (no caso da entrega A segunda receberá o quádruplo da primeira, ou seja:
do primeiro) foram pagos e ainda sobraram NCr$ 2. 00 .
Como o.s dois objetos valem juntos NCr$ 20,00 e um 4 X NCr$ 400.00 = NCr$ 1.600.00.
vale mais que outro NCr.$ 4,00, teremos agora um proble-
ma igual ao de n. 0 24, isto é, a soma de dois números 37) A soma de dois números é 158 e a divisão de um
é NCr$ 20,00 e a diferença NCr$ 4,00. pelo outro dá 13 para quociente e quatro para resto. Quais
são êsses números?
Os objetos valem então:
Se a divisão de um pelo outro desse 13 para, quociente
l.º objeto: 20,00 - 4,00 ie não houvesse resto, a soma dêles não seria 158 e pode-
= NCr$ 8, 00 ríamos dizer que um dos números era 13 vêzes o outro.
2
A soma ser.ia:
2. 0 objeto 20. 00 + 4, 00 = NCr$ 12, 00
158 - 4 = 154.
2
Os dois somados seriam 14, (1 + 13) vêzes o menor,
Se o recebimento do segundo objeto implica na res- que por sua vez seria igual a:
tituição de NCr$ 2, 00 ao devedor, conclui-se que a dívida 154 --:- 14 = 11.
era de:
Se o menor é 11 e a soma dêles é 158, segue-se que o
NCr$ 12,00 - NCr$ 2,00 = NCr$ 10.00 outro será:
158 - 11 = 147.
Multiplicando-se um número por 5', fica aumen-
35)
tado de 1. 316 . Qual é êsse número? ·33) Um trem carrega 410 passageiros de l.ª e 2.ª
classes e apura o total de NCr$ 453,00. A passagem de l.ª
Multiplicar um número por 5, é o mesmo que somar custa NCr$ I.50 e a de 2.ª NCr$ 0,90. Quantos eram os pas-
ao número o quádruplo do seu valor. Foi por isso, então sageiros de l.ª e 2.ª classe?
que o número tornou-se maior l. 316. Ora, o quádruplo de
um número sendo 1.316, o número será: Se todos os passageiros fôssem de l.ª classe a arre-
cadação teria sido de:
1 . 3!16 --:- 4 = 329.
Dividir NCr$ 2 :000,00 entre duas pessoas de modo
36)
410 X NCr$ l, 50 = NCr$ 615, 00
que uma receba o quádruplo da outra. Como foi apenas de NCr$ 453, 00 houve uma diferença
Uma â.as pessoas recebendo uma parte a outra rece- de:
berá 4 vêzes mais, ou sejam, 4 partes . NCr$ 615,00 - NCr$ 453.00 = NCr$ 162,00.

>-16 Pa·u lo Pess oa Problemns de Matemática para o Admissão 47


Tal difere!lça decorre do número d e passageiros de 2.ª Sabemos que o dividendo é igual ao divi;:;or multipl!-
transpor tados e que p agaram: cn do pelo quociente, mais o resto. Então:
NCr$ L 50 - NCr$ O, 90 = NCr$ 0,.60 menos. Dividendo = 127 X 43 + 22 5.483.
Cada NCr$ 0,60 corresponde, pois, a um pasageiro de 42) Numa d ivisão o resto é 12 e o q uociente 34. A
2.ª e êles foram tantos que produziram uma diminuiçã o d a diferença entre o divi<lendo e o divisor é 540. Achar o di-
r eceita de NCr$ 162, 00 . videndo e o divisor.
O se u número foi então:
Se nã o hou.vessQ o resio 12, a diferença entre o dividendo
NCr$ 162.00 -:-- NCr$ 0.60 270 e = o o divisor seria:
540 -- 12 = 528.
os de l.ª classe: 410 - 270 = 140.
39) A soma de três números que figuram numa sub- O quocien te sendo 34 signi.fica q uG a diferença entre
tração é 1. 578. O minuendo excede o res to de 532 unida- ~les seria 34 - 1 = 33 vêzei:: o divisor e conseqiümte;rwntc
&le seria:
des. Determin ar êsses números.
528 ...:.... 33 = 16.
Vimos que a soma dos têrmos de u ma subtraçã o é igual
ao dôbro do minuendo. Considerando porém que por haver resto, a diferença
e ntre o dividendo e o divisor é 540, segue-se que o divi-
Então o minuendo é: dendo será:
1 . 578 -:-- 2 = 789. Dividendo: 16 X 34 12+ 556 em virtude do que
foi dito no problema anterior .
Como êle excede o resto de 532, o resto será:
43) Em uma divisã o de números inteiros o dividendo
789 - 532 = 257. é 92, o resto 7 e o quociente não é l. Ca lcule o divisor.
Conseqüentemente o subtraendo será: I. E. - 1968

789 - Q57 = 532. Como temos procedido de outras vê zes, concluímos que
se não houver resto (7) o dividendo s·eria :
40) Numa divisão o dividendo é 5. 328; o quocien-
te 23 e o resto 15. Qual é o divisor? 92 _.:... 7 = 85.

Se não houvesse resto o dividendo seria: O quociente não sendo 1 e não have ndo resto temos
que ca lcular dois números que multiplicados dêem 85 e
5.328 - 15 = 5.313. acharemos 5 X 17. Como o resto é 7 o divisor não p ode
ser 5 e e ntã o só poderá ser 17.
Como 0 quociente é 23 segue-se que o divisor será:
44) Tem-se um produto de três fatôres . Com a mul-
5. 313 -:-- 23 = 231. 1iplicação do primeiro fator por 4; do segundo por 6 e di-
vidido o te rceiro por 3, o produto fica aun1entado de 1 . 260
41) Num.a divisão o resto é 22; o quocien te 43 e o
unidades. Qual é o produto.
d ivisor 127. Qual é o dividendo?
C. Pedro II - 1968

ProbJe nrno de Matemática· para o Ad m !s::;; ão 49


O primeiro fator sendo multiplicado p or 4 o produ to O menor número que se deve acrescentar a o dividen·
Jica 4 vêzes maior. do para que a divisão seja exata é :
O segundo fator sendo multiplicado por 6, o produto 71 - 14 =! 57.
fica 6 vêzes maior.
O menor número que se pode acrescentar ao dividen-
Se não houvessem outra alteração o produto ficaria d o sem alterar o resto é 71, pois o quociente passará a
4 X 6 = 24 vezes maior. r.er 8 mas o resto continuará sendo 14.
Considerando que o terceiro fator foi dividido por 3 o 46) Numa divisão o resto é 51; o quociente é o dô-
produto final ficava aumentado de: bro do divisor que é o menor possível. Qual é o dividendo?
24 -;- 3 = B vêzes. Se o resto é 51 e o divisor é o menor possível, só po-
o problema diz que o produto ficou aum.antado de derá ser 52 (mais um que o resto).
1. 260 unidades. Dividendo =
52 X (2 X 52) + 51 .
ficar aumentado oito vêzes é ser acrescido de 7 vêzes o Dividendo = 5. 459.
produto primitivo. 47) Um criador comprou 10 cavalos e 8 bois por NCr$
240, 00. A seguir adquiriu 5 cavalos e 12 bois pela mesma
Então 1. 260 unidades corresponde a 7 vêzes o p roduto
importância. Qual o preço de cada animal?
primitivo e então êle será:
Podemos escrever:
1 . 260 -;- 7 = 180.
l.ª compra 10 cavalos + 8 bois = NCr$ 240,00
45) Numa divisão o dividendo é 511 e o divisor 71. 2.ª compra 5 cavalos + 12 bois =
NCr$ 240,00
Escreva:
Se a segunda compra houvesse sido o dôbro da que
a) 0 maior número que se pode acrescentar ao divi- foi, o comprador teria comprado 10 cavalos; 24 bois e teria
dendo sem alterar o quociente. . <J asto NCr$ 480,00.
b) o menor número que se deve acrescentar ao di-
v idendo para que a divisão seja exata. Poderíamos escrever:
e) o mneor número que se pode acrescentar ao divi- l.ª compra: 10 cavalos +
B bois = NCr$ 240,00. Com-
àendo sem alterar o resto.
C. Pedro II - 1968
pra que nodia ter sido feita da 2.ª vez: 10 cavalos 24 +
b ois = NCr$ 480, 00.
Efetuando-se a divisão de 511 por 71 encontraremos: Comparando-se as duas, verifica-se que em ambas os
cavalos comprados teriam sido em igual número, isto é,
511 71 1O, mas na segunda compra haviam roais 16 bois (24 - 8
16) do que na primeira e por isso o gasto teria sido
(resto) 14 7 (quociente). maior. NCr$ 480,00 - NCr$ 240,00 =
NCr$ 240,00 em con·
O maior número que se pode somar ao dividendo sem ucqüência da compra de mais 16 bois .
alterar o quociente é: Então cada boi custou:
71 - 14 - 1 = 56 NCr$ 240,00 -;- 16 = NCr$ 15,00.

501 Pa.nlo P es soa


Como na primeira compra foram adquiridos 8 bois, o enunciado fala em produto de 4 números e só dá 3.
conclui-se que êles custaram: É preciso, portanto desdobrar um dos fatores no produto
de dois outros.
8 X NCr$ 15,00 = NCr$ 120,00
Considerando que o problema pede o produto das quar-
e como a compra total foi de NCr$ 240, 00 conclui-se que tas partes dos fatôres é preciso que todos êles sejam di-
os 10 cavalos custaram: visíveis por 4 . Assim sendo os quatro fatôres serão:
NCr$ 240,00 - NCr$ 120,00 = NCr$ 120,00 28 X 20 X 4 X 4 e o p1oduto de suas quartas partes:
·e conseqüentemente cada cavalo custou:
7 X 5 X l X 1 =
35.

NCr$ 120,00 + 10 = NCr$ 12,00.


EXERCíCIOS PARA RESOLVER
48) Num terreno há galinhas e coelhos num total de
38 cabeças e 102 pés. Quantas são as galinhas e quantos ~ Ao pretender conferir uma soma, en.ganei-me e
são os coelhos? .acr~ntei a soma às parcelas, encontrando 7 . 658. Dizer:
a) qual a soma; b) quantas dezenas ela contém.
Se só houvessem galinhas, isto é, se os coelhos íôssem
galinhas o total de cabeças continuaria sendo 38, mas o de C. Pedro II 1968
pés não seria 102 e sim 38 X 2 = 76. Resposta; 3 . 829 e 382.
Porque existem galinhas e coelhos é que o número de 6!i\ Quatro estudantes foram fazer um passeio e gas-
pés é maior. tara~"'l"Cr$ 10. 573,80.
O primeiro gastou NCr$ 2. 985,50. O segundo gastou
102 - 76 = 26. NCr$ 3. 837, 20 e o terceiro gastou NCr$ 1. 899, 90. Quanto
Isso acontec.:e porque as galinhas têm 2 pés e os coe- gastou o quarto?
lhos 4 pés, ou seja mais 4 - 2 = 2 pés mais que cada ga- Resposta: NCr$ 1. 851.20.
linha.
Por isso houve um aumento de 26 pés em virtude de
@ A soma de três números é 3. 967. O primeiro e o
segundo são respectivamente 1. 111 e 2 . 345. Qual é o ter-
cada coelho ter mais 2 que cada galinha.
ceiro?
Então os coelhos são: Resposta: 511.
26 -7- 2 = 13. @ Um aluno ao escrever as três parcelas de uma soma
Se o total de a nimais era 38 e os coelhos são 13 con- enganou-se e escreveu a primeira com um êrro d e 129 para
clui-se que as galinhas serão: menos; a segunda com um êrro de 347 para mais e a ter-
c:eira sem êrro.
38 - 13 = 25. Qual foi o êrro total cometido?
49) O produto de quatro números é 28 ;< 20 X 16. Resposta: 218 para mais.
O produto de suas quartas partes qual é?
5) Ao fazer uma compra de 3 objetos, separadamen-
C. Militar - 1U68 t , o trôco foi dado erradamente. No primeiro objeto com
um êrro de NCr$ 5,00 pata mais. Ne sêgundo objeto conl <@ o minuendo foi diminuído de 27; q ue devemos
fozer no subtraendo a fim de que o resto fique a umentado
um êrro de NCr$ 2,00 para mais e no terceiro objeto orn
um êrro de NCr$ 10,00 para menos. de 36?
Qual foi o êrro total no trôco? Resposta: Subtrair 63 ao subtraendo .
Resposta: NCrS 3,00 para menos. {l) Quantas unidades ficam somadas ao número 53,
6) A diferença de dois números é de 1. 005. 304. Hou- qua'fr.;l o multiplicamos por 11?
ve um êrro de 1. 000 . 000 de unidades para mais no mi- Resposta: 530.
nuendo e no subtraendo um êrro de 5 unidades também
para mais. /7:;;\ Quantas wlidades ficam somadas ao número 127
qu~ o multiplicamos por 5?
Qual seria o resultado certo se não houvessem os erros
Resposta: 508.
apontados.
.kX Que alteração sofrerá a soma
Resposta: 5 . 309.
7) No mesmo problema anterior os erros cometidos
e:;; 105 + 10 + 25 = 140
se multiplicarmos a segw1da parcela por 20?
;foram 1. 295 unidades para menos no minuend o e :lü. '13'i
Resposta: 190.
unidades também para menos no subtraendo. O resto en-
contrado foi 980. 5lH. Qual o resto certo. @ Que alteração sofrerá: a soma
205 + 11 + 27 = 243
Hesposta: 986 . 862.
se multiplicarmos a terceira parcela por 11?
@Diminuindo 13 do minuenclo e aum entando 17 ao
Resposta: 270.
sub traendo, que alteração sofre o resto?
Resposta: Fica menor 30.
~Sabendo que 14 X 29 - 406 dizer quantas uni.-
dad8á a mais no produto 14 X 35, sem efetuá-lo.
@ Diminuindo 14 unidades do minuendo e 18 unida- Resposta: 84.
des ao subtraendo, que alteração sofre o resto?
18) Sabendo que 130 X 15 = 1950, dizer quantas uui-
R esta: fica maior 4 unidades. dades há a mais no produto 130 X 17.
Que alteração sofre o resto de uma subtração Resposta: 260.
quan aumentamos 21 unidades no minuendo e diminuí-
mos 15 unidades no subtraendo? 19) Sabendo que 140 X 12 =
1.680, dizer quantas
unidades há a menos no produto 140 X 10, sem efetuá-lo.
Resposta: Aumentado de 36.
Resposta: 280.
~ O minuendo foi aumentado de 1. 000 unidades.
Que~vemos fazer no subtraendo a fim de que o resto 20) Sem efetuar a multiplicação 18 X 12, calcular
fique aumentado apenas de 50 unidades? q uantas unidades êsse produto excede 16 X 10.

Resposta: Aumentou 950 ao subtraendo. Resposta: 56.

54 Pa.ulo Pessoa
21) Sem efetuar a multiplicação 17 X 15 dizer quan- Q O menor de dois números é l . 968. A diferença
tas unidades êsse produto excede i5 X 14. en~s é o triplo do menor. Calcular o maior .
Resposta: 45. Resposta: 7. 872.
29) Joel e Fernando têm quantias iguais. Joel dá a
22) Um aluno ao multiplicar um número por 80 mul- Fernando NCr$ 8,00 e recebe de Fernando NCr$ 18,00. Di-
tiplicou-o por 8 e e.squeceu-se de colocar o zero à direita zer quem ficou com a maior quantia e quanto mais.
do produto, cometendo um êrro no resultado de 141.696
para menos. Que número o .aluno deveria ter multiplicado Resposta: Joel ficou com NCr$ 20,00 mais do que Fer-
por 80? n ando.
Re ~p osta: 1 . 868. 30) Duas pessoas têm quantias iguais. Calcular quan-
to um dE:ve dar a o outro para que êste fique com NCr$
23) Uma p essoa deveria multiplicar u_rn número por 32,00 mai
65. Enganou-se e multiplicou-o por 35. O p roduto encon- Resposta: NCr$ 16,00.
traào tinha 420 unidades menos do que de via. Determinar
êsse número. 31) Um reservatório contém 5.824 litros dág ua. Uma
Resposta: 14. torneira esgota 67 litros por hora. Que quantidade de água
haverá no reserv atório no fim de 15 horas?
1

J::\ Se o produto de dois números é 2. 041, qual será Resposta: 4. 819 litros.
o ~o de um número 3 vêzes maior que o primeiro fa-
tor, por outro 4 vêzes maior que o segundo? 32) Um tanque contém 2 . 520 litros dágua. Tem uma
torneira que lhe fornece por hora 1. 892 litros e outra que
Resposta: 24. 492. esgota l. 532 litros no mesmo tempo. Quantas horas serão
necessárias para o tanque ficar cheio?
&'\Se o produto de dois números é 2. 826, qual será
o p~to de um número 3 vêzes maior que o primiero por Rasposta: 7 horas.
outro 6 vêzes menor que o segundo?
Resposta: 1.413.
6.i)) A soma de dois números inteiros e consecutivos
é 7 . 093. Achá-los.
26) Um aluno efetuou a multiplicação de 189 X 203 Resposta: 3 . 546 e 3. 547.
e escreveu o segundo produto parcial sob o primeiro, des-
locando-o para a esquerda uma única ordem. Determi·
Q A soma de três números inteiros e consecutivos é
2 . 703. Calcular êsses números.
nar o êrro sem fazer a opera ção .
Resposta: 900; 901 e 902.
Resposta: O êrro foi de 34. 020.
27) O menor de dois números é 196. A diferença en- ~A soma de 3 números pares consecutivos é 2. 706.
tre êles é o quádruplo do menor. Calcular o maior. Achar êsses números.
Resposta: 980. Resposta: 900; 902 e 904.

Pl'obl~mn.• de Matemátic<t para o Adml5são 57


~A som;i de, três números ímpares consecutivos e
·o que pcsauo. Quanto custam o rádio, a televisão e quem'"
Sl. ~r os tres numeros.
u possuo?
Respos:a: rádio, NCr$ 300.00; televisão, NCr$ COO, 00;
Res posta: 15, 17 e 19. lJU:>suo NCr$ 420,00.

-7Q A soma de 4 números ímpares consecutivos é 35. 44) Cyro ei.nprestou NCr$ 78,90 a Fernando. Se ti-
Ach~~s quatro números. v esse e m!)restado mais NCr$ 20,50 teria ficado com NCí$
1. 008. DO." Quanto possuía Cyro?
Resposta: 3, 5, 7, 9 e 11.
Resposta: NCr$ 1. 107;40.
G8i'\ A
soma de quatro números pares consecutivos é
Gfil}A soma de dois números é 24 e um dêles é o ,q uín-
44. ~r êsses números.
tuplo do outro. Achar os dois números.
Respos ta: 8, 10, 12 e 14.
nespos a: 4 e 20 .
~ Uma pessoa deve NCr$ 590,00. Se ela tivesse
NCr~O. 00 mais do que tem pagaria à sua dívida e a inda ~ A soma de dois riú.meros é 72 e um dêles é o ~ri­
ficaria com NCr$ 64,00. Quanto tem a pessoa? plo do outro. Qtõ.ais são os dois números?
Resposta: NCr$ 314,00. Res:JosLa: 18 e 54.

~Um estudante para pagar NCr$ 80,00 que devia, ~A difernnca de dois números é 60 e o xnai.or é o
pediu a um de seus colegas NCr$ 35, 00 emprestados. Feito auí~o
.. . do meno~. Achar os números.
o pagamento ainda ficou com NCr$ 6, 50. Quanto tinha antes
de receber o dinheiro emprestado'? Resposta: 75 e 15.

Resposta: NCr$ 51,50. ~A diferença de dois números é 64 e o quocie11te


~Três estudantes combinaram fo:zer uma excursão. elo mmor pelo meno.r éJ5. Achar os números.
O p~iro concorreu com NCr$ 3,00, o segundo com NCr$ RcsI'osta: 16 e 80.
5,00 e o terceiro com NCr$ 4,50. Ao regressarem verifica-
ram que as despesas não passaram de NCr$ 8,10. Quanto @ A soma de dois números é 96 e o quociente t.lo
d everá receber cada um do saldo, uma vez que as despe- m a ior pelo menor é 5. Açhar os números.
sas foram distribuídas igualmente entre êles?
Respos:a: 16 e 80.
Resposta: NCr$ 0,30; NCr$ 2,30 e NCr$ 1,80.
~A diferença de dois números é 58. O ma~or é o
Q A soma de dois números é 900 e a diferença en- trip~o menor mais 8. Achar os números.
tre ~é 300. Quais são os números? Resposta: 83 e 25.
Resposta: 300 e 600.
l5i1) A diferença de dois número:.> é 82. O maior é o
43) Um rádio e uma televisão custa m juntos NCr$ quá~lo do m enor menos 11 . Quais são os número;:;?
SOO.DO. Se comprar o rádio ainda me sobrarão NCr$ 120,00,
m as p ar a adquirir a televisão precisava ter mais NCr$ 180, 00 Resposta: 113 e 31.
e'O
<lc Matemática para o Ad rn is~ilo Ú :JI
•53 l '>,ulu l' misoa
Problemas
r:à A soma d.e d.ois números ~
4. 370. ?
menor é a
qu~arte do maior mais dois. Achar os nu.meros. 60) Um atirador devia receber NCr$ 0.40 por tiro
acertado e pagar a metade cada vez que errasse. Depois
Resposta: 728 e ·3 . 642. de 32 tiros recebeu NCr$ 8, 60 . Quantos tiros acertou?
~Um pai tem 5 vêzes a idade de .seu filho. A so- Resposta: 25 tiros.
ma ~~as idades é 78. Qual a idade de cada um?
61) As idades de duas pessoas somam 120 anos. Sub-
Resposta: 13 e 65 anos. traindo-se 10 anos da idade da mais velha e acrescentan-
do-os à da mais môça as idades tornam-se iguais. Qual
-fr;;f.. Um filho tem 11 anos e seu pai 35 . Daqui a quan- a idade de cada uma?
tos~ a idade do pai será o triplo da idade do filho?
Resposta: 70 e 50 anos .
Resposta: 1 ano.
G1 Tenho42 anos e meu filho 15 anos. Há qu.anto~ 62) Um fazendeiro ofereceu como pagamento de dívi-
anosYoi a idade do meu filho a quarta parte da mmha. ,das um cavalo ou um boi, que valiam juntos NCr$ 480, 00. Se
o credor aceitasse o cavalo o fazendeiro ainda ficaria de-
Resposta: Há 6 anos. vendo NCr$ 50,00, mas se preferisse o boi teria que devol-
ver NCr$ 30,00. Pergunta-se: qual o valor do cavalo e do
56) Um pai tem o dôbro da idad~ ~o filho._ Se tivesse boi, respectivamente, e qual a dívida do fazendeiro?
25 anos menos e seu filho 13 anos mais eles tenam a mes-
ma idade. Qual a idade de cada um? Resposta: cavalo .. . ... NCr$ 200,00
Resposta: 76 e 38 anos. boi .. ..... .. NCr$ 280,00
57) Um pai tem 53 anos e seus qu.atr~ ~ilhos ,33, 32, dívida . ..... NCr$ 250,00.
31 e 29 . Há quantos anos a idade do pai foi igual a soma 63) Multiplicando-se um número por 7 fica aumentado
das idades dos filhos? de 378. Qual éi êsse número?
Resposta: Há 24 anos. Resposta: 63.

58) Um avicultor pretende vender cada frang? a 64) Multiplicando-se um número por 17 êle fica au-
NCr$ 2,20. No percurso do aviário para o mercado fugrram mentado de 1 . 024. Qual é êsse número?
9 frangos. Para não ter prejuízo vendeu cada um dos res-
tantes por NCr$ 2,50. Quantos frangos levou para o mer- Resposta: 64.
cado e qual a quantia apurada na venda?
65) Dividir NCr$ 3 . 500,00 entre duas pessoas de modo
Resposta: 75 frangos e NCr$ 165, 00. que uma receba o sextuplo da outra.
59) Um chapeleiro comprou seus chapéus por NCr$ Resposta: NCr$ 500,00 e NCr$ 3. 000,00.
9, 00 cada um e os revendeu por NCr$ 12, 00 . Quantos cha-
péus vendeu se obteve um lucro de NCr$ 165, 00? 66) Dividir NCrS 6. 000, 00 entre duas pessoas, de mo-
do que uma ienha NCr$ l . 200,00 mais do que a outra.
Resposta: 55 chapéus.
Responta: NCr$ 2 . 400,00 e NCr$ 3. 600,00.

·60 Paulo Pesso11


, ,4 /4
1 d. ·
A soma de dois numeres e A/! e a iv1sao e
d ~ G Numa divisão o resto
visor 12. Calcule o dividendo.
é 3; o quociente 127 s o di . .
um pe o outro dá 13 para quociente e 12 para resto. Quais Resposta: 1. 527.
~
são êsses números?
. Numa divisão o resto é 81 e o quociente 546. A
Resposta: 33 e 441. diferença entre o dividendo e o divisor é 189 . 196. Calcule
@ A som~ de dois núme~os é 1 . 552 e a divisão ~e o dividendo e o divisor.
um p elo outro da 102 para quociente e 7 para resto . Quais
são êsses números? Resposta: dividendo 189. 543
divisor . .. ... , . 347
Resposta: 1. 537 e 15.
69) Assistindo a um iogo de futebol haviam 5. 300 ~Numa divisão o resto é 3 e o quociente 127. A di-
pessoas e a renda foi de NCr$ 45 .'000, 00. Considerando que Jerença entre o dividendo e o divisor é 1. 515. Calcule o
o preço das cadeiras é de NCr$ 10.00 e das gerasi NCr$ 5,00, <lividendo e o divisor.
pergunta-se: quantas pessoas assistiram o jôgo de cadeiras
e quantas foram para as' gerais. Resposta: dividendo: 1 . 527
divisor ....... . 12
Resposta: 3. 700 em cadeiras e 1 . 600 nas gerais .
70) A soma de três números que figuram em uma
subtração é 283 .492. O minuendo excede o resto de 10. 52 1
® N~ma divi,são o dividendo é 75; o divisor 19 e o
resto o maior possivel. Ache o quociente .
unidades. Determinar êsses números. Resposta: 3
Resposta: Minuendo . . . . .
Subtraendo . . .
141. 746
10. 521
Eâl) Em uma divisão de números inteiros o dividendo
é 184; o resto 14 e o quociente é um número de dois alga-
Resto . . . . . . . . . 131.225. 11smo.s. Calcule o divisor.
@ A soma de três números que figuram numa sub- Resposta: 17.
traçao e 7. 090. O rninuendo e xcede o resto de 129 unidades.
Determinar êsses números.
~ Tem-se um produto de três fatôres. Com a multi-
plicação do primeiro por 4; do segundo por 5 e dividindo o
Resposta: Minuendo . . . . . 3. 545 terceiro por 2, o produto fica aumentado de 14 . 580 unidades.
Subtraendo . . . 129 Qual é o produto?
Resto . . . . . . . . . 3. 416
Resposta: l .1620.
'!"-) t;;'\Numa divisão o dividendo é 189 . 51 4; o quociente
é 5~o resto 81. Qual é o divisor? 80) Tem-se um produto de três fatôres. Com a mul-
tiplicação do primeiro por 2; do terceiro por 5 e dividindo
Resposta: 347. o segundo por 2, o produto fica aumentado de 4. 860 uni-
-j) t;;;''Numa divisão o dividendo é l. 527; o resto é 3 e 1 lades. Qual é o produto?

o q~nte 127. Calcule o divisor. Resposta: 1 . 620.


Resposta: 12.
P robl emn s ele Matemática para o Admis são 63
81) Numa divisão o dividendo é 1.317 e o divisor 121. 88) No meu caderno de desenho geométrico , desenhei
Escreva: 11iungulos e quadrados, ao todo 35. Contando-se os seus la-
dos encontraremos 125. Quantos são os triângulos e or.
a) o maior número que se pode acrescentar ao divi-
quadrados?
dendo sem alterar o quociente;
b) o menor número que se deve acrescentar a o divi- Resposta: 15 triângulos e 20 quadrados.
dendo para que a divisão seja exata;
c) o menor número que se pocie acrescentar ao di- 89) Duas peças de fazenda são da mesma qualidade
videndo sem aiterar o resto. e da mesma largura. Uma tem mais 33 m que· a outra
Resposta: a) 13; 14 e 121. 10 custou NCr$ 102, 00. A outra custou NCr$ 62, 40 . Dê o
comprimento de cada peça.
82) Mesmo exercício para dividendo 21.975 e divisor
215. .r espostu; 85 rn 52 m . 'ol 'gio Ped10 II -· 1968
Resposta: a) 169; 170 ·e 215.
90) Escrova o número qu dividido por 41 tem para
83) Numa divisão o resto é 214; o quociente é o dôbro quociente 9 e o resto 5.
do divisor, que é o menor possível. Qual é o dividendo?
Resposta: 92. 664. Resposta: 374. Ginasial - 1968
84) Mesmo problema para: resto 32. 91) Paguei com uma nota de NCr$ 5,00 dois quilogra-
Resposta: 2 .210. mas de íeijáo que custa NCr$ 0.80 o quilograma. Quanto
receberei de trôco?
85) Um tio deixou metade de sua fortuna para 3 so-
brinhos e a outra metade para 2 sobrinhas. Qual será Resposta: NCr$ 3,40. Ginasial - 1968
a importância que caberá a cada pessoa, sabendo-se que a
~ ortW1a do tio é de NCr$ 231, 60. 92) Quantos centavos existem na têrça parte de NCrS
Resposta: Sobrinho: NCr$ 38,60 0,90?
Sobrinha: NCr$ 57,90.
Resposta: 30 centavos. Ginasial - 1968
86) Uma escola tem 400 alunos entre m eninos e me-
ninas; o.s meninos pagam NCr$ 10,00 por mês e as meni- 93) A soma de quatro números pares consecutivos é
nas NCr$ 5,00. A renda mensal é de NCr$ 3. 500,00. Quan- 252. Escreva o maior dêles.
tas meninas e quantos meninos tem a escola:?
Resposta: 300 meninas e Resposta: 66. C. Pedro II - 1968
100 meninos
87) Comprei 4 maçãs e 16 peras por NCr$ 52,00. Por 94) O quociente de uma divisão exata é 48. Dividin-
3 maçãs e 17 peras teria pago NCr$ 49,00. Calcular o preço do-se o divisor por 3 quanto fica sendo o quociente?
de cada maçã e de cada pêra .
Resposta: 144. C. Pe.r:lro II - 19,$8
Resposta: NCrS 2,00 e NCr$ 5,00.

Paulo Pessoa
Problemas de Malcmátice· para o Admis s fio 65
95) Carolina pagou por 4 maç· s NCr$ L 20; sabendo-se
que 11 maçãs pesam 2kg, calcule o preço de lkg de maçã.
Resposta: NCr$ 1.65.
Ginásio I. E. e C. Dutra - 1968

96) Um fazendeiro vendeu 6 cavalos a NCr5 80,00


cada e alguns bois a NCr$ 50,00 cada Com a importân-
cia obtida comprou uma casa por NCr$ 600,00 e ainda lhe
sobrou NCrS 280,00. Quantos bois vendeu o fezendeiro?
Chama-se potência de um número, o produto dêsse nú-
Resposta: 8 bois. Liceu Nilo Peçanha 1968 mero por si mesmo, um certo número de vêzes.
O produto de um número por si mesmo, duas vêzes é
a segunda potência do número, comumente chamada qua-
drado, por analogia à área do quadrado.
Assim quando tivermos, por exemplo
32 ; 52 ; 62 , diremos:

três elevado ao quadrado; cinco elevado ao quadrado; seis


'e levado ao quadrado, cujos valôres são:
3~ = 3 X 3 = 9; 5 = 5
2 X 5 = 25; 63 = 6 X 6 = 36.

O produto de um número por si mesmo três vezes é a


rceira potência do número, comumente chamada cubo, por
onalogia ao volume do cubo.
Assim, quando tivermos:
33 ; 53 e 63 , diremos

t1 ôs elevado ao cubo; cinco elevado ao cubo; seis elevado


' to cubo, cujos valôres são
3:i = 3 X 3 X3 = 27; 5:: = 5 --:--- 5 + 5 = 125 e 63
53 = 5 X 5 X 5= 125 e 63 =
Assim como existem as segunda e terceira potências
(quadrado e cubo) de um número, também existem a quar-
lu; a quinta; a sexta, etc. potências do mesmo número,

66 Pn.ulo I' e'"'-S OR


PrulJJ.cnrn..s llL: Muic1nútica pura u Admhrnfio 67
mas não têm nomes especiais e são designadas pelos or- Do mesmo modo,· constatamos que os cubos podem tet~
dinais; assim: minar por qualquer dos algarismos, mas só não pode ser
cubo se o número de zeros não fôr múltiplo de 3
34 , 54 e 64 são as quartas potências de 3, 5 e 6.
Quando as bases de várias potências são diferentes
Nos exemplos apresentados dos quadrados, cubos e mas os expoentes são iguais, diz-se que as potências são
quartas potências de 3, 5 e 6, êsses números são as bases semelhantes.
da segunda, terceira e quarta potências.
Os números 2, 3 e 4 'escritos em tamanho menor, aci- No caso porém de se ter
ma e à direita das bases são os expoentes das potências 53 , 7~ e ll3
de 3, 5 e 6 e representam o número de vêzes que suas bases
são potências semelhantes. As bases 5, 7 e 11 são diferentes
figuram como fatôres. Convencionalmente a primeira po- mas seus expoentes são iguais.
tência de qualquer número não se escreve', pois é o próprio
número. Seu expoente designativo seria a unidade, que não 23 , 2 e 2 2
se escreve, mas que se considera igual a 1, como iremos ver. teremos o que se chama: potências da mesma base .
Por isso não escreveremos O produto de· várias potências da mesma base é igual
à base elevada a um expoente igual à soma dos expoentes
das diversas potências que figuram como fatôres.
para indicar a primeira potência de 3, 5 e 6, mas ao depa-
rarmos com os números 3, 5 e 6, por exemplo, devemos con- Assim:
siderá-los como estando elevados ao expoente 1. 32 X 3 X 3~ = 32 + i + 3 = 3r.
Daremos a seguir os quadrados e os cubos dos núme-
ros inteiros de 1 até 12, que devem ser memorisados pelo O quociente de potências da mesma base é igual à
alunos. base elevada a um expoente igual à diferença entre o
xpoente da potência do dividendo, menos o expoente da
Quadrados de 1 a 12 potência divisor.

N.ºs. 1 - 2 - 3 - 4 - 5 - 6 - 7 - 8 - 9 - 10 - Assim:
11 - 12.
1 - 4 - 9 - 116 - 25 - 36 - 49 -- 64 - 81 - 100 -
121 - 144. Quando temos uma potência elevada a uma potência,
Vemos pois que não existe nenhum quadrado termina- remos o que se chama potência de potência.
do em 2, 3, 7, 8 e número ímpar de zeros. Assim:
Cubos de 1 a 12 (2")2 e teremos uma potência de uma quantidade, que
por sua vez é uma potência.
N.ºs. 1 - 2 - 3- 4 5 - 6 - 7 - 8 - 9- 10 - É pois a potência de uma potência. De acôrdo com a
11 - 12. dofinição da potência podemos escrever
l - 8 - 27 - 64 - 125 - 21G - A4 3 - 512 - 72G - -·
1.000 - 1.33 1 - 1.728. (23)2 = 23 X Z3

t'toblomas de Matemátic" para o Admissão 69


e teremos então o produto de d uas p otências da mesma EXERCíCIOS R ESOLVIDOS
base cujo resultad o será, como vimos :
2:: + ::, is to é 2r.. 1) Classificar as seg uintes potências:
2 4 , 74 e 11 4 •
Verificamos entã o que tal resultado poderia ser obti-
do, conserva ndo-se a base 2 da potência e elevando-a a Considerando que suas bases \são diferentes mas
um expoente igual ao produto dos expoentes isto é, seus expoentes iguais, trata-se de: potência s semelhantes.
3 X 2 = 6.
2) Efetuar o produto das potências sem elhantes:
Isto posto, e para corroborar o que foi dito, considere- P = 22 X 3 2 X 5 2 •
mos o exemplo seguinte:
Podemos escrever:
1- - 1
1 (3 ~i:i 14 p (2 X 3 X 5F ou
1_ _ 1
p 30 2 = 900.

cujo resultado se:rá: 3) Efetuar o produto:


P = 2" X 2° X 2 X 3 2 X 3 X 5 .

Embora sendo a ssunto de nível superior ao dêste livro, Depois do que foi dito sôbre p roduto de potências da
convém deixar dito o seguinte: Não existem restrições quan- mesma base podemos escrever:
to à natureza das bases e expoentes das potências. Elas po-
dem ser inteiras, fracionárias, decimais, positivas ou ne- P = 22 + 3 + 1 X 3~ + 1
X 5 ou
gativas.
p =
2G X 3:J X 5 = 64 X 27 X 5 =- 8. G40 .
1) l\.char o quociente de:
Convém a ssinalar também que qualquer quantidade
elevada ao expoente zero é igual à unidade. Assim: (2 16 X 31u X 5 1i) --;- (21 X 3:; >< s~)

Como vemos o dividend o e o divisor contêm po tôncius


4º = l; 12º = 1.
tkr mesma base. Dividindo-as respectivamente teremos:
O expoente zero aparece quando temos duas potências =
z1 l• --;- 2J 21 u --· t = z1~ a
da mesma base e expoente, para dividir. Assim, de acôr- 3 1" -;- 3:: = 3i:; ., - 21~ b
do com a regra dada para dividir potência s da mesma bas,e 511 --;- s ~ =
511 - " =s l~ e
se tivermos, por exemplo:
Os resultados obtidos em a, b e e depois de m ultipli-
cados dão o quociente procurado, que são :
2 12 X 31~ X 51 ~ OU, por se tratar de po-
o resulta do será : tencias semelhantes
13" - r. = 13º = 1. (2 X 3 X 5) 1 ~ = 30 12

7W
) r n.111 0 P essoa
5) Achar o valor de EXERCíCIOS PARA RES.OL VER
(2 3 X 3' X 5) 2 l) Classificar as seguintes potências:
Trata-se de uma potência de várias potências e então, a) 7rn 101 5 2915
depois do que foi dito, escreveremos: b) 25 27 21 8
c) (2ª)2 (35)3 (72)5
zs X 2 X 32 X 2 X 51 X 2 ou
2 6 X 34 X 5' 129.600. Resposta: a) Potências semelhantes
b) Potências da mesma base
6) Simplificar a expressão:
c) Potências de potências.
(2 3 X 34 X 5) 3 X (2 X 3' X 54) 4
2) Efetuar os produtos:
(2 2 X 3 X 5')"'
a) 43 X 6 3 X 73
Podemos escrever: b) 23 X 3 ~ X 54
(2 9 X 3 1 ' X 53 ) X (2 4 X 38 X 5 1 6 ) e) (3~)3 X (S ~P X (T2 )2
ou
24 X 32 X 5·1 Re.sposta: a) 168:1
b) 8 X 9 X 625
2 9 +4 X 3 l' + 8 X 5 :1 + 1G
e) 3u X 5'; X 7 1 ou !Sr, X 7 1•
1 2
2 X 3 X 5"
:1) Efetue o produto:
213 X 320 X 5n
(Z2 X 35 X 7') X (2 1 X 3 'X 57 ) X (2 X 3:> X l l2).
21 X 32 X 51 Resposta: 2 7
X 3 11
X 5 7 X 7 2 X l l2.
Dividindo-se as potências da mesma base do produto 4) Achar o quociente de:
·SÔbre o traço pelas suas correspondentes sob o traço, vem:
(11 9 X 13 1 X s i) (5 2 X 13 11 X ll7).
213 -;- 21 = 213 - 1 = 29 (a)
320 32 = 3 20 - 2 = 31s (b) Resposta: 13 3 X l l2.
5 -;- 54
19 =5 10 - 4 = 5'.~ (e) 5) Achar o valor de:
Multin!icando-se os resultados obtidos em (a), (b) e (c), (3 4 X 58 X 11 3 ) 5
vem com; resultado:
Resposta: 32 º X 54 0 X I l15.
29 X 31s X 51 r. .
6) Simplificar a expressão:
7) Dividir: 1937 6 -;- 1937 6 •
2 X 33 X 5 X (32 X 5l3 X 3T~
Para dividir potências da mesma base, conservamos a
base e subtraímos os expoentes·. Então: 33 X 5

1937 6 -7- 1937G = 1937G - G 1937º = 1. Resposta: 2 X 3•; X 5.; X 37~

l'l'UlJ!cma;; de Matemática·
7) Calcule a s expressões:
a) 23 5u g:1. + +
b) 18 4 - 6 ~ .
c) 123 - ll2.
Resposta: a) 13 .197
b) 104.97H
c) 1.607
DIVISIBILIDADE
8) Numa divisão inexata, o resto maior possível é o
resultado de: 7 X 12 - 24 • A diferença entre o divisor e o Diz-se que um número é divisor de outro, quando o
quociente é s~. Determinar o dividendo. divide exatamente.
Colégio Militar - 1968 O número que divide, chama-se divisor, submúltiplo ou
Resposta: 3. 104.
fa tor do dividendo.
Tratando-se de quantidades concretas podemos dizex
parte alíquota em vez de divisor.
O número que é divisível chama-se múltiplo do divisor.
Podemos então dizer que: Múltiplo de um número é d
p roduto dêsse número por um número inteiro qualquer.
Existem regras que permitem saber ràpidame nte, se um
número é divisível ou não por outro e no caso de não ser,
qual o resto a encontrar.
Essas regras, vantajosas apenas para certos divisores,
constituem o que se chama Caracter.es de Divisibilidade, que
passamos a enunciar.

Divisibilidade por 2
Para um número ser divisível por 2 é necessano e su-
:ficiente que o seu algarismo das unidades seja: O, 2, 4, 6
ou 8, isto é, par.
Se tal não acontecer o número não será divisível por 2
e o resto de sua divisão por 2 será o mesmo que se obtiver
1..lividindo-se o algarismo das unidades do número por 2.
Assim, no caso do número 1967, concluimos: n ã o é di-
visível por 2 porque o seu algarismo das unidades (7) não
é par. Não sendo divisível deixa resto que é o da divisão
de 7 por 2, isto é, 1.

74 Pa. ulo Pe ss oa Problemas de Mnülmútica pnl'a o Allmi sniío 75


Convém lembrar que o resto de uma divisã o, quando Divisibilidade por 5
é muito grande (maior resto possível) é igual ao divisor me-
nos um. Então se o número considera do não fôr divisível Para um número ser divisível por 5 é necessano que
p or 2 deixará sempre resto um, isto é: 2 - 1 =
1 (divisor seu algarismo d as unidades seja zero ou cinco. Caso con-
menos um). trário não será divisível e deixará para resto, quando divi-
dido por 5, o mesmo que o algarismo das unidades do nú-
Divisibilidade por 3 mero deixar . Assim , 1967 não é divisível por 5, p orque ter-
mina em 7. O resto será aquêle da divisão de 7 (algarismo
Pa ra um número ser divisível por 3 é p re ciso que a das unidades do núm ero) por 5, isto é, 2.
soma d os valôres absolutos de seus algarismos s eja múl- Se o número fôsse 13. 824 também não seria divisívei
tiplo (d ivisível) de 3. O resto da divisão de um número por por 5 e seu res to seria 4.
3 é o m esmo que o da divisão da soma dos valôres abso-
lutos de seus algarismos, dividida por 3. Divisibilidade por 6
Entende-se por valor absoluto dos algarismos que
constituem um número, os valôres que êles (algarismos) Para um número ser divisível por 6 é preciso que o seja
têm, independente da posição que ocupam no n úmero con- por 2 e 3. Assim 1962 é divisível por 6 porque é d ivisível
siderado. por 2 (porque é par) e por 3 porque (1 + + + =
9 6 2 18)
é múltiplo de 3.
Assim em 1967 os algarismos 1, 9, 6 e 7 têm para va-
Por outro lado o número 1562 não é divisível por 6,
lôres absolutos 1, 9, 6 e 7.
porque embora sendo divisível por 2, não o é p or 3.
Então 1967 só será divisível por 3 se: 1 + 9 + 6 H- 7
fôsse igual a um múltiplo de 3. Divisibilidade por 8
Como não é, pois sua soma é 23, o número 1967 não
é d ivisível por 3 e o resto de sua divisão pelo divisor 3 Para um número ser divisível por 8 é preciso q ue os
é 2. (23 --;-- 3). seus três últimos algarismos da direita, formem um núme-
ro divisível por 8 . Se tal não acontecer, o número não s erá
Divisibilidade por 4 /divisível por 8, e o resto de sua divisão por 8 será o mesmo
que o do número assim .formado, dividido por 8.
Para um número ser divisível por 4 é preciso que o nú- Por exemplo. O número 21. 967 não é divisível por 8
mero formado pelos dois últimos algarismos d a direita d o porque 967 (n úmero formado pelos três últimos a lgarismos
número, sej a divisível por 4. Se tal não acontecer o número da direita d o número 21 . 967) não o é. Seu resto p or 8 será
não será divisível por 4 e o resto de sua divisão pelo divi- o resto d a divisão de 967 por 8, isto é, 7 .
sor 4, será o do número formado pelos d ois últimos alga-
rismos (do direita) do número por 4. Divisibilidade por 9
Assim o número 1967 não é divisível p or 4, porque 67
(número formado pelos dois últimos algarismos da direita Para um número ser divisível por 9, é pr ·is qno 1
de 1967), não o é. Seu resto será o da divisão de 67 p or 4, rmma dos valôres absolutos de seus algarismos sejn um
isto é, 3. rnúltiplo de 9. E'ntão, o número 1967, cuja soma dos valô-

!76 P1w lo P ess oa


res absolutos de seus alarismos é 1 + 9 + 6 + 7 = 23, Se o número fôr 27 .691:
\não é divisível por 9, porque não é múltiplo de 9 . O resto I I I
da divisão de 1967 por 9 é o mesmo que o de 23 por 9, ou 2 7 6 9
seja, 5. p p

Divisibilidade por 10 1 + 6 -1- 2 9 (soma dos algarismos de ordem


ímpar - 1.0 - 3. 0 - 5.0 ).
Um número é divisível por 10; 10 2 ; 10 3, •• •• 10", quan-
;do termina em 1, 2, 3 . . . u zeros . O resto da divisão de 9 + 7 16 (soma dos algarismos de ordem par
um número por 10; 100; 1. 000 . . . 10n é o número formado 2. 0 e 4.0 ).
pelo último dois últimos; três últimos; n últimos alga- Não sendo possível de 9 subtrair 16, somamos a 9 um
rismos da direita do número. múltiplo de 11 que permita a subtração.
Assim o resto de 21 por 10 é l; o de 245 por 100 é 45;
o de 1. 967 por 1 . 000 é 967, etc. Então:

Divisibilidade por 11
9 + 11 20 (soma dos algarismos de ordem ímpar
acrescida do múltiplo de 11 necessá-
rio à subtração).
Um número é divisível por 11 quando, a partir da di-
reita a diferença entre a soma dos algarismos de ocrdem Então:
ímpar e a soma dos algarismos de ordem par, dá zero,
onze ou múltiplo de 11.
20 16 = 4. 4 11 - resto 4.
O resto da divisão de um número por 11 é o mesmo que Concluímos: o número 27 . 691 não é divisível por 11 e
o da divisão do número que se obtém subtraindo-se a so- quando dividido deixa resto 4.
ma dos algarismos de ordem par, a partir da direita, da
soma dos algarismos de ordem ímpar, aumentados de um Teorema
múltiplo de 11 tal que permita a subtração.
Se dividirmos uma soma e cada uma das parcelas pelo
Assim para o número 1967 mesmo número, a soma dará o mesmo resto que a soma
I I dos restos das parcelas .. Seja achar o resto por 4 da se-
1 9 6 7 guinte soma:
p p
123 + 215 + 389 = 727
7 + 9 16 (soma dos algarismos de ordem iímpar
Achando os restos das parcelas e da soma, por 4 en-
1.0 e 3. 0 ).
.6 + 1 7 (soma dos algarismos de ordem par contraremos:
2.0 e 4. 0 ) .
16 - 7 .= 9 9 -;- 11 - resto 9. Resto de 123 por 4 -3}
Resto de 215 por 4 -3
O número 196'7 não é divisível por 11 e seu resto por Resto de 389 por 4 _ 1 Soma dos restos 3 +3+1= ?
aquêle divisor é 9. Resto de 727 por 4 - 3 .

73 ri..'l.il dU J1 c:;s ua Problernaf( el e Mntcmútiea par a o Alln1issão 79


Resto da soma l Para o produto:
dos restos 7 por ~ Iguais como diz o teoremcr
4 - 3 J 16 V JS 11 - r
1 1 1

Podemos então achar o resto de uma soma de vanas + + +


7 ::.< 6 X 4 Restos <los fatôres)
parcelas, por um determinado número, sem efetuá-la. \._ ------.,.,.-----_.)
1
Se dividirmos o produto de vários fatôres e cada +
um dêles pelo mesmo número, o produto dará o mesn10 168 - Produto dos restos dos fa-
rnsto que o resto do produto dos restos dos fatôres. tôres.
6 - Resto do produto dos rc .. !o>i
Seja achar o resto por 9 do produto seguinte: dos íatôres, por ~ .
16 X 15 ,( 13 = 3.120 EXERCfCIOS IWSOLVIDOS
Achando-se o resto dos lôr s e do prodn!o por 9, en - 1) Determinar os restos das divisões por 2, 3, 4, 5, 8,
contraremos:
9. 10 e 11 do número 235. 898.
Resto de l G por 9 - 7 l Produto dos l or 2 - (8 -:- 2) - · resto zero.
Resto de 15 por 9 - G r 7 X 6 X 4 Por 3 - (2 +3+5+8+9+ 8) 3 85 3 -
Resto de 13 per 9 -- 4 ) = ma. resto 2.
Por 4 - (98 -:- 4) - resto 2.
Hesto de 3-120 por 9 - - 6 l Iguais como Por 5 - (8 -:- S) - - resto 3.
Resto do produto dos restos por 9 - 6 ~ diz o teorema Por 8 -· 898 -:- 8) - resto 2.
J. Por 9 - (2 3 + + + +
5 8 9 1 8) -:- 9 = 35 -:- 9 -· -
resto 8.
Como no caso da soma, é possível calcular o resto de Por 11 - (8 + 8 + 3) - (9 + 5 + 2) = 19 - 16 = 3;
um produto, por wn determinado número sem efetuar o resto 3 .
3 -:- 11 -
produto indicado. 2) Determinar o valor de a para que o número 3.0aS
~; cja múltiplo de 3 e 9.

Costuma-se dispor os cálculos acima explicados da A regra de divisibilidade por 3 diz que é preciso que
maneira que se segue. 3 + +
O a + 5 seja múltiplo de 3, isto é, divisível
por 3.
Para a soma:
Então:
123 + 215 + 389 = s
1 1 1 3 O + +a +5 = 8 +a para ser múltiplo de 3
+ + + preciso que tenhamos:
3 + 3 + 1 (restos das parcelas)
8 +a 9 e a = 1
7 (soma dos restos das parcelas 8 +a 12 e a = 4
3 (resto da soma dos restos das parcelas por 4) 8 +a 15 e a = 7

8() Paulo Pesso1t


Pl'oblcmas de Ma tem{d.ica 11ara o Ad mis são 81
Como a representa um algarismo, no máximo poderi.a 2. 0 - 8 + a + 5 + 2 = 15 + a, e pelas razões expostas
•r.:er 9, mas no problema nã o pode s er maior do que 7. acima: a= 3.
A regra de divisibilidade por 9, diz que é preciso que:
3. 0 - 8 +a + 5 + 4 = 17 + a e pelo mesmo motivo
a = 1.
3 + O+ a + 5 seja múltiplo de 9, para ser divi- 4. 0 - 8 + +
a 5 + R - 19 + a e a = 8.
s ível por 9. 5. 0 - 8 + a + 5 + 8 = 21 + a e a = 6.
Então: Os valôres de a e b são pois:
3 + O + a -1- 5 = 8 + a para ser múltiplo elo 9 a = 5 e b o
é preciso que: a = 3 e b 2
8+a = 9e a = l. a 1 e b 4
Pelos motivos expostos para o caso do divisor 3, o a a = 8 e b 6
não pode ter outro valor que não seja 1. a = 6 e b 8
Então, comparando os valôres achados para que o nú- 4) Determinar um número de dois algarismos que di-
mero seja divisível por 3, bem como os que o tornam divi- vididos por 7 dão resto 4 e dividido por 5 dá resto 3.
sível por 9, concluímos que: a = 1.
3) Achar os valôres de a e b para que o número
Os números de dois algarismos que divididos por 7 dão
8 a 5 b seja divisível por 2, 3 e 9. resto 4 são os múltiplos de 7 que somados a 4 não formam
uúmeros de três algari::imos e que são:
Para o número ser divisível por 2 é preciso ser par.
Então, de início, substituindo b pelos seus possíveis valôres,
vem:
8 a 50 l.º 7 + 4 11 23 +4 32 49 + 4 53 70 + 4, 74
8 a 52 2.º 14 + 4 18 35 +4 39 56 + 4 60 77 + 4 81
Grupo I 8 a 54 3.º 21 + 4 25 42 +4 46 63 + 4 67 81 + 1l 83
{ 8 a 56 4.º 91 + ~ 95
8 a 58 5.º

Qualquer dêles, independentemente do valor de a é di-


visível por 2. Todos êsses números divididos por 7 dão resto 4 .
Considerando que todo número divisível por 9 é tam- Trata-se portanto de identificar entre êles, os que divi-
bém por 3, calculemos os valôres de a, que tornam o grupo didos por 5 deixam o resto pedido 3. Com o auxílio da re-
I divisível por 9. gra de divisibilidade por 5, verifica-se serem apenas os nú-
Teremos: meros 18, 53 e 88.
l.º - 8 + a + 5 + O = 13 + a, que precisa ser múltiplo 5) Determinar os números de 3 algarismos que sejam
de 9, isto é, 18, 27, 36, etc. divisíveis por 3 e 4 e nos quais o algarismo das dezenas
Então a = 5 . ·oja zero.
0')
32 Pa.ulo P ess oa Proi.J lc nm::; de Matcrn:i.t.icu: lJara o Adrni:;~5 fí.o 01)·
Inicialmente podemos representar o número do seguin- O problema se transforma em: dado o número 23 a S,
te modo: determinar o valor de a para que ê ie seja divisível por 9.
a o b
Então:
Para ser divisível por 4, é necessano que os dois últi- 2 -l- 3 + a+5 = 10 + a
mos algarismos formem um número divisível por 4.
deve ser múltiplo de 9. Como a representa um algarismo,
Então b só poderá ser O, 4 e 8 e o número tomará o as- é fácil verificar que seu valor é 8.
pecto:
l.º a O O 9) Dado o número 5 a 8, de<erminar a para que o
2. 0 a O 4 número forma do seja divisível por 4 e 11.
3. 0 a O 8
Para um número ser divisível por 4 é preciso que os
A determinação do valor de a ·e m cada caso, comple- seus dois últimos algarismos da direita formem um múl-
tará a resposta pedida: tiplo de 4.

No Lº caso: a = 3; a = 6; a = 9 No caso em questão a poderá ser O, 2, 4, 6 e 8.


No 2.0 caso: a = 2; a = 5; a = 8
No 3.0 caso: a = l; a = 4; a = 7 Teremos então:

6) Calcular o menor número que se deve tirar de 1967 1. 0 508


para se obter um múltiplo de 8. 2. 0 528
3. 0 548
Já vimos que 1967 não é divisível por 8, porque os três 4. 0 5S8
últimos algarismos da direita formam o número 967, que 5.0 588
não é divisível por 8 e deixa para resto 7. Entã o o menor
número a tirar é 7 . Dêsses cinco números o que fôr divisível por 11, res-
ponderá a questão.
7) Qual o menor núme~o que se deve somar a 1 . 520,
para torná-lo múltiplo de 11. A regra de divisibilidade por 11 mostra que apenas o
O número 1.520 não é divisível por 11 e deixa o resto :, gundo, 528 satisfaz ao pro:Plema.
2 achado como já mostramos, isto é:
10 Determinar o menor número que se deve somar
(O -\- 5) - (2 + =1) 5- 3 =2 <r '1 967 para se obter um múltiplo de 5 e 7.
2 -7- l.l - resto 2.
Para um número ser divisível por 5 e 7 é preciso ser
Precisamos então somar ao resto (2) o menor número múl tiplo de 5 e 7 ao mesmo tempo ou de 5 X 7 35 (se-=
capaz de torná-lo m últiplo de 11, para o que é preciso 1nelhante ao caso da divisibilidade p or 6). Basta então di-
somar 9. vidir 1967 por 35, para, em face do resto encontrado, veri-
Jtear quanto devemos somar para torná -lo divisível por 5
8) C locar um alqarismo en t e os alg irismos 3 e 5 dv l' 7, isto é, por 35. O resto sendo 7, como é fácil de veri~
11úm cro 23 ... para 1ornú-lo
m múltíplo de 3. ficar , to na-se necessário somar 28 ao número 1967.

Prob lemas de Matcm{tlica para o Admi ssão 35


- - - - - - - - - - - - - - - -- - - - -- - - - - -
] 1) Calcular o resto da divisão por 3, da soma: Podemos escrever:
3.789 + 25.484 + 27.81'7 X 27.817 + 48 298 X 48.298 ,< 48.2!J8
2 .347 + 1. 9íl7 + J .853 .007
i i y
1 1
+
1
+
1
+
1
+
sem e!e tuá-la. 5 + 8 + 9 X 9 + 8 X 8 X 8
'---y---' ~ '----~--__J

Podemos escrever: 13 + 81 + 512

+ + 1.853.007
1
-t i i
2-347
1
1 .967
1 1
2 + 4 + 6
+ + ..... 12
Resto por 3 1 + 2 + o Resto 1
' - - - -- ---y- - - · ·--
Soma dos restos 3
1
+ EXERCíCIOS PARA RES.OL VER
Resto par 3 da soma O

1) Achar os restos das divisões por 2, 3, 4, 5, 8, 9, 10


12) Calcular o res to por 5 do produto: e 11 do número 184 . 327.
Resposta: l; l; 3; 2; 7; 7; 7; O.
2 .347 X 1 -967 X 1.853.007
2) Determinar o valor de a para que o número 7 a 6
sem efetuá-lo. seja divisível por 4 e 9.
Resposta: a = 5.
Podemos escrever:
3) Dado o número 28 a 6, cleterm· nar o valo1 de ti, ds
2.347 X 1-967 X 1.853.007 mo lo a formar um número divisível por 3 e 8.
1 1 1

+ + + Resposta : a = 5.
Resto por 5 2 X 2 X 2
4) O número 5 a 28 b será ao mesmo tE:rnpo divisível
Produto dos restos 8 por 2, 3, 5 e 9 se colocarmos que algarismos nos lugare3
1
de a e b?
+
Resto do produto por S 3 Resposta: 980 e 3. I. E. C. Dutra - 1968
13) Achar o resto por 11 da seguinte expressão, sem 5) O maior número formado de três algarismos difo-
efetuar as operações: ientes que é divisível por 5 é?

3 . 789 + 25.484 + 27.817 2 + 48 . 298:: Resposta: 985. Ginásio Nilo Peçonha - 1968

36 Pn.ulo Pe~soa
U7
6) Escreva um número de 4 algarismos diferentes, que 14) Calcular o menor número que se deve tirar de
iseja divisível ao mesmo tempo por 2, 3, 4 e 5. 2. 725 para torná-lo um múltiplo dé 11.
Resposta: 1.320. Ginásio Nilo Peçanha - 1967
Resposta: 8.
7) O número 4 a , 8 b, 6 e é múltiplo ao mesmo tempo
de 2, 3, 5, 9 e 10. O algarismo que ocupa a 3.ª ordem é a 15) Qual o menor número que se deve subtrair de
metade do da 5.ª ordem. Quais são os valôres de a, h e e? 31 . 415 para que o resultado obtido seja múltiplo de 9?
Resposta: 5. C. Militar - 1968
Resposta: a = 6; b = 3 e c = O.
16) Colocando-se um algarismo entre os algarismos
C. Militar - 1968
8 e 6 do número 286, forma-se um número de 4 algarismos
8) Dado o número 18 a 4, determinar o valor de a divisível por 3 e 8. Dizer qual é o algarismo.
para que o número formado seja divisível por 3 e 8. Resposta: 5.
Resposta: 2.
17) Dentre os números 26, 30, 35 e 44 um dêles é di-
9) Qual o algarismo que se deve escrever no lugar visível ao mesmo tempo por 2 e 5. Qual é êsse número?
da letra a para que o número 356a4 seja simultâneamen- Resposta: 30. Ginasial - 1968
te divisível por 4 e 9?
Resposta: zero. 18) Determinar o menor número que se deve somar a
3. 528 para se obter um múltiplo de 3 5. e
10) Dado o número 3 a 7 b, substituir a e b por alga- Resposta: 12.
rismos, de modo a se obter um número divisível por 2, 3,
5, 9 e 10. 19) Calcular o resto da divisão de:
Resposta: a = 8 e b = O. 38 . 421 -+ 89.754-+ 3 .521.471
11) Substituir a letra b de modo que fique sendo um por 4 e 5 sem efetuar a soma indicada.
número divisível por 5 e 11, 374 b.
Resposta: 2 e 1.
Resposta: b = O.
20) Calcular, sem efetUar a produto indicado os restos
12) Determinar um número de três algarism.os, menor de suas divisões por 3 e 11 da expressão:
que 150, que dividido por 11 'dá resto 6 e dividido por 9 dá
resto 8. · · · 58.372 X 1945 X 83.708
Resposta: 116. Resposta: 2 e 2.

13) Determinar os números de 3 algarismos que seja m 21) Determinar os restos das divisões por 2 e 8 da ex-
divisíveis por 5 e 9 e nos quais o algarismo ~as dezenas pressão seguinte, sem efetuar as operações indica das:
seja 3.
3. 5272 X 74835 -+ 1. 5273 -+ 2. 823
Resposta: 135 e 630.
Resposta: 1 e 1.

88 l 'i1.cdv l' cssoa


P rvulc mas de Mntcmá li cn para o Admiasão 39
------- - --- - - - - - - -- -
22) Determinar os restos dos divisores por 2, 5 e 9 da
expressão seguinte, sem efetuar as operações indicadas:
215.379 >< 27.274 -+ 35.829-+ 1.327
Resposta: l, 4 e O.
23) A soma de três números pares e consecutivos é NÚMEROS PRIMOS
'i gual ao quadrado de 16 mais 50 unidades. Qual o maior
dos números?
Resposta: 104. Colégio Militar - 1968 Chama-se número primo absoluto a todo número que
só é divisível por si e pela unidade.
Número múltiplo ou composto é todo aquêle que não
é primo. Dois ou mais números são ditos primos entre si,
mesmo sem serem primos absolutos, quando só admitem
para divisor comum a unidade.
Crivo de Eratóstenes é o processo que se adota para
fazer uma lista de números primos e que se denomina tá-
bua de números primos.
Vejamos como podemós organizar uma relação de nú-
meros primos, desde 1 até 100 empregando-se o Crivo de
Eratóstenes.
Escrevemos seguidamente os números desde 1 até 100.

Assim:

1
J 2 3 4 5 6 7 8 9 lfl 11
·12 D lli 15 16 17 18 19 2fl 21 22
23 24 25 26 27 28 29 30 31 32 33
3il 35 36 37 38 39 4fl 41 42 LL3 n
45 lHi 47 48 >!9 50 51 52 53 511 55
56 57 58 59 60 61 62 63 64 65 66
67 68 69 70 71 72 73 74 75 'R6 'R7
78 79 80 81 82 83 84 85 86 87 88
89 9fl 91 92 93 9li 95 96 97 98 99
lfHI

Problemas de Matemática para o Admis siio 91


Risquemos todos os números do quadro, de 2 em 2. a
partir de 2, exclusive. Riscaremos assim todos os números Empregando-se os caracteres de divisibilidade concluí-
mo~ . não ser êle divis~vel por 2, 3 e 5. Dividindo-o por 7
divisíveis por 2.
A seguir risquemos todos os números do quadro de 3 venhcamos que tombem não o é. A regra de divisibili-
.em 3, a partir de 3 excl,usive. Assim serão riscados todos dade por 11, mostra não ser o número 503 múltiplo de 11.
os números divisíveis por 3. A partir de então teremos que dividí-lo sucessivamen-
Depois de 3 o primeiro número ainda não riscado é 5, te p~r _13, l '., 19, 23, etc., até encontrar um quociente nas
por isso riscaremos de 5 em 5, a partir de 5 exclusive, os cond1çoes_ ditas anteriormente, e isso se verifica, no caso
números do quadro e assim teremos riscado todos os nú- em questao, quando usamos o divisor z3,,
meros divisíveis por 5. Naturalmente encontraremos núme-
ros já riscados anteriormente não havendo necessidade d e Assim:
riscá-los novamente.
Depois de 5, o primeiro número ainda sem riscar é 7
503 23
ü por isso, como anteriormente, riscaremo3 a par[ir de 7 ex-
clusive, mas de 7 em 7 os números do quadro, eiiminando 43 21
assim todos os números divisíveis por 7. 20
Depois de 7 o primeiro número ainda não riscado é o
11 e por isso, excluído êle riscaremos os demais, de 11 em Como. ~e:rios -ºquoc!ente (21) é. menor que o divisor
11, eliminando assim os divisíveis por 11. Como nos limi- (23 e at_d1v1sao na~ se fez exatamente (resto 20). Concluí-
tes adotados encontramos já todos riscados, concluímos que mos en ao que o numero 503 é primo.
os não riscados são os números primos compreendidos en-
tre 1 e 100.
Para simplicidade da operação poderíamos ter escri- Decomposição em fatôres primos
to a série de 1 a 100 sem ·e screver os números pares, e x-
ceto 2 (porque é o único número par que é primo·) e co-
meçar a riscá-los de 3 em 3 e a seguir, como foi feito. . ~~comp~r um número em fatôres primos é determinar
os fa.ore~ ~nmos que entram na ·c omposição do número e
Dependendo da exiensão da tábua que se possua po- que multiplicados, reproduzem o nú.mero.
deremos consultá-la para saber se determinado número é
primo. Como porém, na maioria dos casos isto não é pos- Seja decompor o número 420 em fatôres primos.
sível. diremos como proceder, sempre que tivermos neces- Para .isso ,esc_:re~eremos o n~mero à esquerda de um
sidade de verificar se um número é primo. traço vertical, a direita: do qual sao escritos em ordem cres-
Para reconhecermos se um número é prjmo, basta di- cente, de c0'.:a para ,baixo, os números primos que entram
vidí-lo sucessivamente pela série natural dos números pri- nc; compos1çao do mtmero, ou melhor dito, que dividem 0
mos (2, 3, 5, 7, 11, 13, etc.), até encontrarmos um quociente numero.
- caso a divisão não se faça exatamente - igua l ou me-
nor que o divisor, sem que a divisão se faça exatamente, Êsses ,números primos são, portanto, os divisores pri-
para concluirmos que o número é primo. mos do numero. Os números escritos sob o número dado
Seja número 503, que queremos investigar se é o a P?r_a decomp~r, soo o~ quocientes de suas divisões pelos
não primo. 1d1v1~endos prunos escritos, como foi dito, à direita do traço
verhcal .

92 Paulo Pes soa


O número 420 tem 24 <livisores. Vmno::i cctlcu1é.c - lo~ :
Assim:
420 2 (div. primo) 1 (todo n. 0 é divisível por 1)
420 2 -7 210 2 (div. primo) 420 2 2 X1 = 2
210 2 -7 105 3 (div. primo) 210 2 2 X2 = 4
105 3 -----7 35 5 (div. primo) JOS 3 3 X1 = ; 3 X 2 = 6; 3 X 4 12 =
35 5 __.,. 7 7 (div. primo) 35 5 5 X1 = 5; 5 X 2 =' 10; 5 X 4 = 20; 5 X 3 ::-:
7 7 __.,. 1 = 15; 5 X 6 := 30; 5 X 12 = 60.
7 7 7 X 1 = 7; 7 X 2 := 14; 7 X 4 = 28; 7 X 3 =
Então podemos escrever: = 21; 7 X 6 = 42 ; 7 X 12 = 84; 7 X 5 = 35;
7 X 10 = 70; 7 X 20 = 140; 7 X 15 t_ 105;
420 2 X 2 X 3 X 5 X 7 ou 7 30 = 210 e >< 60 t= 420.
420 22 X 3 X 5 X 7.
Verificamos que o menor divisor é a unidade e o maior
Na prática a decomposição de um número em fatôrns é o próprio número.
primos toma o aspecto seguinte:
420 2
210 2 EXEHCfCIOS RESOLVIDOS
105 3
35 5
7 7 1) Decomponha 1960 em fatôres primos e calcule a
1 soma dos expoentes daquêles fatôres.
Assim ::;ão achados os divisores primos do número 420
que, entretanto, possui outros divisores. 1960
980
Vejamos como é possível saber quantos são ao todo e 490
como achá-los depois. 245
O número de diviso;;es (n. d) de um determinado nú- 49
mero é obtido depois de somarmos a unidade a cada ex- 7
poente do fator primo que entra na composição do número 1
e em seguida multiplicar os resultados.
Então: 1960 = 2?. X 5 X 72 e a soma dos expoentes
Vimos que: d os fatôres primos é:

420 = 2i X 3 X 5 X 7

Depois do que foi dito: 2) Decompor em fatôres primos o produto: 60 X 130


X 250, sem efetuá-lo.
n. d = (2 + 1) (1 + 1) (1 + 1) (l + 1) = 24.
Problemas de M ntemática pora o Arlmi ssiio 95
94 !'uulo Pc s$ oa
Decomponhamos separadamente os fatôres do produto: Então vem:
60
30
1
1
"h
2
130
65
1 2
5
250
125
1 2
5
16 3 = (2 4 ) 3 e 12 2 = (2 2 X 3)' = 2~ X 3'
1 1
15 1 3 13 1
13 25 1 5 O número N será então:
5 5 1 5 5
1
1
1
1
1
1

1
N =
212 X 24 X 3' = 21e X 32
e o seu número de divisores:
60 = 2:: ~< 3 X 5 130 '= 2 = 5 X 13 250 = 2 X 5"
n. d = (1 6 + l) (2 + 1) = 17 X 3 = 51.
Então podemos escrever:
5) Qual deve ser o valor de n para que· o número
60 X 130 X 250 15 X 811 admita 8 divisores?

1---, Como no exemplo anterior


22 X 3 X 5 X 2 X 5 X 13 X j 2 X 5 j 3
ou 15 = 3 X 5 e 811 = (2 3 )n = 2an
1 1
O número será pois
22 X 3 x 5 X 2 X 5 X 13 X 2 X 53 N = 23 " X 3 X 5
60 X 130 X 250 = 2 X 3 X 5 4 5
x 13 .
Seu número de .divisores que é 28 é dado por
3) Decompor em fatôres primos a terceira potência de n. d = (3n + 1) (l + 1) (l + l) = 28 ou
280. (3n + 1) X 2 X 2 = 28 ou 3n + 1 = 7.
A terceira potência de 280 é 280 3 , que pode ser escrita
(2 8 0) ~ .
O número que somado a um dá 7 é 6; então:

(280) 3 = (2 3 X 5 X 7) ~ == 3 :M X 5ix:: X 7 1 ""~ = 3n =6e n = 2.


= 29 X 5'1 X 73 • 6) Achar os divisores pares do número 420, direta-
mente.
4) Calcular o número de divisores de: N = 16~ X 1 2~.

Vimos como é achado o número de divisores de um 420 2 12


d eterminado número. 210 2 j 2 X 2 = 4.
16 e 12 não sendo primos, não é aos seus expoentes
105 3 1 3 X 2 =~ 6; 3 X 4 12 . =
35 5 5 X 2 = 10; 5 X 4 = 20; 5 X 6 = 30; 5 X 12 =
que devemos somar a unidade para multiplica r os resul- = 60 .
tados e achar o número de divisores. 7 7 7 X 2 =
14; 7 X 4 28; 7 X 6 42; = =
Devemos decompor 16 3 e 12 2 em fatôres primos, para 1 1 7 X 12 = 84; 7 X 10 = 70; 7 X 20 140; =
d epois aplicar a regra. 1 1 7 X 30 =
210 e 7 X 60 420 . =
96 ]'a.ulo Pe ssoa
l.' roblemas de Mn temátlca: parn o Admls siio 97
qu t r(r o menor valor desde que usemos o maior expoen-
7) Achar os divisores ímpares do número 420, dire-
11 (li) p 11u o m nor fator (2). Sendo os outros dois expoen-
tamente.
1 a 1 1, o número será
1
N = 24 X 3 X 5 = 240
420 2
9) Dotermínar todos os números que admitem 15 di-
210 2 vlsoros e que s6 sejam divisíveis pelos fatôres primos 3 e 7.

105 3 3 X 1 = 3. Temos:
n. d = 15 = 3 X 5
35 5 5 X 1 = 5; 5 X 3 ·= 15. Como no exemplo anterior, os expoentes dos fatôres
primos, que no caso são 3 e 7 serão: 3 - 1 2 e 5 - 1 =
7 7 7 X 1 = 7; 7 X 3 = 21; 7 lX 5 = 35; 4.
1 7 X 15 = 105. Existirão dois números, que são:
N = 32 X 74 21.609 e
8) Calcular o menor número que admite 20 divisores. Nl = 34 X 72 = 3.969.
Temos:
n. d 20= =
2 X 2 X 5 10) Calcular os dois múltiplos consecutivos de 7 que
Depois do que foi dito têm para soma 147.
n. d = (* + 1) (* + l) (* + 1) Dois múltiplos consecutivos de 7 diferem de 7. Portan-
to, se subtrairmos 7 da soma 147, o resultado será o dôbro
'-y--' "'---v-' "--y----'
1do menor dos múltiplos, que terá para valor a diferença
2 X 2 X S
147 - 7 = 140, dividida por 2, isto é, 70.
No interior dos parênteses e no lugar dos asterísticos Um dos múltiplos sendo 70 (o menor) o outro, conse-
(*), devemos escrever: l, 1 e 4. cutivo, será 77.
Os asterísticos estão nos lugares dos expoentes dos fa- 11) Qual o menor número pelo qual se deve multi-
tôres primos que entram na composição do número. plicar 192 para obter um produto múltiplo de 80.
Se pretendemos obter o menor número, devemos esco- Para que o produto a ei;contrcr: seja múltiplo d~ 8.0 é
lher os menores fatôres primos para compô-lo. preciso que contenha os fatores pnmos de 80, multiplica..,
Como o número de divisores foi obtido multiplicando- dos por outro ou outros fatôres.
se o interior_de três parenteses, segue-se que os fatôres pri- Decompondo 80 em fatôres primos teremos:
mos que compõem o número, são três e como devem ser
os menores, terão que ser: 2, 3 e 5. 80 = 24 X 5.
Façamos o mesmo com 192.
Posto isto, o número pedido será da forma
N = 2x X 3Y X 5z,
192 =2 6
X 3

98 Paulo Pessoa
Problerna• de Matemática. para o Admlasão 99
Vê-se pois que em 192 não existe o fator 5 que é o que
será usado para multiplicar 192, de modo a se obter um EXERCfCIOS PARA RESOLVER
múltiplo de 80 e que é:
192 X 5 =
960. 1) Verificar se os números 299, 397 e 797 são primo..
12) Verificar pela decomposição em fatôres primos, se Resposta: São primos 397 e 797. Não é 299.
o número 720 é divisível por 45.
2) Dentre os números 91, 121, 211 e 235, um dêles é
Decompondo-se 720 ein fatôres primos encontraremos:
primo. Destaque êsse número primo.
720 = 24 >< 32 X 5
. Resposta: 211. Ginasial - 1968
Por outro lado
45 = 32 X 5 3) Dentre os números 21, 49, 55 e 59 um dêles é pri-
mo. Escreva êsse número primo.
Vemos então que 720 contém os fatôres primos de 45
elevados aos mesmos expoentes. Então 720 é múltiplo de Resposta: 59. Ginasial - 1968
45 e, portanto, divisível por êle.
4) Verificar se os números
13) Quantos múltiplos de 5 há entre 153 e 314? 1.533
O primeiro múltiplo de 5 que. vem depois do número 1.919
153 é obtido dividindo-se 153 por 5, que dá 30 para quo- 1 . 967 são primos.
ciente.
Então o primeiro múltiplo de 5 que aparece depois de Resposta: Nenhum é primo.
153 é 5 X 31 = 155.
5) Decompor 2. 964 e 5. 544 em fatôres primos e somar
O último múltiplo de 5 que vem antes de 314 é obti-
os expoentes, em cada caso.
do do mesmo modo, isto é: 314 -;-5 que dá 62 para quo-
t:iente e 4 para resto. Então êle é 62 X 5 = 310. Para sa- Resposta: 22 X 3 X 13 X 19. Soma dos expoentes: 5
bermos quantos múltiplos de 5 existem entre 155 e 310, con-
siderando ambos é bastante dividir por 5 a diferença 310 23 X 32 X 7 X 11. Soma dos expoentes: 7
- 155 = 155 e somar um ao resultado. Concluiremos en- 6) Decompor em fatôres primos os números 1. 533 e
+
tão que 155 ..;- 5 1 = 32 múltiplos de 5, nas condições do 1.967 e somar os expoentes, em cada caso.
problema:
Resposta: 3 X 7 X 73. Soma dos expoentes: 3 .
14) Multipliquei um número pelo produto dos três me-
pores números primos. O resultado excedeu de 145 unidades 7 X 281. Soma dos expoentes: 2.
aquêle número. Que número foi multiolicado? 7) Decompor em fatôres primos o produto 30> X 65 X
Os três menores números primos são I. 2 e 3 e seu pro- 125, sem efetuá-lo.
duto 6. Então o número ficou seis vêzes maior do que era,
significando que aumentou 5 vêzes êle (o número), razão Resposta: 2 X 3 X 55 X 13.
pela qual ficou acrescido de 145.
8) Decompor em fatôres primos o produto: 24 X 686
Concluímos depois disso que o número multiplicado foi
X 405, sem efetuá-lo.
o quociente de 145 por 5, que é 29.
Resposta: 24 X 35 X 5 X 78 •
17) Calcular o número de divisores do número N
9) Decompor em fatôres prlmôs a quinta pot~ncia de 25 X 36 X 58.
1.500.
Resposta: 72.
Resposta: 2ª X 34 51 2.
18) Calcular o número de divisores de: N - 42 X
10) Decompor em fatêres primos a quinta potência de X 2.S3 X 343.
645. Resposta: 140.
Resposta: 3i'í X 51) X 43 5 • Para que 2ª X 3 2 admita 18 divisores, qual deve
19)
sAr o valor de a?
11) Decompor em fatôres primos a quinta potência de
360, Resposta: a = 5. Liceu Nilo Peçanha - 1968
Resposta: 2rn 3Jo X 55 • 20) Qual deve ser o valor de n para que o número
63 X 4n admita 54 divisores?
12) Decompor 30 2 X 45~ X 60 4 em fatôres primos. Resposta: n = 4.
Resposta: 2 1 ºX 3 12
X 5u. 21) Sendo N = 2ª X 3x X 5 3 X 7 2 , qual deve ser o
valor de x para que tenha 148 divisores?
13) Calcular o núme10 de divisores de 1968.
Resposta: 20.
Resposta: x = 2.
22) Achar os divisores pares do número 660.
14) Calcular o número de divisores de 1958 e deter-
Resposta: 2 - 4 - 6 - 10 - 12 - 20 - 22 - 30 -
ir;niná-los. 44 - 60 - 66 - 110 - 132 - 220 - 330 e 660.
Resposta: n. d = 8. Divisores: l - 2 - 11 - 22 -
23) Achar os divisores ímpares do número 2. 860 .
89 - 118 - 979 e 1958.
Resposta: 1 - 5 - 11 - 13 - 55 - 65 - 143 ·- 715.
15) Calcular o número de divisores de 180 e determi-
;JJá-los. 24) Achar os divisores pares do número 1 . 968 .

Resposta: n. d = 18. Divisores: 1 - 2- 3- 4- 5-


82 -
Resposta: 2 -
164 - 246 -
4 - 6 - 8 - 12 -- 16 - 24 -
328 - 492 - 984 e 1.968.
48 -
6 - 9 - 10 - 12 - 15 - 18 - 20 - 30 - 36 - 45 - 60 -
90 - 180.
25) Achar os divisores ímpares do número 264.
16) Formar os divisores de 3. 540. Resposta: 1 - 3 - 47 e 141.

Resposta: 1 -
2 - 3 - 4 - 5 - 6 - 10 - 12 - 15 - 26) Calcular o menor_número que admite 30 divisores .
20 - 30 -,§a.- 60 - JJ. 8.. - JIL - ~ - 295 - 354 - R sposta: 720 .
. 590_- ZQL-j§.§.- ~- l.770 e 3.540. - -
Problemws <!e Ma.l"mát.!ca para.,· · o Admfo slló 103
102 Pa.nlo -eess o a '
27) Calcular o menor número que admite 12 diviso- 36) Calcule o menor número dilerente de zero pelo
res. qual d vemos multiplicar 72 para que o produto seja divi-
Resposta: 60. o{v 1por12. l .E. e C. Dutra - 1968
28) Achar os dois menores números que tenham 30 di- n posta: 7.
visores cada um; que sejam divisíveis por 3, 5 e 11 e não
37) Vorificar pela decomposição em fatôres primos, se
sejam por nenhum outro fator primo. o número 420 é divisível por 28.
Resposta: 61.875 e 49.005. Resposta: É.
29) Achar os números que tenham 6 divisores cada
um, que sejam divisíveis por 7 e 13 e não sejam por ne- , 38) Verificar pela decomposição em fatôres primos se 0
nhum outro fator primo. numero 270 é divisível por 14.
Resposta: 637 e l.183. Resposta: Não.
'39) Quantos múltiplos de 7 há entre l . 967 e 2. 835?
30) Quais os dois números que a dmitem 8 divisores e
que não são múltiplos de outros números primos além de Resposta: 122.
2 e 5? 40) Quantos múltiplos de 11 existem entre 2. 837 e
Resposta: 40 e 250. C. Militar. - 1968 3.529?
Resposta: 63.
31) Qual o maior número primo que é divisor de 7 .260?
Resposta: 11. 1. E. C. Dutra - 1968

32) Calcular os dois múltiplos consecutivos de 13 que


têm para soma 65.
Resposta: 26 e 39.
33) Calcular os dois múltiplos consecutivos de 11 que
têm para soma 121.
Resposta: 55 e 66.

34) Qual o menor número pelo qual se deve multipli-


car 135 para obter um produto múltiplo de 90?
Resposta: 2.

35) Qual o menor número pelo qual se dev~ multipli-


car 250 para obter tuna potência da 10?
Resposta: 4.

104 Pa.ulo· Pessoa Problemma de Matemáti ca pa,ra o Admissão 105


MAIOR .DIVISOR COMUM

Maior divisor. comum de dois ou mais números, é o


maior numero que os divide exatamente. Se nao houver
nenhum número que divida os números dados, a lem da
unidade, o maior divisor comum é um e os números consi-
derados dizem-se primos entre si.
Para acharmos o maior divisor comum (m. d. e.) de
vários números, usamos os processos da decomposição dos
números em tatóres primos ou o das divisoes sucessivas,
chamado algarítimos de Euclid.e s.
Quando usamos o primeiro processo, isto é, o da de-
composição dos números, separadamente, em iatôres pri-
mos, o maior divisor comum é igual ao produto dos :tatôres
pnmos comuns, elevados aos menores expoemes.
No caso do processo empregado ser o das divisõe~
sucessivas o maior divisor comum é o penúltimo resto en-
contrado na série de divisões efetuadas e que corresponde
ao Último divisor.
Temos pois, dois processos para encontrar um divisor
comum a varios números, e que é, como o seu nome diz, o
maior divisor comum.
füe (m. d. c.), portanto, divide os números, (dos quais
é o maior divisor comum), exatamente, fornecendo quo-
cientes que são primos entr.e si. ·
Sendo êle o divisor comum maior de vários números,
quando fôr muito grande será igual, no .máximo, ao menor
pos números.
Faremos a seguir uma síntese, sob forma de questi9-
;nário, que uma vez memorizada pelo aluno, servira como
um roteiro para a solução de problemas de maior divisor
comum.
EXERCiCIOS RESOLVIDOS
Vejamos a síntese:

a) O que é o m. d. e. de vários números? l) Achar o m. d. c. dos números 4 .725, 3 . 450 e 8 .655,


:t o maior número que divide exatamente os m'.i, decompondo-os em fatôres primos.
meros dados. 4.725 3 3.450 2 8.655 3
b) Como se chama o m. d. c. de vários números? 1.575 3 1.725 3 2.885 5
Pela deepm.posiçéio .dos n4nu~ros, em fatôres primos, 525 3 575 5 577 577
separadamente ou pelas divisões sucessivas. 175 5 115 5 1
35 5 23· 23
c) A que é igl1.al .o m. d. c. de vários niimeroa? 7
7 1
No caso de, .em .-sua procura, ser empregado o pro- 1
cesso da deco~posição em fatôres primos, o m. d. e.
é !gual ao produto dos fatôres primos comumns, Então:
elevados aos menores expoentes. Se a sua procura
fôr feita através dos algarJtimos de Euclides será 4. 725 =
33 X 52 X 7; 3. 450 = 2 X 3 X 52 X 23;
o penúltimo resto, ,q ue é o último divisor. 8 655 = 3 X 5 X 577.
d) Quando ·dividimos · os números considerados pelo Sendo o m. d. e. o produto ·dos fatôr.e s· primos -comuns;
seu m. d. c., como são os quocientes encontrados? elevados aos menores expoentes; concluímos que seu va-
Primos entre .-si. lor será:
e) Qual o maior valor dom. d. c. de vários números?
Em cértos casos êle será, no máximo, igual ao me- m. d . e. = 3 X 5 = 15
nor dos números.
2) Calcular o m. d. c. dos números do problema an-
f) Dos divisores comuns .a vários números, qual aquê- terior pelo processo das divisões sucessivas.
le que sqbemos achar? De preferência tomemos os dois maiores números para
O maior divisor comum dos números. a primeira série de divisões sucessivas.
g) Quando o m . d. c. de -vários números é unidade,
como são os -números? 4 "16 2 Linha dos c.<uoclentes
....
Primos entre si. 1
8G5 5 1 4725 1 1 1 1 1
1 3930 i 795 1 750 1 46 1 30
\ 1
15 1 Linha dos divisores

~
1 1
Propriedades do menor múltiplo comum 3P80 795 750 45 1 ªºº 1 16
o
Linha dos restos

1 30
\
1) Quando -multiplicamos ·ou dividimos vários núme- 1
ros por uma mesma quantidade, o m. d. e. fica multipli- 1
1 1 1
<::ado ou dividido pela mesma -quantidade.
Procuremos agora o m. d .. e. do terceiro número 3. 450 e
2) .Todo divisor comum de dois ou mais números é do m. d. e. dos outros dois números e que achamos ser 15.
divisor do m. d. c. dêsses números. ·

1 108 Pe.u!Q Peuoa


Então:
230 E conclui.remos . que o tn . d .e. procurado é 20 ·

3450 15 ! linha dos divisores 2450 - - - - -


1640 - m. d . c.=i 40 (1)
1 820 m d e ~' 20 (2)

45 ! linha dos restos Vê-se que os resultados (1) e (2) são diferentes, como
dissemos que podia ocorrer, ,q uando expUcamos o exer-
00 1
cício 2.
Concluímos ser 15 o m. d. c. dos 3 números dados.
Costuma-se esquematizar o que foi dito da seguinte 4) O m. d. e. de 2 números é 12 e os quocientes en-
maneira· contrados na pesquisa por divisões sucessivas são: 2, 2 e 5.
8655 ~ Determinar os números.
------ m. d. c. (15)\ (1) Procedamos como nos acsos anteriores 2 e 3 e repre-
4725 ------- .sentemos por A e B os números procurados e por R, R' etc.
3450 · m. d. c. (15) (2) os diversos restos encontrados .
~ste segundo resultado (2) é a resposta do problema. Assim:
Convém ressaltar que o resultado (1) podia ser um número 2 2 1( 5
P,iferente do encontrado em (2). como veremos no exemplo 1
seguinte.
3) Calcular o m. d. c. dos números 2 .480, 1.640 e 820. A B R tR 1

Como no exemplo anterior 1


1
l l 1 2 linha dos quocientes R Rl o 1
1
1
2480 i 1640 840 800 40 linha dos divisores 1.

1 R1 sendo o penúltimo resto e conseqüentemente o úl-


.,, 1 timo divisor, será o m. d. c., então é, 12.
840 1 800 40 00 linha dos restos
A seguir calcularemos o m. d. c. entre 820 (3. 0 número Podemos então escrever:
do problema) e 80 (m. d. c. dos outros dois números). 2 2 T 5 linha dos quocientes
Vem: 1 1

20 2 linha dos ,q uocientes


A B R 1. 12 lirihà dos divisores
820 40 1
20 linhas dos divisores 1
. •·
1
020 00 ·1 linha dos restos · R 12 o 1
linha dos restos
1
110 Pa.ulo Pessoa
Problemn-s de Matemãtlca para o Adm!Rslío 111
5 é o quociente ,de uma diviaão exata cujo divisor é 12 No cálculo do m . d . e . de dois números pelas divisões
e o dividendo é R. sucessivas, o último quociente não pode ser a unida de. Pôsto
Então: isto, porque o problema diz que êles são três e os menores
R = 2 X 5 + 0 = 60 possíveis, s6 poderão ser 1, l e 2 e como da vez anterior:
A situação pas,sa a -ser então: l l 2
2 2 5 1

1
A B R 1
R1
12 1 1
I~
60 12 o R Rt o 1
1
i 1
1 1
1 /,
Novamente teremos ,q uociente 2; divisor 60 e re·sto 12,
faltando apenas calcular B, que é o dividendo. Então: Onde R1 32 e teremos por isso
B = 2 X 60 + 12 = 132 1 2
Teremos finalmente:
i
2 2 5 A i B R 32
i
A 132 60 12 32 o
60 12 o
E, como vimos, R =2 X 32 + O = 64.
Um quociente 2; um divisor 132 e um resto 60, faltan- Podemos então esquematizar novamente:
do apenas calcular o dividendo A e teremos seu valor,
como das vêzes anteriores: 1 l 2

A = 2 X 132 + 60 = 324
A B 64 32
Os números procurados s.ão pois 324 e 132.

5) O m. d. e. de dois .números é 32 e os três quocien- 64 32 o


tes encontrados na pesquisa pelos divisores sucessivos, são
os menores possíveis. Determinar os números.

J. }2 Pn.11!0 ·Pe.s soa Problemas tlc Matemática para o A d mi ss ão J.13


- - -- --~ --
I:ntão:
Então B = 1 X 64 + 32 = 96 e
1
Finalmente 'HIO 2 2
11!0 2 4
l 1 2 'HI 2 li
'1 'I :1 () 12 - 24
1 11 'I 1) 10 36 - 72
A 96 64 32 '• :1
!i l () .o 40 - 15 - 30 - 60 - 120
'1' DO 100 . 360

64 32 o /\qo1a ' n6 1-1 l cionú.-los: 120 e 180.


U) O m. d. e. de dois números é 11.
s multiplicarmos cada um dos números por 22 X 5
q11ol o nôvo m. d. c.
A = 1 X 96 + 64 ::::::l 160 Vimos que, multiplicando-se ou dividindo-se vários nú-
Os números são pois 160 e 96. ro.s p~r uma quantidade, o m. d. c. aparece multiplicado
lll
u d1v1d1do pela mesma quantidade.
6) Achar os três maiores divisores comuns aos núme- Então, se o m. d. c. de dois números era 11 multipli-
ros 1.440 e 1.080 cando-se cada um dêles por 22 X 5 ou seja 20, ~ m. d. c.,
quo era 11 passará a ser 11 X 20 = 220.
O item (f) do roteiro diz que: dos divisores comuns a
vários números sabemos achar o: maior divisor comum. 9) Achar os divisores comuns dos números 200 e 440.
Dos divisores comuns a vários números, iremos achar
Então, dos três divisores maiores o maior é o m. d. c., o maior (m. d. c.) (f do roteiro).
que é 360.
Vamos então achar o m. d. c. dos números 200 e 440,
Achado o maior, basta dividí-lo sucessivamente pelos que é, pelas divisões sucessivas.
seus menores divisores que são 2 e 3, para acharmos: 180
e 120. 2 5
Os divisores pedidos são: 360, 180 e 120. 1
440 1 200 40
7) Quais os divisores comuns de 1 . 440 e 1 . 080 com - 1
preendidos entre 100 e 200? 40

Como vimos no exemplo anterior o maior divisor co-


1

1
ºº
1
;murn entre os números dados é 180 m. d. c. Como quere-
mos os compreendidos entre 100 e 200 é bastante achar os 1
1
divisores de 360 e selecionar os do intervalo pedido.
Problemas de MalemáLica l'"""'' o AdrnÍs tiáo 115
114 Po,ulo Pessoa
igual a 40. A 2.ª propriedade do m. d. c. diz: Os divisores
do m. d c. são também divisores do número, por isso é bas- Dois números pares, quaisquer que êles sejam, são di-
tante calcular os divisores de 40 visíveis por 2. Então o seu m. d . e. é 2.

1 13 Determinar os 3 menores números pelos quais de-


40 2 2 vemos dividir 320, 250 e 440 para que os quocientes sejam
20 2 4 iguais.
10 2 8 Já dissemos que o m. d. c. é o melhor número para di-
5 5 5 - 2 - 20 - 40 vidir outros, quando êle é o m. d. e. daquêles outros .
1 Calculemos pois o m. d. c. dos números dados:
320 2 280 2 440 12
160 2 140 2 220 12
Os divisores de 40 são os divisores comuns aos nú- 80 2 70 2 110 12
.meros 200 e 440. 40 2 35 5 55 j5
10) Achar o maior número pelo qual devemos dividir 20 2 7 7 11 1 11
254 e 352 para que os restos sejam 4 e 2, respectivamente. lO 2 1 1 1
O melhor núme:ro para dividir outros é o m. d. c. dos 5 5 l
números. 1 1
Suponhamos que 254 são laranjas e 352 são maçãs.
Se queremos dividir as laranjas e as maçãs pelo maior
Então 320 = 2 6 X 5; 280 1= 23 X 5 X 7 e 440 = 2ª 1
X
X 5 X 11 e o m. d. c. é:
número de pessoas mas desejamos ficar com 4 laranjas e
2 maçãs, devemos de início retirar dos totais de laranjas e n. d. c . 23 X 5 = 40
maçãs a dividir, aquelas com que desejamos ficar.
Se assim procedermos restarão 254 - 4 =
250 laran- Façamos as divisões
üas e 352 - 2 = 350 maçãs para dividir pelo maior número 320 ,j 40 280 ~ 440 1 40
de pessoas.
O maior número de pe·s soas é o m. d. e. dos números
250 e 350 e que é 50.
ºº 8 00 7
Observando-se os resultados encontrados, verificamos
40 11

Então, 50 é o maior número pelo qual dividindo-se 254 que os divisores são iguais e os quocientes diferentes.
e 352 sobram, respectivamente, os restos de 4 e 2. O que o problema pede é que sejam iguais os quo-
cientes e os divisores não.
11) Qual o m. d. c. de dois números ímpares conse-
cutivos? Basta então que troquemos os quocientes pelos divi-
Dois números ímpares consecutivos são primos entre sores para se ter

,__
si, então o seu m. d. c. é a unidade.

12) Qual o m. d. e. de doüi números pCTres consecuii ·


320
,__
18 280
__
7
,1 440 111
00 40 00 40 00 40
VOB?
Onde os quocientes são iguais e os divisores diferentes.
116
· · - - - ---·-·-. .. ---- --
Problemas de M~temâtica pa.r11 o Admissão
---- -- --
117 -
No entanto os cêstos podiam conter tangerinas e la-
Considerando que 8, 1 e 11 foram. os quõeientes dds ranjas alterando a resposta do problema.
divisões dos números pelo maior de seus divisores (m. d. No caso em questão, cada caixa deve conter bolas das
e.) conclui-se íàcilmente que êles são os menores. três côres, isto é, verdes, vermelhas e amarelas. Nessas
Então 8, 7 e 11 são os menores números pelos quais condições, o m. d. c. do número de bolas de cada côr irá
dividindo-se 320, 280 e 440 obte:rn-se ,q uocientes iguais. representar o total de caixas, permitindo assim que o me-
nino possa ter o maior número de caixas. Assim sendo, o
14) Colhi 300 tangerinas e 850 laranjas para vendê- número de caixas será 16.
~as em cêstos contendo o maior número possível de irutos
da mesma espécie. Pergunta-se: quantos cestos vendi e por Em cada caixa haverá: 112 + 16 =
7 bolas verdes.
quanto vendi cada fruto, sabedo-se que cada cêsto foi 48 + 16 = 3 bolas vermelhas e 80 + 16 = 5 bolas ama-
vend.ldo por 7,bu cruzeiros novos. relas, num total de 15 bolas em cada caixa.
Se os cestos devem conter o maior. número possível da
16) Duas estradas que se cortam em um T têm 5. 980
mesma espécie e o número de frutos nos cêstos deve ser o
mesmo, conclui-se facilmente que aquêle maior número .e 3. 360 metros respectivamente. Pretende-se colocar postes
de iluminação ao longo das estradas, de modo que exista
de irutos aeve ser o m. d. c. do número de iaran1as e tan-
um poste no cruzamento das estradas e um em cada ex-
gennas, que é 5U.
tremidade do trecho considerado. Exige-se que a distância
Dividindo-se 300 tangerinas por 50 (50 em cada cêsto),
entre cada dois postes seja a mesma e a maior possível.
conclui-se que são necessários 6 cêstos para conter as tan-
germos. \;,,uu -7- bU = o). Pergunta-se quantos postes serão empregados?
, Do mesmo modo são 17 (850 -7- 50 = 17) cêstos petra , Como a distância tem que ser a mesma e a maior pos-
·c onter as laranjas. Então o total de cêstos é: ~1vel, temos que calcular o maior número capaz de dividir
ao mesmo tempo 5. 980 e 3 . 360 metros. ~sse número é o
6 + 17 = 23. ;m. d. c., que uma vez achado nos diz ser 840 metros que
corresponderão às distâncias entre cada dois postes.
Como em cada cêsto existem 50 tangerinas ou 50 la-
ranjas, e cada uma delas foi vendida por NCr$ 7,50, segue- Dividamos cada um dos comprimentos das estradas por
sa que cada tangerina ou cada laranja custou: 840 metros e encontraremos 7 e 4, que correspondem aos
n úmeros de intervalos marcados pelos postes.
Ncr$ 7,50 -7- 50 = Ncr$ 0,15. Para separarmos cada 'uma das 7 partes em que vai
ser dividi.da a estrada maior, precisaremos de 8 postes e
15) Um menino possui 112 bolas verdes; 48 vermelhas para separarmos cada uma das 4 partes em que vai ser
e 80 amarelas. Deseja arrumá-las em caixas de modo a d ividida a estrada menor precisaremos de 5 postes .
;c onterem o mesmo e o maior número possível de bolas no
total e côres. Acontece porém, que o poste colocado no cruzamento
Quantas caixas são necessárias e quantas bolas de ta nto servirá para limitar as partes da estrada menor como
cada côr figurarão em cada caixa? para marcar uma das extremidades da estrada maior.
É conveniente ressaltar o modo escolhido pelo menino Não fôsse a circunstância assinalada e seriam necessá-
para colocar as bolas nas caixas: +
rios: 8 5 = 13 postes para as duas estradas. Diante delas,
No problema anterior os cêstos continham só tange- bastam apenas 13 - 1 = 12 postes.
rinas e sàmente laranjas. ·
rrobluuutti de Mutcmúticu imrn, o Admi sa üo J.} 9
17) O m. d. c. de três números é 96. Se dividirmos ca- 2) Calcule o m. d. e; dos números seguintes pelas di-
da um dos números por 24 X 3 qual o m. d. c. dos quo- visões sucessivas:
cientes obtidos? '
Vimos que multiplicando-se ou dividindo-se vários nú- a) 1.577 e 2.093
meros por uma quantidade, o m. d. c. dos vários números b) 406, 435 e 493
aparece multiplicado ou dividido peia mesma quantidade. e) 1.106 e 790.
No caso em questão os números cujo m. d. c. é 96 serão
divididos por 24 X 3 ou 48, por isso o m. d, c. apar:ecerá Resposta: a) 91; b) 29 e c) 158.
dividido por 48 e posará a ser 2, isto é 96 --;- 48 = 2.
18) O salto de um coelho tem mais de 20 decímetros 3) Se M = 16 X 3; N = 32 X 125 e P = 3 X 5 2 , qual
e menos de 30 decímetros. Ao percorrer 240 decímetros dá é o m. d. e. de M, N e P?
um número exato de saltos e o mesmo acontece ao per- Resposta: 1. C. Militar - 1968
correr 336 decímetros. Qual o comprimento do salto?
Para que o comprimento do salto esteja contido em 4) Aqui temos dois números:
número exato de vêzes em 240 e 336 é preciso que seu va-
lor seja igual a um número que os divida exatamente. A = 23 X 3 X S
Vimos que o melhor número para dividir números é o B = Tf X :J2 X 7
m:. d. c. dos números.
ProcuI'emos então o m. d. c. de 240 e 336. Aplicando-se Observe e indique, apenas, o m. d. c. dêsses números?
qualquer dos processos já explicados, concluímos que o m.
d. c. é 24, que representará o comprimento do salto do Resposta: 24. Liceu: Nilo Peçanha - 1968
coelho, isto é, 24 dm.
O fato do problema dizer que o valor do salto está 5) O m. d. c. de dois números é 12. Na sua procura
compreendido entre 20 dm e 30 dm é para o problema pelos algarítimos de Euclides encontram-se os quocientes l,
não ter mais de uma solução, pois se não fôsse estabeleci- 1 e 3. Quais são êsses números?
da restrição, qualquer divisor de 24 indicava uma solução
para o problema que teria tantos quantos são os divisores Resposta: 48 e 84.
de 24 ou 23 X 3, que tem para número de divisores: n. d.
(3+ 1) (1 +
1) = 8 soluções. 6) Na procura do m. d. c. de dois números encon-
tram-se três quocientes iguais a 1 e o quarto igual a 4.
Sabendo-se que o m. d. c. dos números e igual a 6, calcular
EXERCíCIOS PARA RESOLVER os dois números.
1) Calcular o m. d. c. dos números seguintes pela de- Resposta: 84 e 54.
composição em fatôres primos:
a) 108, 144 e 216 7) Na procura dom. d. c. de dois números pelo proces-
b) 220, 300 e 630 s o das divisões sucessivas, foram encontrados em ordem
c) 792, 1. 05-S e 1. 848. os quocientes 2, 4 e 6. Se o penúltimo resto é 9, quais
são os dois números?
Resposta: a ) 36; b) 10 e c) 264.
Resposta: 504 e 225. C. Militar - 19138

Probiem11:11 de Mo.temática pua " Admissão 121


. -----·-- - ·- - - - -- - - -
8) O m. d. c. de dois números é 16 e os três quocien-
ies encontrados na pesquisa pelas divisões sucessivas são 16) Achar os divisores comuns dos nfuneros 936, 792
os menores possíveis. Determinar os números. e 504.
Resposta: 80 e 48. Resposta: 1 - '2 - 3 - 4 - 6 - 8 - 9 - 12 - 18
24 - 36 e 72.
9) O m. d. c. de dois números é 15 e os quatro quo-
cientes encontrados nas divisões sucessivas, são os me- 17) Achar os divisores comuns dos números 252, 468
nores possíveis. Determinar os números. e 396.

Resposta: 120 e 175. Resposta: i- 2- 3- 4- 6- 9- 12 - 18 e 36.


18) O m. d. e. de dois números é 7. Se multiplicarmos
10) Tem-se: A = 23 X 3m X 5 X 7 e B = 2° X 32 X 7: cada um dos números por 32 X 11, qual é o nôvo m. d . c.?
calcule m e n de modo. que o máximo divisor comum de
A e B seja 252. Resposta: 693.
Resposta: n =2 e m = 2. C. Pedro II - 1968 19) O m . d. c . de três números é 12 . Se multiplicarmos
cada um dos números por 3 X 52 qual é o nôvo m. d. c?
11) Calcule os 4 maiores divisores comuns de 1. 872;
1 . 584 e 1. 008. Resposta: 900.
Resposta: · 144, 72, 48 e 36. 20) O rn. c. d. de quatro números é 124. Se dividirmos
cada um dos quatros números por 2 X 31 qual o m. d. e.
12) Calcule os três maiores divisores comuns dos nú- dos quocientes obtidos.
números 128, 72, 40 e 32.
Resposta: 2.
Resposta: 8, 4 e 2. Liceu Nilo Peçanha - 1968
21) O m. c. d. de três números é 144. Dividindo cada
13) Dê todos os divisores comuns aos números 72 e um dêles por 22 X 3, o m. d. c. dos quatro números será?
84 que sejam divisíveis por 3.
Resposta: 12. l. E. e C. Dutra
Resposta: 3 - 6 e 12. Liceu Nilo Peçanha - 1968
22) Qual o maior número pelo qual devemos dividir
14) Quais os divisores comuns de 468, 396 e 252, com- 243 e 391, para que os restos obtidos sejam respectivamen-
preendidos entre 4 e 18? te 3 e 7?
Resposta: 6, 9 e 12. Resposta: 48.

15) Quais os divisores comuns dos números 1 . 800, 23) Determinar o maior número pelo qual se deve di-
2. 400 e 3. 900, compreendidos entre 75 e 300? vidir 1. 647 e 1. 325 para se obter os restos 7 e 5, respecti-
vamente.
Resposta: 100 e 150.
Resposta: 40.
}2,2: Pat\lo Pesa.oa ,.'..; · .
em caixas com o mesmo número de botões de cada tama-
24) Dividindo-se os números 2.483, 1.645 e 1.331 pelo nho, e tendo o maior número possível de botões, quantas
tnaior número possível obtém-se os restos 3, 5 e 11, respec· serão as caixas ao tôdo?
tivamente. Determinar êsse número.
Resposta: 10 caixas. C. Pedro II - 1968
Resposta: 40.
30) Desejamos arborizar o contômo de um terreno .: e-
25) Pedem-se os três menores números pelos quais tangular de 1. 320 m por 1.456 m, mantendo as árvores
devem ser divididos respectivamente os números 2. 480, eqüidistantes com a maior distância possível. Se em cada
1. 320 e 1648, para que os três quocientes obtidos sejam canto plantarmos uma árvore, quantas serão necessárias?
iguais.
Resposta: 694 árvores.
Resposta: 62, 33 e 40.
31) Um triângulo tem para lados 9 m; 12 m e 5 m.
26) Quais os três menores números pelos quais se de- Deseja-se iluminar os seus lados com postes de luz equi-
vem dividir os números 208, 816 e 2.112 para que os quo- distantes e com a maior distância possível. Se em cada
cientes sejam iguais? vértice colocarmos um poste, quantos serão necessários?
Resposta: 13, 51 e 132. Resposta: 12 postes.
27) Três peças de fazenda têm respectivamente 168 i:n 32) Um terreno retangular tem 120 m de comprimen-
254 m e 312 m. Deseja-se cortá-los em parte de compri- to e 75 m de largura. Para medí-lo quer-se usar uma trena
mentos iguais, de maneira que cada parte seja a maior .de maior comprimento possível, que fique contido um núme-
possível. Qual o comprimento de cada parte e o número ro exato de vêzes nas suas dimensões. Qual deve ser o
de partes? seu comprimento para que tal aconteça?
Resposta: Cada parte tem 24 m e o número de partes Resposta: 15 m.
é 31.
28) Um quitandeiro resolveu distribuir 36 laranjas, 60
:abacates e 84 cajus, com várias crianç~s, de mo~o que
cada um recebesse o mesmo e o maior numero poss1vel de
frutas de cada espécie . Pergunta-se o número de crian; as
aquinhoadas e o número de frutas de cada espécie q ue rece-
beu cada criança?

Resposta: 12 crianças
3 laranjas
5 abacates
7 cajus.

29) Um comerciante para vender 180 botões grandes,


71 botões médios e 108 botões pequenos, deseja colocá-los
l'1·9bl•mas de :Matem~ller, parit o Admi•são J.2S

MÍNIMO MULTIPLO COMUM

Mínimo múltiplo comum ou menor. múltiplo comum de


dois ou mais números, é o menor número divisível pelos
números dados.

Para acharmos o mínimo múltiplo comum (m. m. c.) da


vários números, usamos a decomposição dos números em
fatôres primos, ao mesmo tempo (processo comum) ou se-
paradamente. Podemos também calculá-lo com o auxílio do
maior divisor comum, se não quisermos decompor os núme-
ros em fatôres primos.

Quando usamos a decomposição dos números separa-


damente, em fatôres primos, o mínimo múltiplo comum é
igual ao produto dos fatôres primos comuns e não comuns,
elevados aos maiores expoentes.

Assim, por exemplo, seja calcular o m. m. c. dos nú-


meros 2 . 480, 1. 640 e 1 . 320, decompondo-os em fatôres pri-
mos separadamente.

Se assim fizermos encontraremos:

2 . 480 24 X 5 X 31
1.640 2?. X 5 X 41
1. 320 = 23 X · 3 X 5 X 11

Então o m . m . c . = 24 X 3 X 5 X 11 X 31 x 41
3. 355. 400.

Problemas de Mateml\tlca para o Adm!oo íio 12·7


----- --------- --------------
~ualquer n~mero, ,pô:rà a substituição desde ,q ue fôsse mul-
Se a decomposição for feita em conjunto, o menor múl- llplo dos vanos numeres a substituir. Essa escôlha torna-se
tiplo comum é igual ao produto dos fatôres primos que mais fáci~, ~e em lugar de qualquer múltiplo escolhermos o
forem escritos à direita do tra~o vertical. menor mult1plo comum, pois como vimos existem processos
para calculá-lo.
2.480 1.640 1.320 2
1.240 820 660> 2 , . Por ~sso não devemos esquecer que dos múltiplos de
620 410 330 2 vanos numeres, sabemos achar o menor (menor múltiplo
310 .205 165 2 c~mum). O m. m. c. de vários números é divisível pelos
155 205 165 3 nu_meros e os quocientes de s uas divisões são números pri-
155 205 55 5 m1s entre si e divisores do m. m . c.
31 41 11 11 Para que seja divisível pelos números é preciso que
31 41 1 31 embora muito pequeno não seja menor do que o maior dos
1 41 1 41 números, do qual é o m. m. c.
1 l l
No caso dos números cujo m. m. c. queremos achar,
o m. ru. c., como foi dito, é: serem primos entre si, seu valor será o produto dos nú-
meros.
m. m. c.: 24 X 3 X 5 X 11 X 31 x 41 = 3.355.400 .
Assim o m. m. c. de 2, 5 e 8, é: (números primos entre si)
Se pretendermos obter .o ·m · m . c. de dois números,
com o auxílio do m. d. c. (decompondo-os ou não em fa-
m. m. c. = 2 X 5 X 8 = 80.
tôres primos), empregaremos a seguinte expresão, que tam- Como fizemos quando tratamos do maior divisor comum
bem servirá para calcular o m. d. c. de d.ois números se daremos a seguir uma síntese, que prestará os mesmos ser-
conhecermos os dois números e o m. m. c. dêles, viços aos alunos, como prestou a do m. d. c.
Se chamarmos de A e B os dois números a expressão
referida é: Vejamos:

A X B = m. d . e . .x m . m. e, a) O que é o m. m. c. de vários números?


que nos permite escrever: É o menor número divisível pelos números dados.
b) Como se acha o m. m. c. de vários números?
A X B Descompondo-os em fatôres primos separadamen-
m. m. c. te ou em conjunto ou com o auxílio do m. d. c.
m. d. c. ou
c) A que é igual o m. m. c. de vários números?
AXB Decomposto separadamente, é igual ao produto dos
m. d. c. fatôres primos comuns e não comuns, elevados aos
m. m. c. maiores expoentes.
De;ompo~tos em conjunto, é igual ao produto dos
Muitas vêzes há conveniência na substituição de vá- fatores pnmos escritos à direita do traço vertical.
rios números por um único. Neste caso poderíamos escolher
P1·0)1Jen111>s de MatemiHica pa»a o Admissiío }29
128 Pa.ulo Pe ..oa
Através do m. d. e. (sendo 2 mS.meros), com ô âuXÍ·
lio da expressão
:EXERCfCIOS RESOLVIDOS
A X B
1) Determine pelo processo da decomposição em fa-
m. m. e . = AeB
m. d. c.' tôres primos, separadamente o m. m . c. dos números 40, 60
sendo os dois números. e 90.
Temos:
d) Para que serve o m. m. e.?
Para substituir vários números dos quais êle seja 40 2 so 2 90 2
o m. m . e. e de um modo mais prático, para ser di- 20 2 30 2 45 3
vidido pelos números. 10 2 15 3 15 3
e) Como são os quocientes das divisões do m. m. e. 5 5 5 5 5 5
pelos números? 1 1 1
J?rimos entre si e divisores do m. m. c.
f) Qual o menor valor do m. m. c. de vários números? Podemos escrever:
Nunca poderá ser menor do que o maior dos nú-
meros. 40 = 23 X 5; 60 =2 2 X 3 X 5; 90 ,- 2 X 32 x 5
g) Dos múltiplos de vários números qual o que sabe-
O m. m. c. procurado dêsse modo é igual ao produto
mos achar?
dos fatôres primos comuns e não comuns, elevados aos
O menor múhiplo comum. maiores e xpoentes.

Propriedades do menor múltiplo comum Então

a) Quando multiplicamos ou dividimos dois ou mais


m. m. c. = 23 X 32 X 5 = 720.
números pe o mesmo n úmero, o m. m. c. dêsses nú-
2) Determine o m. m. c. pelo processo de decomposi-
meros hca multipl.J.caâo ou d1vidicto pelo mesmo nú-
ção em conjunto ou simultâneamente dos números : 340, 890
mero. Por exempiO: O m. m. c. de l~. 15 e 18 é 180.
e 1.400.
O m. m. e. de i2 X 2 = 24; de 15 X 2 =30 e de
18 X 2 =
36, o m. m. c. é 180 X 2 = 360. Temos
b) Dado dois ou mais números, se o maior fôr divisí-
vel pelos outros, será êle o m. d. e. dos números. 340 890 1.400 2
Por exemplo: O m. m. c. de 24, 12, 6, 4, 3 e 2 é 24 170 445 700 2
porque 24 é divisível por 2, 3, 4, 6 e 12. 85 445 350 2
c) Quando elevamos vários números a uma mesma 85 445 175 5
17 89 35 5
potência o seu m. m. e. fica elevado a essa potência.
17 89 7 7
Assim o m. m. c. de 12, 20 e 15 é 60. Então o m.
17 89 1 17
m. c. de 12 2 , 20 2 e 15 2 é 60 2 •
l 89 1 89
l l l
130 P11,uJo

.- ---- ---·- - - - - - -- - -
8J Qual o menor n&meró que dividido pôr 15, 21 e 35
deixa sempre resto de 13. Para que o número considerado seja divisível por 3 e
Se o número não deixasse resto quando dividido por 15, por 7, é preciso que seja múltiplo de 3 e múltiplo de 7.
21 e 35, seria múltiplo de 15, 21 e 35 e poderíamos escrever: O múltiplo de 3 e 7 que sabemos achar é o m . m. c.
dêles igual a 3 X 7, por serem primos entre si.
N m. 15 (m - múltiplo).
Conhecido o menor, basta calcularmos os seus múlti.
N m. 21
plos (de 21), de dois algarismos, como pede o problema,
N m. 35 para acharmos os números que o satisfazem.
Como deixam sempre resto 13, poderemos ecrever: Assim:
N m. 15 + 13 21 X 1 = 21
N m . 21 + 13 21 X 2 = 42
N m . 35 + 13 21 X 3 = 63
21 X 4 = 84
e de um modo mais simples:
Não prosseguiremos na determinação dos múliplos de
rm 15 l 21, pois o seguinte ao último encontrado, já contém três
N = im 21 r + 13 algarismos.
Lm 35 J
11) Calcule os números compreendidos entre 1. 000 e
Considerando que dos múltiplos de vanos números sa- 3. 000, que sejam divisíveis por 24, 30 e 50.
bemos (temos processos) achar o menor (roteiro - g) é bas· Para os número serem divisíveis por 24, 30 e 50 é pre-
tante achar o m. m. c. dos números 15, 21 e 35, que sabe- ciso que sejam múltiplos dêles. Dos múltiplos dêles sabe-
mos ser 105 e então escreveremos: remos achar o menor múltiplo comum, que é 600 que está
fora dos limites fixados.
r 105 l
i Procuremos então os seus múltiplos compreendidos
N r+ 13 ou
ntre 1.000 e 3 . 000.
L J
N = 118 Teremos:
600 X 2 1.200
9) Qual o menor número divisível ao mesmo tempo 600 X 3 1.800
por 18, 24 e 48? 600 X 4 2 .400
O item d do roteiro diz ser o m. m. c. o melhor número
para ser dividido por outros. não prosseguiremos pois o seguinté é 3 . 000 e não serve.
Basta então acharmos o m. m. e. de 18, 24 e 48, que
é 144 para ter resolvido o problema. 12) Calcule os três menores números pelos quais se
d .vem multiplicar 12, 16 e 18, respectivamente, de modo
10) Determinar os números de dois algarismos, divi- quo os produtos sejam iguais.
iSÍveis por 3 e 7. Se depois de multiplicarmos 12, 16 e 18 por três núme-
1 os diferentes os produtos se tornam iguais é porque os re-
134 Paulo Pessoa

Probl emn·s de Matemática p1ua o Admis são 135


sultados achados (produtos), são múltiplos de 12, 16 e 18. Para deixar resto 1 quando dividido por 2, 3 e 5 é pre-
Dos múltiplos de vários números, sabemos achar o menor ciso ser, no mínimo
(m. m. c.); devemos pois procurar o m. m. c: de 12, 16 e 18,
que é 144 e, em seguida dividí-lo sucessivamente por 12, 30 +
1 = 31,
16 e 18. que não é divisível por 13.

Assim: Procuramos então um múltiolo de 30, isto é, 60 que


144 --:--- 12 = 12; 144 --:--- 16 =9e 144 --:--- 18 = 8 para deixar resto um quando dividido por 2, 3 e 5, terá
que ser
:f:sses quocientes, 12, 9 e 8 são os números pedidos.
60 + 1 = 61
13) Determinar o menor número ao qual faltam 7 uni-
dades para ser ao mesmo tempo divisível por 12, 40 e 48. mas que também não é divisível por 13.

Como no exercício 8 podemos escrever: Procuramos outro múltiplo do m. m . c . 30 e teremos


3 X 30 ,_ 90 que terá aue ser acrescido de um para deixar
N = m . 12 7 resto um, quando dividido por 2, 3 e 5, sendo portanto
N =
m. 40 7
90 +1= 91.
N = m . 48 7 ou

m. 12 } Como 91 é divisíve l por 13, segue-se que é o número


N m. 40 - 7 ou procmado.
{ m . 48
16) Achar o menor número múltiplo de 30 divisível
N 240 - 7 = 233, que é o número pro- por 54.
curado. Para o número procurado ser múltiplo de 30 e divisível
por 54, terá que ser também múltiplo de 54, por isso a so-
14) Achar o menor múltiplo de 13 que dividido por
15, 24 e 40 deixa sempre resto 10.
lução do problema consiste em achar o m. m. c. entre 30
e 54 que é 270.
Temos que calcular um número que dividido por 15,
24 e 40 deixe sempre resto 10. Dos vários números que po- 17) Sendo 120 o m .· m . c. de três números, qual será
dem ser encontrados procuraremos aquêle que seja múlti- o m. m . c. de três números que são respectivamente o
plo de 13. triplo de cada um dêles?
O m. m. c. de 15, 24 e 40 é 120. A propriedade (a) nos autoriza a dizer que o m. m. c.
Para que a divisão dôle por 15, 24 e 40 dê resto 10 é 360, isto é, 3X 120.
o número terá que ser
18) Sendo o m . m. c. de dois números 180, qual será
120 + 10 = 130 que é dividido por 13. o m . m. c. de dois números que são respectivamente a me-
15) Achar o m enor múltiplo de 13, que dividido por tade de cada um dêles?
2, 3 e 5 deixa sempre resto 1. A m esma propriedade (a) nos autoriza a dizer que o
O m. m. c. de 2, 3 e 5 é 30. m. m. c. procurado é 90, isto é, 180 --:--- 2.

··-
136 Paulo Pessoa
--- - -
Prnblcmii;s ue Matemática pa.r a o Admissão 137
, bs períodos dos plantões dos enfermeiros são múlti-
19) Qual a menor quantia que pode se pode obter plos de 15, 35 e 21. Para voltarem a ficar juntos novamente
grupando notas de NCr$ 5, 00 e NCr$ 10, 00 . é preciso que decorra um tempo que seja múltiplo de 15,
35 e 21 e além disso o menor.
A menor auantia será múltipla, respectivamente de
Então o m. m. c. dos tempos que é 105 dias, findos os
NCr$ 5,00 e NCr$ 10,00. Será portanto o m. m. c. entre
NCr$ 5,00 e NCr$ 10,00 q ue é NCr$ 10,00. quais ficarão juntos novcnnente.

20) Tenho mais de 200 livros e menos de 300. Con- Considerando que ficaram juntos no dia 5 de março,
tando-os de 8 em 8, 10 em 10 e 15 em 15 sempre sobram 5. teremos:
Quantos livros possuo? De 5 a 31 de Março 27 dias
Se conta ndo os livros de 8 em 8, 10 em 10 e 15 em 15 De 1 a 30 de Abril 30
i.sempre sobram 5 é porque os livros que tenho, chamando-os De 1 a 31 de Maio 31
de N, são: De 1 a 30 de Junho 30

N = m. 8 +5 118
N m. 10 +5 ou
N = m. 15 +5 Considerando que o período é de 105
como fizemos de outras vêzes. 118 - 105 == 13

rm 8 l Então o próximo encontro dos e:Plermeiros será no dia


N = ~ m 10 }- +5 ou 30 de Junho - 13 = 17 de Junho.
l m 15 J
N = 120 + 5 = 125.

Considerando que são mais de 200 e menos de 300, EXERCiCIOS PARA RES.OLVER
o múltiplo 120, que é o menor, não serve.
Procurando o seguinte, isto é, 1) Determinar pelo processo da decomposição em fa-
tôres primos o m. m. c. dos números:
120 X 2 = 240
a) 28, 72 e 98; b) 60, 84 e 2'64; c) 36, 54 e 96.
Teremos:
Resposta: a) 3. 528; b) 9. 240 e c) 540.
240 +5= 245
que é o número de livros que possuo. 2) Determinar pelo processo de decomposição em
fatôres primos, em conjunto o m. m. c. dos números:
21) O s três enfermeiros de um pôsto médico fazem a) 30, 50 e 80; b) 125, 375 e 950; c) 18, 27, 40 e 35 .
plantão respectivamente de 15 em 15 dias; 35 em 35 dias
e 21 em 21 dias, mas ficaram juntos no dia 5 de março. Resposta: a) 1 . 200; b) 14. 250 e c) 7. 560.
Quando voltarão a ficar juntos novamente?
Pl'oblemas de Matemática para o Admissão 139
138 _ Paulo_ Pessoa --~- - --- -
~) 'rem-se A = 23 X 3m X S X 7 e Í3 = 2n X 32 X 1.
Calcular m e n de modo que o m. m. c seja 5. 040. 12) Determinar os três menores múltiplos comuns de
l4, 30 e 35.
Resposta: = 2 e n = 4.
m C . Pedro II - 1968
Resposta: 210, 420 e 630.
4) Sendo A = 23 X 3 2 X 53 X
7; B =
23 X 33 x 5 x 7
e C = 2 2 X 3'1 X 52 , determinar: o m. m. c.; o m.d.c. 13) Determinar os 4 menores múltiplos comuns de 9
e o quociente do m. m. c. pelo m . d. c. e 12.
Resposta: m. m. c. = 23 X 34 X 53 X 7; m . d. c. = Resposat: 36, 72, 108 e 144.
= 2 2 X 3 2 X 5 e o quociente 3.150.= 14) Determinar os dois menores múltiplos comuns aos
5) Qual a diferenca entre o m. m. c. e o m. d. c. dos números 36, 54 e 90.
lll.Úmeros 3 e 11? ,
Resposta: 540 e 1 . 080.
Resposta: 32. Ginasial - 1968 15) Calcular os três menores múltiplos comuns de 18
6) Determinar o m. m. c. dos números, com o auxílio e 27.
dom. d. c. Resposta: 54, 108 e 162.
a) 240 e 360; b) 520 e 956 15) Qual o menor número que dividido por 18, 24 e
Resposta: a) 720; b) 125 . 236. 48, deixa sempre resto 5.
7) Determinar o m. m. c. de dois números cujo pro~ Resposta: 149.
duto é 2 . 160 e m. d. c. = 6. 16) Determinar o menor número que dividido por 18,
45 e 63, deixa sempre resto 13.
Resposta: 360.
Resposta: 643.
8) Determinar o m. d. c. de dois números sabendo-se
que o produto dêles é 3 . 375 e o m. m. e. = 225. 17) Qual o menor número divisível ao mesmo tempo
por 18, 27 e 36.
Resposta: 15'.
Resposta: 108.
9) Determinar o m. m. c. de 2. 401 e 360, GQm Q a.uxí~
lio do m. d. c. 18) Qual o menor · número divisível ao mesmo tempo
por 18, 24, 30 e 40?
Resposta: 720.
Resposta: 360.
10) Determinar o m. m. c. dos número.s: 5.24 e 956., com
o auxilio do m. d. e. 19) Calcular os números de três algarismos que são
divisíveis por 36 e 40.
Resposta: 1.122.
Resposta: 360 e 720.
11) Determinar o m. d. e. de dois números: sabendo·
que o produto dêles é 5.040 e o seu m. m. e. '= 420. 20) Quais são os números inferiores a 1 . 400 que são
rlivisíveis por 8, 9 e 10?
Resposta: 12.
Resposta; 360, 720 e l. 080 .
140 Pa.ulo Pessoa
80) Âchcir o menor múltiplo de 1, que dividido pór S
e 9 deixa sempre resto 4.
21) Quais são os múltiplos de 15 compreendidos entre
29 e 59? Resposta: 49.

Resposta: '30 e 45. Liceu Nilo Peçanha - 1967 31) Achar o menor múltiplo de 60 divisível por 108.

22) Determinar os números compreendidos entre 2. 100 Resposta: 540.


e 4. 850 que sejam divisíveis por 84, 98 e 168. 32) Achar o menor múltiplo de 15 àivisível por 27.
Resposta: 2. 352, 3. 528 e 4. 704. Resposta: 135.
23) Quais são os números múltiplos de 84 e 96, meno- 33) O m. m . c. de dois números é 50. Qual é o
res que 2 . 000? m. m. c. de dois números que são respectivamente o quá-
Resposta: 672 e 1 . 344. Liceu Nilo Peçanha - 1968 pruplo de cada um dêsses?

24)Determinar os números compreendidos entre 1 . 500 Resposta: 200.


e 4. 600 múltiplos de 108 e 168. 34) O m. m. c. de três números é 150. Qual é o m.
m. c. de trés números que sao respec,1vame11te, um têrço de
Resposta: 1. 512, 3 . 024 e 4. 536.
cada um dêsses?
25) Calcule os três menores números pelos quais se
Resposta: 50
deve multiplicar 60, P.O e 120 respectivamente, de modo
que o.s produtos obtidos sejam iguais. 35) Qual a menor quantia que se obtém grupando
moedas de r~Cr~ O, W; NCr.i!i U, ~u e NCr$ O, 50?
Hesposta: 4, 3 e 2.
Resposta: NCr$ 1,00.
26) Calcule os três menores números pelos quais se
deve multiplicar 36, 54 e 90, respectivamente, de modo que 36) Um colecionador de moedas pretendeu separá-~s
os produtos obtidos sejam iguais. de b em b; 1:.:: em 1~ ou Hl em H>, 1uas sempre sobra am
'* moedas. Con tou-as tôdas e vermcou que elas eram mais
Resposta: 15, 10 e 6. que 1 HJ e menos que 180. Quaillas moedas tinha o cole-
27) Determinar o menor número ao qual faltam 9 uni- c1onador?
dades para ser ao mesmo tempo divisível por 18, 27 e 36. Resposta: 148.
Resposta: 99. 37) Numa granja existem mais de 1.600 aves e me-
28) Determinar o menor número ao qual faltam 36 uni- nos de 2. 4Uú contando-as de 64 em 64; 80 em 8U e 120 em
dades p a ra ser ao m esmo tempo divisível por 36, 40, 516 e l:t.U sempre sobram 40. Quantas a ves existem na granja'?
72. Resposta: 1 . 960.
Resposta: 2. 484. 38) De uma estação urbana partem ônibus para o bair-
29) Achar o menor múltiplo de 5 que dividido por 4 ro A de 18 em 18 minutos; para o bairro B de 12 em 12 minutos
ie 28 deixa sempre resto ~. e para o bairro C de 10 em 10 minutos. Sabendo-se que às
Resposta: 115.
Problemas d~ Matemática para o Admissão 143
lô hor<'.rs e 48 minulós partiram juntos ós ônibus dessas tr&s
linhas, a que horas partirão juntos novamente?
Resposta: 13 horas e 48 minutos.
39) Três automóveis disputam uma corrida em pista
circular. O primeiro dá cada volta em 4 minutos; o segundo
em 5 minutos e o terceiro em 6 minutos. No fim de quanto
tempo voltarão os três automóveis a se encontrar no início FRAÇÕES
)da pista, se êles partiram juntos?
Resposta: 60 minuos ou uma hora.
Fração é a medida de uma grandeza que contém uma
40) Três amigos vêm ao Rio respectivamente de 30 em ou mais partes iguais em que se dividiu a unidade.
30 dias; de 48 em 48 dias e de 60 em 60 dias.
É representada por um traço horizontal separando dois
Se êles chegaram ontem, há quantos dias estiveram
11úmeros que ficam um sôbre o traço e o outro sob êle.
reunidos no Rio pela última vez?
r:sses dois números são chamados têrmos da tração. Par-
Resposta: 240 dias. ticularizando, o que fica sôbre o traço é o numerador e in-
dica o número de partes tomadas à unidade. O que fica
41) Duas rodas de uma engrenagem têm 30 e 36 den-
sob o traço é o denominador e indica o número de partes
tes respectivamente. Cada roda têm um dente es~agado. Se
J(j uaís em que a unidade foi dividida.
num dado instante os dois dentes quebrados estao em con-
tato, pergunta-se: depois de quantas voltas de cada engre- 3
:r;i.agem se repetirá o encontro? Assim - - - é uma fração que se lê: três quintos. O
5
Resposta: 6 e 5 voltas. C. Militar - 1968 número 3 é o nume1·ador e o número 5 o denominador. O
denominador 5 indica o número de partes em que a uni-
42) Tem-se: A = 72; B = 120 e C = 900. Escreva:
dade (qumquer que ela seja) foi dividida.
a) O maior múltiplo comum entre A, B e C com-
preendido entre 1. 000 e 4. 000. O numerador 3 diz terem sido tomadas 3 daquelas 5
partes. As significações atribuídas aos têrmos de um fração
b) O menor número primo inferior a A. iuva:m a concluir que:
c) O número de divisores comuns a A, B e C.
l.~ - Se. duas frações têm o mesmo denominador,
Resposta: a) 3.600; b) 71; c) 6. C. Pedro II - 1968 ~quela: que possuir maior numerador é a maior.
3 4
Assim, se tivermos as frações - - e - - , a segunda:
5 5
i; rá maior do que a primeira, porque seu numerador 4 é
maior do que o numerador 3 da primeira.
2 ° - Se duas frações têm o mesmo numerador, aque-
la que tiver menor denominador será a maior.

144 ro,~lo reuol\ Problem~a de Mu~cmática para o Adtnh~ii.o 145


7 7
Por issõ se Hvermo.s as frações - - e a primeira
2 2
8 9
será maior do que a segunda pois o número de partes, Assim, o número mixto 1 ou 1 + será
3 3
tomadas à unidade, em ambas as partes foi 7, enquanto
transformado em fração imprópria de acôrdo com o que foi
que a mesma unidade foi na primeira dividida em 8 partes e
tlito, corno se segue:
na segunda em 9, sendo as primeiras, portanto maiores.
Depois do que acabou de ser dito ficamos em conw-
ções de comparar írações, desde que tenham os mesmos
2 2 1 X 3 + 2
denominadores ou numeradores.
1 +--
3 3 3
Quando a fração tem o numerador menor do que o
3 3+ 2 5
denominador ela é dita própr.ia, como por exemplo - -- , -~

7 ~ 3 3
- - - , etc. No caso de dar-se o contrário, isto é o dlenomina-
8
dor ser menor do que o numerador, a fração é dita impró- /\ lf!'u mas pr<J•p r:iedades das frações
8 7
pria como no caso das ±rações - - - e - - - . l.ª - Quando se multiplica o numerador de uma fracão
3 o por uma determinada quantidade·, a fração fica tantas vê~es
lJuando a fração é imprópna podemo.s ext:.air -lhe os maior 01'1'1nta~ forem as unidades da quantidade aue serviu
int1;;•,1u;;;, o que St: cunseyuc: Ul Vl LÜuUO o l1UH1c;1UUO! }'t:iO ele multiplicador . '
Ci.enornmactor.
s 2 2 2.ª - Quando se multinlica o denominador de uma fra-
Assim, - - - é igual a 1 + ou 1 - -- , re- cf'ro p or uma certa auantidade. a fracão fica tantas vê.ze:::
mer.or quantac:; fôrem as un idades da quantidade que serviu
3 3
15ultado que é conhecido como número mixto. O numero 1 e ~m~~&~ m. -
a parte imerra; ào1s o resto aa à1v1sao de ::i por ::i e ;;s o a1v1sor. Como com:eaüência: <la s duas propriedades anteriores,
A ode-s e dizer que: multio liccmdo-s9 os têrmos de uma fracão
Dai a fração - - , sempre própria (o resto não pode ser (numerador e denominador) por uma mesma quantidad~. a
3 fração não se altera. .
maior que o divisor), que se junta com a parte inteira, e
1 3.o - o.uando se div_ide o numerador de uma fração por
por isso, o nome de nuine10 m1x10. uma determmada quantidade, a fração fica tantas vêzes
Assim como ioi possível transformar uma ir.ação im- menor quantas forem as unidades da quantidade que serviu
própna em numero mixto, podemos trausrormar um numero de divisor.
mixro em iraçao impropria, para o que é bastame mulhpll-
car o inteiro peio aenommauor da ±ração e somar ao pro- 4.ª - Quando se divide o denominador de uma fra-
duto o numeraaor. Esse resul1aao é o numerador da tração ~·ão por uma determinada qµantidade, a fracão fica tantas
imprópria, 1que terá para denominador, o denominador da vêzes maior quantas forem ª!? unidades da quantidade que
nerviu de divisor.
fração.
Corno conseqüência das duas últimas propriedades,
,..onclw-se que uma fração não se altera quando se divide
146
os seus têrxnos (numerador e denominador) pela mesma
A segunda môstrou~rtos como podemos, certos casoi, em
lcterrrtinar frações equivalentes a uma fração dada, bas..1
quantidade. la ndo apenas que a fração dada, seja redutível.
Essa conseqüência permite fazer o que se chama: sim- A primeira nos permite calcular frações equivalentes à
plificação de uma fração, que sendo continuada até seus uma fração dada, seja ela redutível ou irredutível.
têrmos serem primos entre si, ela se tornará irredutível ou
reduzida à sua expressão mais simples. Vejamos como conseguir uma fração equivalente à hs.,
3
A simplificação de uma fração redutível (a que não tem ção rredutível - -
os seus têrmos primos entre si) pode ser feita por reducões 5
rmcessivas até que se torne uma fração irredutível. Pode
também ser feit~ com uma única simplificação, desde que Para isso é bastante multiplicar os seus têrmQs· por um
seus têrmos sejam divididos pelo m. d. e. dêles. mesmo número, 7 por exemplo, para termos:

27 3 3 X 7 21
Ass im, seja tornar irredutível a fracão redutíve.! ~'
5 5 X 7 35
por simplificações sucessivas.
De dois modos podemos então achar frações equivalen-
Teremos: tes à frações dadas:
0
27 27 3 9 9 3 3 l. - Simplificando-as, no caso de serem redutíveis ou mul-
- -- - ----- - - - - t.== - -' que tiplicando-lhes os têrmos por u.in mesmo número.
45 45 3 15 15 3 5 2. 0 - Multiplicando-lhes os têrmos por um determinado nú-
é irredutível. mero, no caso de serem as frações irredutíveis.
No caso de querermos obter a fração irredutível com
uma simplificação apenas, procuraríamos o m. d. c. de 27
EXERCíCIOS RESOLVIDOS
e 45 que sabemos ser 9 e teremos:
27 27 9 3 5 3 7
---
45 45 9 5
1) Somar as frações:
17
+ 17
+ 17
27 9 3 As frações cuja soma vamos fazer são homogêneas, pois
As frações - -' - - e '
são equivalentes, isto e, lêm o mesmo denominador.
45 15 5
têm o mesmo valor. As duas primeiras são redutíveis e a Em tais casos somam-se os numeradores e conserva-se o
denominador.
última irredutível.
Assim:
As duas conseqüências enumeradas anteriormente po·
dem ser sintetizadas em uma só e que é a seguinte: 5 3 7 5 +3+7 15
"O valor de uma fração não se altera quando se mul- 17 + 17i
+ 17 17 17
tiplicam ou dividem os Reus têrmos por un1 mesmo núme;r'!"· 1U l11J; j ,lJ

Problemas ele Matemática pa.ra o Admissão } 49.1


5 3 9
2) Somar d.s frações:
7
+ 7
+ 7
Considerando que as frações passaram a ter o denomi-
nador 36, concluímos que os denominadores das frações
<lados, foram multiplicados respectivamente por 4, 9 e 18,
Como da vez anterior: razão pela qual teremos que multiplicar os respectivos nu-
5 3 9 meradores pelas mesmas quantidades para que não se alte-

7
+ -7 + 7
1 em, pois se assim não fôsse, como vimos anteriormente, as
frações dadas ficariam divididas por 4, 9 e 18, deixando de
17 3 ser as dadas para somar.
~
---
7
2+ 7 Então teremos:
depois de extrair os inteiros. 4 X 4 3 X 9 1 X 18
3) Somar:
---+ 4 X 9
+ -2 -X -18
9 X 4
1 3
3 - +s- 16 27 18
-4 4 - 36- + --+
36
- - -, equivalentes às
36
fra ções dadas e que somadas dão:
Trata-se de somar doís números mixtos Devemos trans-
formá-los primeiramente em frações impróprias e depois 16 + 27 +28 61 25
somá-las, como nos exémplos anteriores. 1-,
36 36 36
De acôrdo com o que foi ensinado depois de extrair os inteiros.
1 13 3 23
3--=-- e 5--= 5) Efetuar as operações:
4 4 4 4 5 5 1
Então iremos somar: 12 - - + 4 - - + 3 - -_
6 9 2
13 23 36
- - + - - ,= --= 9 De início transformaremos os números mixtos em fra-
4 4 4 ções impróprias.
4 3 1 Vemos:
4) Somar as frações: - - + -- + --.
9 4 2 77 41 7
-6+ 9
+ 2
Como os denominadores das frações são diferentes, é
~ preciso reduzí-Jos ao mesmo
denominador, que será o m. m.
e. de 9, 4 e 2, isto é 36. A seguir reduziremos as frações ao mesmo denomina-
dor que é o m. m. c. de 6, 9 e 2, igual a 18.
.}50 1
Paulo .Pessoa
Problemas de Mate má tica para " .l\dmissão 151
A seguir procederemos como no e xemplo anterior e 8) Subtrair:
vem:
2
77 X 3 41 X 2 7 X 9 7-
18
+ 18 + 18
ou 3

23 1 + 82 + 63 376 16 Se em lugar do sinal menos entre o inteiro 7 e a fração


20 - - - 2
18 18 18 - - -, estiver o sinal mais, já vimos como proceder .
3
8 16
20 - - , depois de simplificar a fração
A diferença entre o que fizemos quando ensinamos a
9 18
transformar um número mixto em fração imprópria e a sub-
8
torna ndo-a na irredutível - - . tração de uma fração de um inteiro, está em que: do pro-
duto do inteiro pelo denominador da fração, subtrai-se o
9
numerador do mesmo, conservando-se, entretanto o, resul-
10 9 tado achado sôbre o denominador da fração:
6) Subtrair as frações:
li 11
Assim:
As frações sendo homogêneas como as do exemplo l,
2 7 X 3-2 19
é b a stante sub trair os numeradore s e conservar o de"nomi- 7 - --
n ador . 3 3 3
1
Assim: ô
3
10 9 10 1 1
------- -
9 4
11 11 11 11
9) Subtrair:
2 4 11 5
7) Subtrair de 8 - - o número mixto 5 - -.
9 9
Como os denominadores são diferentes procederemos
Transformando os números mixtos a subtrair, em frações como no caso da soma de frações com denominadores dife-
impróprias, vem: rentes, como foi feito no exemplo 4.

74 49 25 7 O m. m . e. das frações equivalentes a subtrair, por se-


rem denominadores das fra·ç oes dadas, números primos en-
9 9 9 9 tre si, é: m m. e. =
11 X 5 55 . =
152 Pirnlo Pessoa i>roblem1ts de Matemática pa,·a o Admissão J.53
'haia-se de subtraír de uma fráçãô imprcSpiià, uni n~­
Então: mero inteiro. Em tais casos, multiplica-se o inteiro pelo de-·
n ominador da fração; subtrai-se o resultado encontrado
9 X 5 4 X 11 do numerador da mesmo e o resultado será o numerador da
ou tração que terá o mesmo denominador.
11 X 5 5 X 11
Assim:
45 44 45 - 44 1
------- 12 12 - l X 7
55 55 -55 55 ---1
7 7
10) Efetuar a subtração: 12 - 7 5
2 3 7 7
11--4-
3 7 12) Multiplicar as frações~

Transformando os números mixtos em frações impró- 2 3 6


prias, vem: X X
5 4 7
·35 31
Para multiplicarmos várias frações, multiplicam-se os
3 7 numeradores, cujo produto será o numerador da fração re.-
sultante.
Reduzindo ao mesmo denominador depois de achar o
m. m. e. que é 21, vem: Multiplicam-se os denominadores das frações dadas e o
produto será o denominador da fração resultante.
35 X 7 31 X 3
ou Assim:
21 21
2 3 6 2 X 3 X 6 36
245 93 245 - 93 152
X X --,
5 4 7 5 X 4 X 7 140
21 21 21 21
5 9
7- que depois de tornada irredutível passará a ser: - - .
21 35.
13) Multiplicar:
11) Efetuar a subtração:
3
12 9 X
--1 4
7
ProblemM de l\latemátlca pa.ra o Admissão 155
!>ara multiplicarmos um inteiro por uma iiaçiío oú urr'1 a
fração por um inteiro, multiplica-se o inteiro pelo numera-
dor da fração e conserva-se, no resultado, o mesmo deno- Para dividir frações conservamos a fração dividendo;
:r:ninador. trocamos o sinal de dividir pelo de multiplicar; invertemos
os têrmos da fração divisora e procedemos como na m ulti-
Assim: plicação.
3 9 X 3 27 3
9 X 6 Então:
4 4 4 4
4 8
14) Efetuar as operações: (fração dividendo) (fração divisora) é o
9 11
' 1 a,j 4 8 4 1 mesmo que:
)
, .1 1 (
.
úi
1
-18+ -
10
x- 6
X
l•
2 4 11 44
1 . J ll
11
X - -- = -- - = . - - -
bevernos efetuar primeiramente as operações que estão 9 8 72 18
cbntidas no parênteses efetuando em primeiro lugar a mul-
tiplicação e a seguir a soma . 16) Efetuar:

Assim: 3 5 8
8 X 4 1
(-4
18
+
10 X 6
) X
2
ou 4 7 9

De acôrdo com o que foi dito acima, aplicado às duas


4 32 1
( -+ 18 60
) X
2
ou
primeiras frações, vem:

3 5 3 7 21
96 1 -'- --- X --
( 40
180 + 180 )x 2
ou: 7 4 5 20

Temos assim o quociente das duas primeiras frações


136 l 136 17 que poderá substituí-las para ser dividido por terceira fra~ãg.
180 2 360
x-- 45 Teremos então:
depois de simplificar o resultado .
15) Dividir: 21 8
e novam~nte
4 8 20 9
- - por - -
21 9 189
9 11
~ x
20 8 160
156 P m:i!o Pessoa
Ássini:
Depois do que foi mostrado poderíamos proecder como 32
se segue: 8 é o mesmo quê
63
3 5 8
é o mesmo que 63 63 3
4 7 9
8 X 15-
32 4 8
3 7 9 189 29
________, X - - - X - -
4 5 8
= -160- ·= 1 -160- (depois de simplificar e extrair os inteiros) .

17) Dividir: 19) Efetuar as operações:

9
2 1 4
11
4
(+ + 5 ) X
23 +- 5-- -

Para dividir uma ração por um inteiro, conserav-se a 7 7


fração; troca-se o sinal de dividir para multiplicar e inver- ----
te-se o inteiro. 12 9

Teremos então:
Para resolvermos ~ma expressão dêsse tipo torna-se
9 necessário ter atenção a ordem das operações
4 é o mesmo que
11
Assim é que:
9 1 9
X 3 2
4 44
H A soma - - - + dev~ ser multiplicada por
4 5
18) Dividir
J '
1
32 e teremos:
23
ae 6-3
3 2 15 +8 23
Para dividir um inteiro por uma fração, conserva-se o + ----
4 5 20 20
inteiro; troca-se o sinal de dividir pelo de multiplicar, inver~
te-se a fração e proce<;i~-se çQmQ PQ exe;rcí.cio 13 ; ·
l'ro)>lemais de MaLemli.tlca pa.1·a o Admloio 159
1 20) Resolver:
f:sse resultado multiplicado p or - -, dá:
23 3

5
+2 5 23
23 1 1
X - - que deve ser somado à fração X X
3 2 3 8 10
20 23 20
4 + -- X
4 3 5
- - e teremos:
5 3 13
O numerador - - + 2 é igual a - -
1 4 1 + 16 17 5 5
+ 5- 3 2 3
20 20 20
O denominador -- + X é igu.:rl a
4 3 5
17 3 2X 3 3 2 15 +8 23
Dêsse resultado - - deve ser subtraído, não a fração - + - - =-+ - =- -= -
20 4 3X 5 4 5 20 20
7 7
- - , mas sim o quociente dela por - - . Então teremos:
12 9 13
---
5 5 23
Teremos então: X X
23 8 10
---
7 7 7 9 3 20
~--
12
+ - - -
9
= ---
12
X ~--
7
- - --
4 A fração cujos numeradore~ e denominadores são fra-
ções, significa, como tôda fração, que o numerador é para
(depois de simplificada) que deverá ser subtraído da fmção ser dividido pelo denomina dor; p oderemos resolvê-la:
17
13 23 13 20 52
20 ~~ + ~~ = ~~ X ~~ = ~~
5 20 5 23 23
Teremos finalmente: (depois de simplificado) e então teremos:
52 5 23 1
17 3 17 - 15 2 1 V
X 3 - - depois de feitas
23 8 10 4
20 4 20 10 s as operações.

160 Paulo l'e111011 Prob!emll;s <le Matemática pa.r a o Admissão 161


21) Escrever em ordem decrbscente as seguintes fra-
5 7 3 3 2
ções: - - , - - e - - - 22) Escrever em ordem crescente as frações - -, - -
5 3 4 6 3
3
Para comparar frações, dissemos no início do capítulo, é o--
necessário que elas tenham os mesmos denominado:es ou 8
numeradores. Problema idêntico ao anterior mas que iremos resolver
tornando iguais os numeradores das frações dadas . Para
Reduzindo as frações dadas ao mesmo denominador
Isso é bastante reduzi-las ao mesmo numerador, procuran-
vem: do o m. m . c. de seus numeradores 3, 2 e 3, que é 6.
5 7 3
- -, - - e - - Entã o teremos:
5 3 4
6 6 6
e ou
O m. m. c. de 3, 4 e 5 é 60, por serem números primos
2 X 6 3 X 3 2 X 8
entre si. Então depois do que já foi ensinado:
6 6 6
12 X 5 20 X 7 15 X 3
ou 12 9 16
60 60 60
Q uando os numeradores são iguai~, a que tem menor
60 140 45 denominador é a maior e a que tem maior denominador
---- e é a m enor.
60 60 60
Postos em ordem crescente será:
A que tem o maior numerador é a maior e a q ue tem
menor numerador é a menor. 6 6 6
Então para colocá-las em ordem decrescente e screve- - --. e - - - ou
16 12 9
remos:
140 60 45 rnpor:ando-nos às frações dadas:
e - - - -
3 3 2
60 60 60 e -- -
que comparados com as equivalentes que foram dadas 8 6 3
seriam escritas
23) Reduza os n úmeros 27, 42, 37, 81 , 103 a quintos .
7 5 3
- -- , - -- e Para reduzi os números dados a quintos é preciso que
3 5 4 fios sejam a presentados sob forma de frações com deno-
minadores 5.
162 Paulo Pessoa
P1 obl emas de }(a temática p:.ra o Adm is são 163
1 3 7
Colocar os números dados sôbre denominadores 5 é o 1 ---.
mesmo que dividí-los por 5 . Para que êles continuem sendo 3 7 3
o que são torna-se, portanto, necessário mubplicá-los por 7
5 e colocá-los sôbre 5.
1 8 9
Então teremos: ---;
27 X 5 135 8 9 8

9
5 5
1 1
42 X 5 210 1 3;
5 5
3
37 X 5 185
1 5 7
5 5 1
5 7 5
81 X 5 405
7
5 5
25) Complete a igualdade:
103 X 5 515 .,.•..
---- 20=--
5 5 8
15 3 8 1 5 Trata-se de procurar um número que dividido por 8, dê
24) Escreva o inverso de - - , ~-, --, - - e - - para quociente 20.
4 7 9 3 7
Temos portanto o divisor (8) e o quociente (20).
O inverso de um número é obtido colocando-o no de-
nominador de uma fração cujo numerador é a unidade. Sabemos que o dividendo (*) é o produto do divisor
pelo quociente.
Então os inversos dos números dados são obtidos:
1 15 4 Então:
---,
15 4 15 * (dividendo) = 8 X 20 = 160.
4
ProblemaoS de Matemática para o Admissão 165
26) Substitua o a sterístico para que a s fra ções sejam
equiva lentes: Com problemas dêsse gênero, a preposição de é subs-
p elo sinal de multiplicação e por isso teremos:
l 1tuída
2 10 3 3 X 25
X 25 = - - - 15.
3 5 5
Vimos como é possível obter fra ções equiva lentes a uma 29) Calcular:
fração dada.
2 4 2
Considerando que - - é uma fração irredutível e que - - - dos ~-- de 15.
3 • 5 3
-:i: equivalente que se pretende comple ~ar tem por numera-
dor 10, conclui-se que o numerador da fração dad~ (2) foi Do mesmo modo que no ·e xemplo anterior, teremos:
multiplicado por 5 (10 -:-- 2 =
5). J;>ara que a fração não se 4 2
altere o denominador (9) da fração dada deve ser também 8
X X 15
multiplicado por 5 e o asterístico (* ) será então: 5 3
3 X 5 = 15. 3 7
30) Determinar duas frações equivalentes a - - e - -
A fração procurada será: 5 8
10 de modo que o numerador da primeíra seja igual ao deno-
minador da segunda.
15 De duas maneiras podemos achar uma fração equiva-
le nte à outra, como já vimos.
27) O problema 25 pode ser enunciado: Escreva uma No caso em questão, por serem as frações irredutíveis,
fra5ão que seja igual a 20 e que tenha denominador 8. neus equivalentes só poderão ser obtidos multiplicando-se
.sous têrmos por quantidades iguais, respectivamente . Te-
Mudou apenas o enunciado. A maneira de resolver e 3
as razões apresentadas naquele problemo continuam as temos então que multiplicàr os têrrnos da fração por
mesmas e por isso a fração será:
7
160 u m número e os dos - - por outro.
8
8 Por isso o numerador da primeira fração p rocurada
1wrá múltip lo de 3 e o denominador da segunda, múltiplo
3 cJ 8.
28) Calcule - - d e 25. Como d evem ser iguais, por exigências do problema,
s g ue-se que êles têm que ser múltiplos de 3 e 8 . Vimos q ue

166 Paulo Pess oa


Problemas de Matemática par a o Admissão 167
.e xistem process os para determinar o menor múltiplo comum mo que muHiplicar os têrmos da fração por 4, para obter-
a vários números e no caso em questão como são p ri- mos:
?'110S êle será igual a 3 X 8, então é 24
12
Temos então:
20
24 36
32) Escreva uma fração equivalente a - - , cuja dife-
1. ª fração - - -
* rença dos têrmos seja 15.

2.ª fra ção * Procedendo-se como no exemplo anterior, vê··se que a


24 2
fração dada é equivalente a - --
Uma vez que o numerador da primeira fr a ção dada era 3
3 e passou a ser 24, conclui-se que tal só ocorreu porque
foi multip licado por 8. Para que a fração não se altere A diferença de seus têrmos é 3 - 2 = 1 . Como quere-
teremos q u e multiplicar seu denominador também p or 8 mos que a diferença seja 15, basta multiplicar os têrmos
e virá: ela diferença (3 e 2) por 15 para termos o valor desejado
24 e a fração
l.ª fraçã o 30
40
45
Raciocinando-se do mesmo modo conclui-se que a se-
45
gunda fração será :
3:3) Achar uma fração igual a cujo denomina-
21 117
24 dor seja 39.
36 45 5
31) Determinar uma fração equivalente a - - , cuja A fração - - - t.Jrnada irredutível é: - -- .
,60 117 13
soma dos têrmos seja 32.
Como o problema pede que o denominador de: frllção
Sempcre que tivermos uma fra ção devemos simplificá-la, procurada seja 39, trata-se de achar um número que mul-
~sto é, torná-la irredutível. Assim sendo, a fração dada se tiplicado por 13 dê 39 e em seguida multiplicar o numera-
3 dor 5 pelo mesmo número, para que a fração não s e altere.
tornará igual a ~-. cujos têrmo.s somados dão 8.
5 O número procurado será pois 39 7 13 = 3 e a fração
15
Como o problema qu er que a soma dêles seja 32, é bas-
pedida - - - .
tante multiplicar a soma 32 7 8 ou seja 4, que é o mes-
39

163 Pa,u lo Pessoa P r obl emas de Ma t emá tica pa.ra o Adm issão 169
34) Determinar o número que se deve tirar do deno- Teremos :
39
minador da fração - - - para tomá-la seis vêzes maior. 65 65 1 5
42 - - - 13 = - - X
73 73 13 73
Para tomarmos um número seis vêzes maior, é bas'an-
39 65
te multiplicá-lo por 6 . Então: para tornarmos a fração - - - Comparando a fração dada - - - e o seu quociente por
42 73
seis vêzes maior, devemos multiplicá-la por 6 e teremos 5
39 13, - -- verificamos fàcilmente que entre as duas existe
- -- X 6 73
42 apenas, uma diferença de 65 - 5, isto é, 60 entre os se:1s
Para que a solução se torne evidente em lugar de mul- numeradores, sendo portanto 60 a resposta pedida.
tiplicarmos 39 por 6 faremos a simplificação do multiplica-
dor 6 com o denominador 42 e acharemos a fração: 36) Três quartos do metro de uma fazenda custa NCr$
36,00. Quanto custará um metro?
39 3
Podemos dizer que a fração - - · -, é igual, isto é. cor-
7 4
39 responde a NCr$ 36.00.
Considerando que a fração dada era e que a
42 Escrev·e remos então:
39
fração seis vêzes maior é - - - , constatamos que entre as 3 corresponde
7 -----? NCr 36,00
duas existe apenas uma diferença entre os denominadores 4
de:
3 1 corresponde
42 - 7 = 35.
Foi bastante tirar 35 do denominador da fração dada 4
-
3 -
4
---~--·-? NCr$ 36,00 + · 3

para que ela se tornasse seis vêzes maior. NCr$ 12,00


35) Calcular o número que se deve tirar do numera- 1 4 corresponde
- - X 4 = (um metro) -7 NCr$
65
dor da fração - - - para torná-la 13 vêzes menor. 4 4
73 12, 00 X 4 = NCr$ 48, 00.
2
Para tornarmos um número 13 vêzes menor é bastante 37) Com 12 litros de vinho quantas garrafas de - - -
·dividí-lo por 13, razão pela qual devemos dividir por 13 a 3
fração dada. do li1ro p oderão ser cheia s?

170 P acilo Pessoa P1·oblemail de Matemática pa.ra o Admissão 171


2
Se cada garrafa contém - - - de _litro, é sinal de que
3 1 corresponde
2 ------~ 8.638
12 litros -;- - - - dará o número de garrafas. 8
3
3 corresponde
Teremos: -- -------~ 25 -914
2 3 8
12 -;- - - = 12 X - - : : = 18
3 2
39) As laranjas colhidas em um pomar foram deposi·
garrafas. 5
5 !adas em duas cestas. Os - - - das laranjas são iguais a
38) Para ladrilhar - - - de um pátio empregaram-se
7 7
3 15 e foram colocados em uma das cestas.
49. 360 ladrilhos. Para ladrilhar do m~smo pá tio,
8 3
quantos ladrilhos serão necessários? Os - - - do total de laranjas correspondem a 6 e fo-
13
ram depositados na outra.
Como no exemplo 36, podemos escrever:
5 corresponde Quanto uma tem mais que a outra?
-7 49.360
7 Na l.ª cesta:

1 corresponde 5
~ 9 . 872 ~ 15
7 7
7 corresponde 1
- -- ,, 69 . 104
7 7
7 8 7
Como - - - - - - podemos escrever: -> 21 laranjas.
7 8 7

8 corresponde Na 2.ª cesta:


~ 69 -104
8 3

172 Pa.ulo Pessoa 13

Problemas de Matemática pa.ra o Admissão 173


5 corresponde
soo
5
13
12
13 1. 200.
--') 26 laranjas. 5
13
Os números são 500 e l. 200.
Diferença:
26 - 21 = 5 laranjas. 41) Sublraindo-se 8 da metade de um número resta
12 1
40) A soma de dois números é 1. 700. Um vale - - - - - - do número. Qual é o seu número?
5 3
do outro. A diferença entre a metade e a têrça parte do número
é 8. Então:
Quais são êles?
5 1 1 3- 2 1
Podemos estabelecer que um número é - - -
5 2 6 6
12
Finalmente
O outro sendo - - - dêle, será:
5 1 corresponde
--') 8
12 5 12 6
X ·---== - - -
5 5 5 6 corresponde
o número - - - --') 48
Os dois somado~ 6

5 12 corresponde 4
42) Uma pessoa gastou - - - - de seus haveres e ficou
5
+ 5
- - - - -- --'.> 1. 700
7

17 corresponde 1
----------') 1. 700 tom - - - dêles mais NCr$ 16,40. Quanto tinha? +
5 3
7
1 corresponde De início a pessoa tinha - - - .
--- 100 7
5

J74 Paulo Pesso&


Prob!em&s de Matemática para o Admissão 175
4 2
Tendo gasto ---.ficou com 7: Se - - - é constituído de meninas é evidente que os
3
7
meninos são:
7 4 3
--- - - -- 3 2 1
7 7 7 - - - - - - - = - - - e como o seu número é 215,
3 3 3
3 teremos:
Êsses - - - , de acôrdo com o enunciado do problema,
7 1
1 ) 215
é o que sobrou dos gastos, isto é - - - + NCr$ 16.40 3
3 3
3 1
Se de (total da sobra), subtrairmos - - - , a "' 1645
3
7 3
fração resultante conesponderá aos NCr$ 1€,40. 4
44) Uma pessoa gasta - - - do ordenado mais NCr$
Então:
5
3 1 9 - 7 2 110,00 Se economizar NCr$ 580,00, quanto percebe?

7 3 21 21 4
Se só gastasse - - - , sua economia seria NCr$
Teremos então: 5
110, 00 + NCr$ 580, OC = NCr$ 720, 00 que corresponderia a
2
- - - -- - - - - + NCr$ 16,40 5 4 l
21
5 5 5
1
- ----+ NCr$ 8,20 Então:
21
21 - - - - - - - - -> 720.00 e
- ----+ NCr$ 172,20. 5
21 5
2
-+ 3 . 600,00
43) Em um ginásio - - - dos alunos são meninas.
5
3
O número de meninos é 215. Qual o total de estudantes?
45) Três operários fazem um trabalho em 4 dias; o
Ginasial - 1969 p11meiro e o segundo se ndo capazes de fazê-lo sozinhos em

176 Paulo Pessoa Problemas d~ Matemática pa;ra o Admissão 177


- -------:--~-------------------,...---~~====-:~=========--~~

9 e 12 dias, respectivamente, pergunta-se o número de dias


que o terceiro levará sozinho, para executar o trabalho . 3
46) Uma pessoa gasta do seu ordenado com
Se os três operários trabalhando em conjunto concluem 5
1 1
o serviço em 4 dias, é porque realizam por dia do despesas de casa e - - - do resto com despesas pessoais.
4 4
trabalho. Sua economia mensal é de NCr$ 30.00 Qual o ordenado?
Do mesmo modo conclui-se que os trabalhos diários do 3 5 .3
1 1 Gastando - - - do ordenado, sobram - - - --
primeiro e segundo são representados por - - - e - --, 5 5 .5
9 12 2
respectivamente.
5
Então o primeiro e o segundo juntos, realizam por dia: 2 1
Dêsses - - - gasta - - - , isto é,
1 1 7 5 4

9
+ --- -
12 36
1 2 1
- - - com despesas pessoais.
1
X
4 5 10
o trabalho diário doo três operários sendo - - - ,
4 Então gastou ao todo:
como vimos, do serviço, segue-se que o trabalho diário do 3 1 6 + 1 7
terceiro será: --+ -~ = - - -
5 10 10 10
1 7 1
5
4 36 18 Dissemos que seu ordenado era mas também
5
Teremos então: 10
podia ser - - -
1 Do trabalho 10.
-----~ em 1 dia 7
18 Então depois dos gastos de - - - , sobraram:
10
18 (trabalho todo)
- -- ------~ em 18 dias. 10 7 3
18 ---- --- I que orcresponde à
10 10 10
173 Paulo Pessoa
parte economizada. i
Nessas condições as duas primeiras teriam recebido:
2 1 13
--- + --- = --- da quantia total, ca-
Então: 3 5 15
bendo à terceira, que recebeu o restante:
3 corresponde 15 13 2
---------? NCr$ 30, 00 da quantia distri-
10· 15 15 15
buída, correspondente a NCr$ 32, 00 .
1 corresponde Teríamos então:
- - - -- ----? NCr$ 10. 00
2
10 - -- - - - - - - ? NCr$ 32, 00
15

corresponde 1
10 ;. NCr$ 16, 00
---? NCr$ 100, 00 15
10
15
15
- - - ---·--? NCr$ 240, 00
Certa quantia foi repartida entre três pessoas. 1\
47)
2 Sendo a quantia total NCr$ 240,00 e a distribuição tendo
primeira recebeu - - - da quantia mais NCr$ 5,00. A se- sido feita de acôrdo com o enunciado do problema, as im-
3 p ortâncias distribuídas foram :
1 2
gunda, - - - mais NCr$ 12,00. l.ª pessoa: - -
3
X NCr$ 240, 00 + NCr$ 5, 00
5
= NCr$ 165, 00.
Tendo a terceira recebido o restante no valor de NCrS 1
15,00, quanto recebeu cada pessoa? 2.ª pessoa: - - X NCr$ 240,00 + NCr$ 12,00
11 1111 ! ' 5
Se a primeira pessoa não houvesse recebido os NCr$
= NCr$ 60, 00.
2 3.ª pessoa: Como disse o problema NCr$ 15.00.
5, 00 além dos - - - da quantia e a segunda não ganhasse
3 48) Dividir 480 laranjas por três pessoas, de tal sorte
1 que as partes da primeira e da segunda sejam respectiva-
NCr$ 12, 00 além do - - - que recebeu, naturalmente que 1 4
5 mente - - - e - - - do que recebeu a terceira.
3 5
a terceira pessoa receberia:

NCr$ 15, 00 + NCr$ 5, 00 + NCr$ 12, 00 = NCr$ 32, 00 Problem,.s de Matemática para o Admissão 131
Podemos escrever: E como das outras vêzes:

1
32
Parte da l.ª - - - -- da 3.ª ~ 480
3 15

4 l
~ 15
Pc.ute da 2.ª - - - - - da 3.ª 15
5
15
Se conhecessemos a parte da 3.ª teríamos a fraçã o da ~ 225 laranjas .
primeira e segunda partes . 15

Esta beleçamos um valor para a parte da terce~IO, que 15 3


Considerando que - - - = ---, segue-se que a
3 15 3
poderá ser - - - terceira recebeu 225 laranjas.
3
Então as duas outras receberam respectivamente :
Então:
l 3 1
l
Parte da l.ª - - - - - X --- - l.ª - - - -- X 225 = 75 laranjas
3 3 3
3
4 3 4
Parte da 2.ª - - - -- X --- = - 4
5 3 5 2.ª - - - - - X 225 = 180 laranjas.
5
3
Parte da 3.ª - - 49) Duas pessoas têm juntas NCr$ 246,00. Uma delas
3 2 3.
g astou - - - e a outra - - - do que possuíam e ficaram
A soma delas deve dar o número total das laranjas d;_s- 5 7
tribuídas, isto é, 480 com q uantias iguais. Quanto possuía cada uma?

Então: 2 3
A primeira tendo gasto fico u com e a
l 4 3 5 + 12 + 15 32
5 5
3
+--+
4 3 15 15 3 4
$egunda gastando - - - ficou com - - -
7 7
182 Paulo Pessoa

P roblemas ele Matemática para o Admissão 183


3 4 EXERCíCIOS PARA RESOLVER
São essas duas frações - - - e - - - que se corres-
5 7 GSomm as t"oçõe"
pondem, isto é, que valem a mesma coisa, qua ndo toma-
dos da q uantia de cada um. 8 4 l
Então:
3 corresponde 4
a)
15
+ 15
+ 15 ---.-:--
--7 - - -
5 7
4 7 8
corresponde 4 4 b) + + --
--7 3 9 9 9
5 7 21
5 corresponde 4 20 10 11 2 3
-~ - - - /' s '-/ c) --+ + _)__

5 21 .21 13 13 13 :i. 3

5 21 13 1
Considerando q ue - - - é igual a - - - podemos di- Resposta: a) - - ; b) 2 - -; c) 2
5 21 15 9
zer que as importâncias de ambas expressas em frações
21 20
são - - - e - - - e que somadas correspondem à impor-
21 21
@somar
tância que ambas possuem juntas. 1 5
a) 8 - - + 9 - -
Então: 6 6
21 20 41
- - + - - = - - - ---7 NCr$ 246,00 4 3 2
21 21 21 b) 3 -- + 7 -- + 1
5 5 5
1
- -- - - -- - - ---? NCr$ 6, OD
21 2 1 3
21 c) +3--+ 5- -
7 7 7
--7 NCr$ 126, 00
21
4 6
20 Resposta: a) 18; b ) 12 - - ; c) 8 - -
--7 NCr$ 120, OG 5 7
21

184 Pau lo Pessoa


Problcmlts de Matemática para o Admissão 185
5) Subtrair:
Efetuar as operações:
3 6
2 1 1 a) 23 - --15--
a ) -- +--+--- 7 7
7 2 3
5 5
3 3 8 b) 9 --- 4--- 3
b) - - + - - + - - 6 6
8 14 21 2 3
e) 4 --- 1 --
1 2 3 5 5
e) 14 - - + 2 -- + 6 --
7 3 5 1 1
d) 5 - - - 3 --
2 3 3 7 3 2
d) 5 + - +-+ 8 + 1 -+ 4 -
3 4 5 9 4 4 s
Resposta: a) 7 ; b ) 2; e) 2 - - ; d) 1 - - -
5 163 43 143
Resposta: a ) 1 - - ; b) - -; e) 23 - - ; d) 20 -- -
7 5 6
42 168 105 180
~ Subtrair:
0 Subtrair:

7 3
a) 4 ~
1
- ·--· -
2
.

a)-- - - - 2
9 9 b) 5 - - ·- . -.
s
15 9 1
b) - - - -- - - - 3
4 4 4 e) 7---
4
8 7 1 4
e) - - - - - - - - d) 8 - --
15 15 15 5
4 1 1 3 1 1
:Resp osta: a ) - - ; b ) 1 - -; e) zero. Resposta: a) 3 - - ; b) 4 - - ; e) 6 - - ; d ) 7 - -
9 4 2 5 4 5

] 86 Pau]o Pess oa Problemas de Matemática para o Admissão 187


G Efetum as sub1'ações'
Efetuar as multiplicações:
3 3
a) 9 - - - 5-2-- 4 1 3
7 8 a) 15 - - X 2 - - X 1 --
9 4 5
1 7
~ -b) 9 - - - 3 - 1 -- 8 1 3 1
9 8 b) X -- X - - X ---
9 2 4 8
3 1
e) 8 + -- - 6--- 10 2 6
4 4 e) - - X - -- - X - -
3 5 4
5 3 8
,.,,,-- d) 4 -- - - - X 3 1
7 14 9 Resposta: a) 55 - - ; b) - - ; e) 2

3 17
Colégio Pedro II -

1
1968

11
e 5

4
24
Efetuar as operações:
3
Resposta: a ) 2 - - ; b) 4 - - ,. e) 2 - ----; d ) 4 -
21 - a) 5 X - - X 2--
56 22 2

8
15 16

Sub trair:

29
~ b) 18 X (

8
+ 9
+ - :- + +-)
4
a) -- - 5
e) -- X - - X 15 - -
4
5 4 9
18
11 1 3
b) - - - 2
Resposta: a) 2 - - ; b) 37 - -; e) 55 - -
7 12 2 5

e) -
15
-
8
- 1 @ Efetuar as oPerações:
2 4 5 l
-a) ( -3- + -5- --6- X -8- ) X
1 4 7
Resposta: a ) 2 - -- ; b) - --,. e)· - -
1 1
4 7 8 1 -+
5 8
188 P a ulo P~ssoa
Proble m as de Ma t emática para o Admissão 189
14) Efetuar:
/ 5
b) ' 4- - 8
\ 7 a) ----;---2
9
19 11
13

--
Resposta: a) 1 - - ; b) - -
~ 25 ° 63 b) - - + 5
7
~ Dividir:

21 7 3
e) 4-- + 5
a) -- - --
4
23 17
4 5
10 20 - d) 2-- + l
b) - - - --
5 6
13 31
11
3 6 x e) 9 - - + 12
e) -- - -- 13
5 7
5
5 5 7 X f) 21--;-- - -
Resposta: a) 2 - - ; b) 1 - - ; e) - - 8
23 26 l(l

G Efetumª' divi•Õe'! . g) --
14

23
+ 7

·a) ( _l6 +-1


8
+-1 )
10
(-1 +-1 +-1 )
5 8 3
10 + - -
3

5
- b) ( :2 + ;5 + :a ) ( ;; - ;~ ) 6
.l( i) 6 + --
e) 7_2_--;--- (-3-+ _2_ + _1_) 7
11 4 5 6 2
2 / l ' 1 ) / ( j) 21 + 5-
-d) 3 ----;--- \_ 2 - - X 1-- 3
5 3 5
4 1
47 6 5 21 k) 6 -- + 2 --
Resposta: a) - ; b) 8 - ; e) 5 - ; d) - 7 2
79 7 11 25
Probl e m a·s de Matemáti ca para o Admis ão 191.
190 Pu.ulo Pessoa
h) ) ·'+ 1 _ l_ - í( 5 + _!_
2 3 L s
3 - 8- 2 + (1
7) X -5- -2- + 1) -4- 1
-;- 1 -2- J
1 5 8 3 9
i) -+-x----;--
5 4 3 4 2

Liceu Nilo Peçanha - 1962


17 4
Resposta : a ) O; b ) l ; e) 4 - - ; d) 4 - ; e) 5;
20 9

7 17 11
b) ( _1_ _ _l _ ) -;- _s_ + _ 2_ X 1 _ l_ f) - ; g) 2 - - h) O; i) 3 -
2 3 6 3 5 72 20 30

G Efetuac
e)
3 _1

--(-1- + _2___3_) X_9_


. 2 3 4 25
a)
(
3
7
2
+ - -- - - x --
1 18
5
7
4
)
10 -
2
2 5 2 3
d) 1 - - - + - - + - x 17- 3
3 3 6 9 5 1-
4

e) ( +-+) (_
\
s __
4 )
6 5
~ b) [( 4 * [(3+-+x) 3+)
+3
f) 2 - - '\ -3-
-9 2 + 1
-5- ) .-;- 13 X 1--;-
7
(~ ~-2+)J 7
3 2
+
·~ x2+)J
g) - 5-;- 2 -
5 9 t

192. Pau ](> Pess Qa


Problernail de Matemática para o Admissão 193
4 1
3--
15 5
- o) [ 40 ...;- [ 6 + 30 ...;- 5 X 2 - 4 X
- e)
3
1 2 24 X 36 X 21 ]
2 -- 2 -- - - -- - 1 X 216
2 3 42 X· 9 X 12
Colégio Militar 1967
1 8
2 - 1 2
2 7 4 b) 1 -- +2 X 3 --
- d) X 3 3
3 1 3
5 ~
Ginasial - 1967
3--
5 5 5
2
1 Resposta: a) 40; b) 8 - -
~ e) 1 + 3
1
2 --
3 6 Reduza a expressão mais simples:

8 51 4 75 1 1
Resposta: a) - - ; b) - - - ; e) - - ; d) ----; 2-- X 3 X
9 8 45 392 3 2
3
e) 1 - -
7 1

- (j Resolver:
1
8 ---
2
7
2-- + Resposta:
3 15 Liceu Nilo Peçonha - 1968
X 46
5
4-- - 2
9 5 X \ 120 - [ ( 3 X 50 - 400 ...;- 4 ) X 2 ] ( _, 100
Resposta: 60 Colégio Militar - l9p7
Resposta: O Liceu Nilo Peçonha - 1967

194 Paulo Pessoa


Problem,.s de :U: atemátlca para o Admissão 195
Q Escrever em ordem crescente as fracões reduzindo-
-as ao mesmo denominador) : ,
ej
17
- - -'
63 24
7 11
e ---
36
8 7 8 3 1 7 7
a) --- e f) - - -' - -- '
---, ---
9 13 5 3 7 4

e
11
7 11 3 2
b) - - - , - - - e - -- Mário p ercorreu de uma estrada; Pedro
8 9 10 7
2 3
7 9 e Antônio - -· - . Quem está mais p róximo do
c) - -- e !1 4
10 7 l1111 do estrada?
5 2 7
d) - -- - - -' ll sposta: Antônio . Liceu Nilo Peçanba - 1967
'
12 5 18
'Í 11 7 5
11 7 17 'l.3) Tem-se as frações - - - , - - - e - - - Dizer:
e) - -- ' - -- e - - - 20 15 12
36 24 63
,i) q11ol a mais próxima da unidade? b) Escreva-as em ordem
3 7 7 , 1 111 e- 11 le de grandeza .
-==- f) - --
'
- - -
'
- -- e -- -
11 5 7 11
5 7 3 4
llosposta: a ) - - -; b) - - -, - - - e - - -
7 8 8 20 12 15 20
Resposta: a ) - - -, - -- e - - -
13 11 9 Colégio Pedro II - 1968
3 7 11
b) - -, - - e - - ( '. 1) Escrever em ordem de grandeza decrescente as
11111 ( 1( u
10 8 9
5 7 3 1
7 9 a) - - -' - - -, e - --
c) e --- 6 8 4 2
10 7
3 7 5 7
7 2 5 b) - -- - - -, e - --
d) - - -' '
e - - - 5 9 18 10
18 5 12

196 Pa.u lo Pes s oa


Problemas de Matemática para o Admissão 197
e)
1 4
- -- , - - - .
3 5
-~-
B

15
7
e - --
12
' 27) Complete as igualdades:

*
3 1 4 5 5
a) -- = 15
d) - - -, - - -, 5
- - - e -- -
4 3 9 12 6 *
b) = 256
Reduzindo-as ao mesmo numerador: 36
7 5 3 1 1050
Respos'. a: a) ---
8
. - -- , - - - e - -- e) 75 = - - -
6 4 2 *
7 7 196
b) - - - . - 10- -, - -53 - e - 5-- d) 28
9 18 *
4 7 8 1 180
e) ---
5
. - - -, - -- e --- e) 36 = - -
*
-
12 15 3
Resposta: 75; b) 9.216; e) 14; e. 5
5 3 4 5 l
d) - --
6
. .
- - -, - - - - - - e 28) Substitua os asterísticos pelos seus valôres nas
4 9 12 3 .1 CJUmtes frações:

.(!!) Reduza os números 29, 37, 41 e 23 a sétimos .


a)
1 4

Resposta
203 259 287 161 5 *
7 7 2 8
b)
@ Escreva o inverso de:
6 *
4 120 1
8 *
e)
4 - - -, 16, -- e - - 12 6
9 37 7
78 3
1 9 37 d)
Resposta: - - - , - - -. - - - - e 7 104 *
40 16 120
Resposta: a) 20; b ) 24; e) 4; d ) 4
198 Pau lo P es soa
Probl emas de M atemática para o Admi ssão 199
~ Escreva a fração cujo numerador seja 51 e que ·1 3
te~~ara valor 17 . 33) Quais as frações equivalentes a - - - e -- - ,
51 7 11
ResPosta: !ois que o denominador da primeira seja igual ao numera-
3 dor da segunda?
12 21
30) Calcular: Respos7a: - - - e - - -
3 21 77 Colégio Militar -- 1967
a) de 28
7 8
Dê a fração equivalente a - - - com numerçrdor
3
b) - - de 32 / 88.
11

4 88
4 7 Resposta:
121 Ginasial - 1967
e) - -- de de 240
5 8
9

e
4
Resposta: a) 12; b) 24; c) 168.

Escreva duas frações equivalentes a


2

9
e
/ @ Escrnva a fração
<los têrmos seja 48 .
18
equivalente a - -- cuja soma
15

tais, que o denominador da primeira seja igual ao ResPosta: ~


5 30
numerador da segunda . 18
36)
Escreva a fração equivalente a - -- , cuja soma
8 36 23
Resposta: --- e - -- d s têrmos seja 123 .
36 45
3 54
~ Determinar duas frações equivalentes a - - - e Resposta:
5 69
9
7
- - - , de modo que o numerador da primeira seja o triplo
8
@) Escreva a fração equivalente a - -- , cuja ciife-
15
do denominador da segunda 1t nça dos seus têrmos seja 12.
72 21 18
Resposta: - - -- e -- - Resposta:
120 24 30

2,00 Paulo Pessoa Problemi>s de Matemáti ca para o Admissão 20}


e 11
Escreva a fração equivalente a - - - . cuja dife-

rença de seus têrmos seja 16.


15
43) Calcular o número que se deve tirar do numera-
35
dor da fração - - - para torná-la sete vêzes menor.
91

44 Resposta: 30
Resposta:
60

e Achar uma fração igual a

nadar seja 91.


180

468
, cujo denomi-
Calcular o número que se deve tirar do numera-
45
dor da fração - - - - para torná-la 15 vêzes menor.
131

Resposta: 42
35

a
Resposta:
91

6
Achar uma fração igual a - - - , cujo denomi-
G Os + de uma peça de fazenda custa NCr$

180,00. Quanto custará a peça tôda?


8
nadar seja 36. Resposta: NCr$ 720,00

27
$
Resposta:
36 Para despachar - - - de uma colheita de laran-
5
jas, empregaram-se 12 caixotes . Quantos caixotes teriam
tri'\\ Calcular o número que se deve tirar do denomina- ~ido necessários Para despachar tôda a colheita?
Q 13
dor da fração - - - para torná-la cinco vêzes maior. Resposta: 20 Colégio Pedro II - 1968
45

Resposta: 36

~ Calcular o número que se deve tirar do denomina-


e Qual o númern cujos

3
~ aumentado de
3
1

\:.:./ 15 diminuído de - -- é igual a 115?


dor da fração - - - para torná-la sete vêzes maior. 4
28
Resposta: 24 Resposta: 5 ~_::)00
202 Paulo Pessoa Problcm a:s de Matemática pu.ra o Admiss ã o 203
~
4 1
Qual o número cujos - - - aumentados de - --
~
4
7 4 Os - - - de um terreno foi adquirido por NCr$
3 7
e diminuído de - - - , é igual a 17? 4
14 5 . 960,00. Quanto teria pago se adquirissem - - - do mes-

e Resposta: 28

Um rese;vatório cheio dágua contém 24 litros.


mo terreno?
5

+
Resposta: NCr$ 8 . 344,00
5
Quantos litros conterão

Resposta: 20
6
do reservatório?

V
@ Os do piso de uma sala foram ladrilhados

3 com 360 ladrilhos . Quantos ladrilhos serão necessários para


50) Qual o número cuios - - - valem 39? 2
ó ladrilhar os - - - do piso?
Resposta: 65 / 3

ô 1
Distribuem-se 3 - -- quiios de açúcar en '.re
Resposta: 640 Ginasial - 1967

2
1
vários pobres. Cada um recebeu - -- de quilo Quantos
0 1
Sabendo-se que 5 - --
2
peças de fazenda

4
5
foram os pobres beneficiados? custaram NCr$ 7.634,00. Qual será o preço de apenas - -
8
Resposta: 14 da peça?

@ A quarta parte dos

Qual é o número?
+ de um número é 17.
Resposta: NCr$ 867,50

~ A soma de dois números é 128 . O primeiro vale


3
- -- do segundo . Achar os dois números .
Resposta: 119 5

(!})Os - --
4
3
de uma Peça de fazenda custam NCr$

12,00. Quan~o custará a metade da peça?


Resposta: 48 e 80

'
~ A soma de dois números é 595 e um dêles é igual
e.e - - - do outro. Achar os dois números.
Resposta: NCr$ 8,00 5
Resposta: 175 e 420
?Q·A
~·'-:1! Paulo Pessoa

Problemas de Matemática pa.ra o Admis são 205


e + Se dos de um número subtraírmos

do mesmo número, sobram 45 . Qual é o número-


3

7 3
2
Em um ginásio - - - dos alunos são meninas .

O número de meninos é 315 . Qual o total de estudantes?

Resposta: 945 Ginasial - 1967


Resposta: 189

f!§i)3
somando-se 25 a um número, o resultado repre-
G uma pessoa gas!a + d o seu º'denado mais
senta - - do mesmo número. Qual é o número?
2 NCr$ 200, 00. Se economiza NCr$ 600, 00, quanto recebe por
mês?
Resposta: 50
Resposta: NCr$ 1 . 400,00 ~
~ Subtraindo-se 150 unidades d eum certo número
\.J 13 66) Uma torneira enche um tanque em 2 horas e outra
em 3 horas . Estando o tanque vazio, em quantas horas as
obtém-se - - - dêsse número . Qual é o número?
18 duas juntas o encherão?
Resposta: 540 1

B Uma pes.oa fêz comprns no valo' d e + · de-


Resposta: 1
5
horas

1
3 5
Pois e por fim - - - do que possuía. Ficam deven- 67) Uma torneira enche um reservatório em
4 12 9
do NCr$ 5,00 . Quanto tinha? 1
do dia e uma outra o esvazia em - - - do dia. Abertas
Resposta: NCr$ 7,00 7
no mesmo instante, no fim de quanto tempo o tanque ficará
Na resolução de um problema, um aluno empregou
cheio?
2
do tempo concedido para pensar; - - - para re- 1
3 9 Resposta: dia
5 2
solvê-lo e - - - para corrigi-lo e passar a limpo . Tendo 68) Dois operários podem fazer uma obra em 6 dias.
18 Um dêles sozinho, seria capaz de realizá-la em 8 dias .
g~st~ apenas 20 minutos, pergunta-se: Com ,q ue antece-
Quantos dias levará o outro para fazer a mesma obra?
denc1a concluiu o problema?
Resposta: 4 minutos Colégio Militar. - 1967 Resposta: 24 dias

:206 Paulo Pessoa


P robl ema:s de Matemática para o Adm issão 207
69)
Um operário faz um trabalho e m 6 horas. Junta - 1
3 t ros . A segunda com da peça e mais 5 metros.
mente com outro seria capaz de fazer os - - - d o traba- 3
4 /\. terceira com os 7 me tros res tantes. Quantos metros rece-
lho em 3 horas. Em quanto te mpo o segundo operário seria IJ u cada p essoa?
3
capaz de fazer do mesmo trabalho? Resposta: 28 m; 25 m e 7 m
5
Dividiu-se uma quantia por três pessoas. A pri-
73)
1 3
Resposta: 7 - - - horas 1nnira recebeu - -- da quantia meI).os NCr$ 140,00; a
5 5
1
/'7õ)\ Uma herança foi distribuída entre quatro irmãos .
~ 2 3 ,1 c1 unda - -- mais NCr$ 25,00 e a terceira NCr$ 1. 900,00.
Ao primeiro coube - - - ; ao segundo os 4
do resto;
3 4 <Jual era a quantia?
1
ao terceiro - - - da soma das partes dos dois primeiros Resposta: NCr$ 11 . 900, 00
33
74) A quantia de NCr$ 680,00 foi dividida entre três
e ao quarto NCr$ 20 . 000,00 . Qual a parte de cada um dos
I '"';. oas. A primeira e a terceira receberam, respectivamente,
herdeiros?
2 3
Resposta: NCr$ 240 . 000,00; NCr$ 90. 000,00; NCr$ - e - - - da importância recebida pela segunda .
10,000,00 4
2 11 mto recebeu cada uma?
71) Um negociante vendeu a um freguês das
3 Hesposta : NCr$ 128,00; NCr$ 320,00; NCr$ 240,00
1
maçãs que possuía e mais 3; a um segundo vendeu - - - 75)
Dividir 671 maçãs entre três pessoas de modo que
4 2
d as que possuía . Quantas maçãs tinha o negociante, sab'3n- 11 pdmeira receba - -- do que couber à segunda e a se-
d o-se que o primeiro freguês recebeu mais 38 maçãs do 5
q ue o segundo? '3
1p11H lu receba do que tocar à terceira.
Resposta: 84 8
72) Uma peça de fazenda foi dividida entre três Pes- H ~posta: 66; 165 e 440
i2
s oas . A primeira ficou com da peça e mais 4 m e- "/G) Gilda comprou no ano passado uma casa por
s 1J< ' 1 S 12. 000,00 . Há um mês, vendeu a casa , obtendo um

.208 Pau lo Pessoa


Problema<i de Matemática pa.ra o Admissão 209
lucro igual à quarta parte do prêço da compra. Qual o 81) A soma de dois números é 380 . Ca lculá-los, sa-
prêço da venda? 2 3
li ndose que os - - - de um equivale aos ___ do
Resposta: NCr$ 14. 400,00 Ginasial 1967
3 5
77) Margarida e Eduardo têm juntos NCr$ 123,00. Mar- ou tro .
2 3
Resposta: 200 e 180
garida gastou - - - do seu dinheiro e Eduardo os - - -
5 7
do seu, ficando com quantias iguais. Quanto possuía . 82) Adicionando-se à maior fração imprópria de d eno-
111111ador 3 compreendida entre 2 e 3, o quíntuplo do seu
Eduardo? 1111merador, quanto obteremos?
Resposta: NCr$ 63,00 J.t:. C. Dutra - 1968
128 2
78) Das frações próprias cuja soma dos têrmos é 53, Resposta: 42--
escreva: a) a maior; b) a menor . 3 3
26 1 Colégio Militar - 1968
Resposta: ---e---
27 52
Colégio Pedro II - 1968
15 9 6
79)Dados as frações - - - ; - - - e - - - , divida
16 - 20 7
cada uma delas por 2 e escreva os resultados em frações
com os mesmos numeradores .
45 45 45
Resposta: - - - , - - - - e - - - -
96 200 105
Colégio Pedro II - 1968
7
80) Iniciei uma viagem de 1210 km, sendo - - - de
11
2 3
avião; - - - do resto de automóvel; - - - do nôvo
5 8
resto de trem . O restante fiz a cavalo. Quanto fiz a cavalo?
Resposta: 165 km. Liceu Nilo Peçonha - 1968

2.10 Paulo Pessoa Probl ema;s de Ma t emática p ar a o Admissão 211


,

NúMEROS DECIMAIS

Chama-se fração decimal tôda fração cujo denominador


1 ttma potência de dez .

Exemplos:
19 137 3819 207
- - -, etc.
10 100 1-000 10 .000
Essas frações decimais, correspondem aos números deci-
"'"is:
1,9; 1,37; 3,819 e 0,.0207;
q 11ociente das divisões dos numeradores das frações deci-
l 1111 • dadas pelos respectivos denominadores.
O algarismo que fica à esquerda da vírgula é a parte
1111< iw; os algarismos que aparecem depois da vírgula, cha-
'1n1m-se algarismos decimais e o seu conjunto, a parte
c/1 • ·11nal.
Com os números decimais podemos fazer as mesmas
• 1it rações que fazemos com os números inteiros . Podemos
l111nbém combiná-los, através das operações correntes,
1 '1111 números inteiros, como mostraremos adiante.

Os números decimais, contudo, não provêm apenas das


1i1 •c;u s decimais.

Qualquer fração ordinária cujo numerador for dividido


I" lo denominador (a fração indica uma divisão do
1111111 rador pelo denominador), gera um número decimal, que
/'mie ou não representar exatamente a fração ordinária de
• 11q11m

Problemas de Matemática pa.ra o Admissão 2 J.3


No primeiro caso, efetuada a divisão, chega-se a um 1 5
resto nulo, depois de obtido um certo número de algarismo$ No caso das frações e - - não é conseguido um
decimais no quociente e por isso o número decimal obtido 3 11
representa exatamente a fração ordinária considerada . 1 esto nulo, pois
Assim teremos os números decimais exatos ou dízimas finitas.
Os números decimais escritos acima, representam exa- 1000 3
tamente as frações decimais ci~ados como exemplo e são
lidos: 10 0,333 ...
10
1,9 - um inteiro e nove décimos.
1,37 - um inteiro e trinta e sete centésimos . 1
l.º
3,819 - três inteiros e oitocentos e dezenove milésimos.

0,0207 - duzentos e se'.e décimos milésimos 50 000 11

O primeiro algarismo depois da vírgula representa dé- 60 0.4545 ...


cimos; o segundo representa centésimso; o terceiro indica
os milésimos; o quarto é o dos décimos milésimos; o quinto 50
é o centésimo de milionésimos; o sexto é dos milionésimos 60
1e assim sucessivamente.
5
13
Na fração---, por exemplo, teremos: 2.º
4
1
Com a primeira fração
(---) os algarismos do
13 4 3
q uociente são o algarismo 3. repetidos indefinidamente, por
10 3,25
20 111ais que se prolongue a divisão, pois o resto é sempre o
111 smo.
o Com a segunda fração o quociente é constituído dos
Como se vê, no quociente encontrado apareceram duas "lqarismos 4 e 5 que se sucedem sempre na mesma ordem,
<:asas decimais, isto é, 25 centésimos e o resto foi nu!o . 11ma vez que os restos encontrados são sempre 6 ou 5 como
•. 1 vê.
13 Nos dois casos temos uma dízima periódica simples,
Dizemos então que a fracão - - - tem o mesmo
~ 4 1 or mais que continuemos a divisão.
valor que o número deema! 3,25 (três inteir·o s e vinte e O quociente obtido em tais casos, pode entretanto, ser
cinco centésimos). rl11 duas naturezas, isto é: os algarismos decimais do

214 Paulo Pessoa Problema·s de Matemãtica para o Admis são 215


quociente são iguais ou quando diferentes, passam a se rismos decimais, que, uma vez aparecidos no quociente, não
repetir a pmtir de um dêles, obedecendo sempre a mesma mais se reproduzem a partir de um instante.
ordem ou seqüência. Teremos então o que se chama uma dízima periódica
Teremos, nesse caso, uma dízima periódica simples. O composta
algarismo decima l ou grupo de algarism os decimais que se
repete, obedecendo à seqüência acima referida, é o que se O algarismo ou grupo de algarismos que apare:::e de
cha ma período da dízima periódica simples . início no quociente e depois não figuram mais nêle, é a par ~e
l não periódica ou o anti-período da d ízima p eriódica composta
Vejamos os dois casos, empregando as frações e os que se repetem numa mesma seqüência constante,
3 como no caso da dízima periódica simp les, é o período ou a
5 parte periódica da dízima .
e - - - , per exemplo .
11 7 23
Vejamos por exemplo a s frações: --- e - --
Fazendo as divisões, teremos:
30 12
l
com um algarismo no período ( 3 ), no caso da fração Fazendo as divisões, vem:
3
5
e com dois algarismos no período ( 45 ), no caso de - - - 70 000 30
11

As dízimas periódicas simples podem ser representados:


100 o. 2 333 ...
100

0,3333 . . . . ou O,( 3) ou 0,3 ou 0,3 10

•• 10
0,454545 ... . ou O,( 45) ou 0.45 ou 0.45

No caso da dízima ter parte inteira, a maneira de


representá-la é a mesma . 230000 12

Assim, se tivermos a dízima periódica simples: 110 l , 9166 ...


3,471471471 . . . poderemos representá-la també m: 20
••• 80
3,( 471 ) ou 3,471 ou 3,471
80
Em outros casos, a repetição acima referida, só começa
8
a ocorrer, d epois d o aparecimento de um ou vários a lga-

2,}6 Pau lo Pessoa. Problemas de Mntcmâtlca para o Admissão 217


Assim se quisermos transformar a fra ção ordinária
7 5
Com a primeira fração - - - , o algarismo 2 do - - - , em número decimal com quatro (4) a lgarismos deci-
30 7 .
quociente aparece mas não se repete . É a parte não ma is procederíamos como se segue:
periódica ou o anti-período. Por outro lado o algarismo
3 se repete indefinidamente. É o período da dízima perió- 5 50 000
dica composta. ou 0,714. 2
7 7
Com a segunda fração, o quociente mostra a parte
inteira do decimal e que é um. A parte decimal apresenta Tal resultado, como dissemos, não é exatamente igual
os algarismos 9 e 1 que depois são sucedidos indefinida- 5
mente pelo algarismo 6. a - - - porque a divieão não se faz exatamente por mais
7
Como no caso anterior, 91 é a parte não Periódica ou que a prolonguemos. Dizemos, entretanto, que o, 714. 2 é e
o anti -período e o 6, a parte periódica, isto é, o período. 5
número decimal correspondente à fração - -- , com êrra.
As dízimas periódicas compostas podem ser repre- 7
sentados: me:wr que um décimo milésimo, por falta como diríamos
que O, 714. 3 é o número decimal correspo'ndente à fraçãai
O, 2 333. . . ou O, 2 (3) ou O, 2 3 ou O, 2 3 5
• com êrro menor que um décimo milésimo (O, 000 . U
1. 91 666 ... ou 1. 91 (6) ou 1. 916 ou 1. 91 6 7
por excesso.
A divisão não se processando exa'. amente, os quocien-
tes obtidos, quer representem dízimas periódicas simples,
. Vimos assim que: tôda fração ordinária gera um número
quer compostas, não correspondem, exatamente à fração decimal,, ~ue tant?, p_ode ~er uma dízima finita ou decimal
exato; dizima penod1ca simples ou dízima periódica com-
considerada. posta.
Quanto maior fôr o número de algarismos decimais Não é preciso porém, efetuarmos a divisão do numera-
do quociente, mais êle se aproximará do verdadeiro valor dor p elo denominador de uma fração, para sabermos a na-
da fração, sem entretanto ,atingí-lo; daí a possibilidade tureza do decimal em que ela se converterá, mesmo porque.
de obtê- lo com maior ou menor aproximação, isto é, com em ceitos casos, a conclusão procurada seria demorada
maior ou menor número de casas decimais. por serem muitos os algaris mos q ue figurariam no quocie;
te, antes que à sua repetição começasse a ocorrer .
Por isso, quando se deseja transformar uma fração
·o rdinária em um número decimal com 5 casas decimais, Por isso, quanào desejamos saber a na tureza do número
por exemplo, acrescenta-se à direita do numerador da deci~nal que será obtido com a divisão do numerado: da
fração, 5 zeros e divide-se o número assim formado pelo f~ ª!ªº p or seu denominador, inicialmente devemos obriga-
denominador da fração como fizemos por ocasião das divi- tonamente ~ornar ': fração irredutível e em seguida decmn-
por em fatores pnmos o seu denominador..
sões apresentadas antes.

218 Paulo Pessoa


Pr ob l em a·s c1•e M a t ema•t·i ca par a o A d missão 219
Feito isto, poderão ocorrer três casos:
2. 0 ) Na decomposição do denominador em fatôres pri-
1. 0
) Na decomposição do denominador em fatôres pri- mos, só aparecem fatôres primos diferentes d e 2
mos, só apareceram os fatôres primos 2 e 5, ou e de 5.
somente 2, ou apenas 5 . Concluiremos então, sem precisar efetua r a di-
visão, que a fração se transformará em uma dízi-
Concluiremos então, sem p recisar efetuar a ma periódica simples.
citada divisão, que a fração se transformará em
dízima finita ou decimal exato. 26
Seja a fração - - - que depois de simplifi-
27 42
Seja a fração - - - que depois de tomada
12 13
9
cada, passa a ser
irredutível é equivalente a - - - · 21
4 O denominador, decomposto em fatôres primos
Decompondo 4 em fa tôres primos vem: 4 = 22 é : 21 =
3 X 7, não possuindo fatôres primos 2 e 5.
27 Trata-se então de uma dízima periódica sim-
Concluiremos então que a fração 26
12 pies, o decimal gerado pela fração - - -
quando transformada em decimal se converte em 42
decimal exato ou dízima finita.
3. 0 ) Na decomPosição do denominador, da fração, em
O número de algarismos da parte decimal é fatôres primos, aparecem os fatôres primos: 2 e
o da maior unidade das potências de 2 ou 5 que outros que são 5; 5 e outros que são 2 ou 2, 5 e
figurar na decomposição do denominador. outros.
27 Concluiremos então tratar-se de uma dízima
Então a fração - - - se transforma em deci- periódica composta, com tantas casas decimais na
12 parte não periódica quantas fôrem as unidades da
maior das portências de 2 ou de 5 que figurar na
mal exato ou dízima finita com duas casas na decomposição do denominador .
parte decimal. 11
7 Figuremos o caso da fraçã o - - - - ,já
2
Se a fração fôsse - - - - , como 200 = 23 X 5 150
200 irredutível .
concluímos que ela se converterá em decimal exata
com três casas decimais, porque dos expoentes de Como 150 2 X 3 X 52 concluiremos que a
2 e 5 que apareceram na decomposição de 200, o fração dada gerará uma dízima per:iódica composta
maior é o do fator primo 2, que é 3, enquanto que com duas casas no anti-período ou parte não perió-
o do fator 5 é 2 dica , porque dos fatôres 2 e 5 que apareceram na
decomposição de seu denominador, o fator 5 está
2.20 Paulo Pes s oa
Probl cma·s de M atem á t i ca pa,ra o A dm is são 221
Como no exemplo anterior, teremos:
elevado ao quadrado (potência 2), sendo o seu ex- 15
poente maior que o do fator dois (expoente 1). 0,039
Assim como a simples observação do denomi- 1, 18
nador da fração ordinária irredutível permite: de
um modo rápido, concluir a naturez~ do de~1mal 16,219
por ela (fração) gerado, assim tambem, mediante
as regras que se s~guem, poderex_:ios pr_oc,e~er de 3) Subtrair:
modo inverso, isto e, achar a fraçao ordmana que 17, 5 - O, 000.087
gerou o número decimal considerado .
A simples maneira pela qual o número deci- Para efetuar a subtração de decimais, procede-se no
mal se nos apresenta escrito, indica a natureza da que diz respeito à arrumação do minuendo e subtraendo,
dízima a ser convertida em fração . como se procedeu com as parcelas da soma . A seguir sub-
Tais regras serão enunciadas por ocasião da trai-se como se fôssem números inteiros A vírgula da dife-
apresentação dos exercícios resolvidos da espécie. rença entre os números fica na mesma vertical que as vír-
gulas do minuendo e subtraendo .
EXERCíCIOS RESOLVIDOS Assim:
1) Efetuar a soma: 17, 500 000
o. 000 087
12,52 + 123,435 + 7,8
17, 499 913
Para somar decimais escrevem-se os números a somar,
uns debaixo dos outros, de modo que as partes inteiras (se Como vimos, para facilidade do aluno, as casas deci-
existirem) figurem com as ordens do_ mesmo noi;ie, umas mais de 17, 5 foram completadas com cinco zeros para que
debaixo das outras . Isto implica em ficarem as virgulas na houvesse correspondência das casas do rninuendo com as
mesma linha vertical. do subtraendo.
A seguir somam-se como se fôssem números in_teiros e
coloca-se na soma, a vírgula na mesma linha vertical em O mesmo poderá ser feito quando tratar-se da soma.
que figura nas parcelas. embora não sendo necessário em qualquer dos casos .
Assim: 4) Subtrair de 6, 327, quatro inteiros
12,52
123,435 DePois do que foi dito, teremos:
7, 8
6,327
143,755 4,000
2) Somar 2,327
15 + 0,039 + 1, 18
Probkmas de Matemática pe.,·a o Admissão 223
Pessoa
5) Multiplicar 3, 471 por 12 . .n1aís do dividendo e do divisor, acrescentando zeros se fôr
Para multiplicar números decimais, procede-se como em necessário ao que tiver menos casas decimais, de modo
casos de número inteiros o separam-se no produto tantas a igualá-las, depois do que retiramos a vírgula do dividen-
casas, .ªpartir da direita quantos fôrem as do muliiplicando do, ou do divisor ou de ambos, conforme o caso, efetua-se
e multiplicador, somadas . a divisão como se fôssem números inteiros.

Então:
3,471 15,039 12 (inicialmente)
X 12
6 942 15,039 12, 000 (igualando as casas)
3 471
41, 652
. . Foram sep~radas três casas no produto porque 0 mul- 15 039 12 000 (retiradas as vírgulas)
1tphcando tem tres casas decimais e o multiplicador nenhu-
ma, sendo assim 3, o número de casas dos fatôres (multi- 3 039 l
plicando e multiplicador) .
Encontramos o ,q uociente 1 e o resto 3 039, que na reali-
6) MultiPlicar 14, 015 por 3,19 . dade é 3, 039. Para continuarmos a divisão uma vez que
não existe nenhum outro algarismo no dividendo, coloca-se
Depois do que foi dito, vem: uma vírgula no quociente, depois de 1, no caso do exemplo
e acrescenta-se um zero à direita do resto e colocamos o
14,015 .segundo algarismo no quociente. Para continuarmos a divi-
X 3,19 são repetiremos a colocação de outro zero no segundo resto
126 135 "' assim sucessivamente até obtermos o número de casa.<J
140 15 decimais desejadas, no quociente.
420 45
Assim teremos:
44,70785
1
Foram separadas cinco casas no produ'. o a contar da 15 039 1 12 000
direita para à esquerda porque o multiplicando tem três carns 1

decimais e o multiplicador duas casas decimais, sendo o . 3 0390 1.253


total 3 + 2 = 5.
.63 900
7) Dividir 15, 039 por 12. (Decimal por inteiro) .
Para dividir decimal por inteiro ou inteiro por decimal .39 000
ou decimal por decimal. começamos igualando as casas deci-
,3 ººº
224
Problem~s
I'aulo Pessoa
de MatemúLica para o Admissão 22.S
Ós zerós grifados ( O) forarri os acrescentados para de vírgula e acrescenta-se tantos zeros necessários para
efetuar a operação A seguir prossegue-se como no exem-
continuar a divisão que foi prolongada até a terceira casa plo anterior.
decimal.
Aproveítomos a oportunidade para mostrar como se sabe A.o;sim:
o resm encon~rado, que apare11lemente é 3. 000
A prova real da divisão é feita, como vimos, multipli-
canoo-:::.e o divisor pelo quociente e somando-se ao prociuto 15 390 4000
o resto. Se a operaçao estiver cena, o resultado sera igual
ao dividendo, que, convém não esquecer, é 15, 039 e o divi- 3 3900 0,38475
sor 1~ . .l;;ntão:
19 000
quociente -----7 l,253
aí visor -----7 12 30000

2506 20ººº
1253 oººº
Produto -----7 15,36 Neste exemplo a divisão acabou se fazendo exatamente,
depois de encontrar no quociente 5 casas decimais
Ao produto deve ser somado o resto para acharmos o
dividendo 9) Dividir 7 por 8, 45

Então: Depois do que foi dito nos exemplos (7) e (8), efetuaremos
15, 036 + Resto 15,039 a divisão sem: repetir as fases assinaladas naquele·s exercí-
cios.
Então:
Resto = 15, 039 - 15, 036 O, 003 (três milésimos) Teremos:

8) Dividir l, 539 por 4


7,000 8,45
Depois do que dissemos, vem: 0,828402
l 2 400
l, 539 1 4, 000 ou
7100
1 539 4 000. 3 400
2 000
Como o dividendo é menor que o divisor, a divisão não
pode ser feita. Coloca.mos então zero no quociente seguido 310

226 Paulo Pessoa


A colocação de um zero não permitiu a d1vi.são. Colo-
12) b1vídir ó, 0382 per 1, 5
cou-se um zero no quociente e outro no resto para que fôsse
possível pTosseguí-la Como já foi feito das outras vêzes:

10) Dividir 5 por 0,32 0,0382 l, 5

Teremos: 0,0382 1. 5000 ou

38 200 ,15 000


5 00 O, 32

1 80 15,625 . 8 2000 O, 0254


2 00 . 70000
o 80
160 10 ººº
00 NOTA: nos diferentes exemplos apresentados, o zero
grifado (O) representa os zeros acrescentados nos restos
para prosseguir a divisão.
11) Dividir 20, 457 Por 11. 3 5
13) Converter em decimal a fração ordinária
6
Como já foi dito:
Como dissemos anteriormente, é bastante dividir o nume-
rador pelo denominador da fração, com tantas casas no
quociente quantas desejamos, desde que a divisão não se
20,457 1I. 300 faça exatamente.

. 9 1570 L 8103 Teremos então:


50 6
.1 1700
.40 000 20 0,833
.6 100
20
O resto é: O, 00061.
2

Problemas de Matemática pa,r a o Admissão 229


14
14) Converter em decimal o número misto 10 depois de simplificada a fração - - - e extraídos os intei-
10
De início teremos: 20
7
ros da fração - - - .
11 211
10 5
e depois
20 20
1 17) Converter em fração ordinária irredutível o deci-
1 mal 1, 875.
211 1
20
- - - - -- - Problema idêntico aos anteriores que iremos resolver de·
.110 10,55 (divisão exata) outro modo, isto é, colocando-se sôbre o traço de fração o
número dado, sem a vírgula, (numerador da fração) sob
100 o traço, a unidade seguida de tantos zeros quantos fôrem as
00 casas decimais do número (no caso em questão, três).
15) Converter em fração ordinária o decimal O, 73
Assim sendo:
Pelo modo que foi escrito o decimal, concluiremos tra-
tar-se de um decimal exa to ou dízima finita e que se lê: 1875 15 7
setenta e três centésimos 1,875 = - - - 1 depois
1000 8 8
Então: de simplificar e extrair os inteiros.
73
O, 73 18) Converter em fração ordinária irredutível o deci-
100
ma l O, 4
Como vemos, o decimal é igual a uma fração, que t6in
Pelo modo como está escrito o decimal, verificamos tra-
para numerador a par:e decimal e para denominador, a uni-
tar-se de uma dízima periódica simples, cujo período é 4.
dade seguida de tantos zeros quantos fôrem as casas deci-
mais. Para transformá-la em fração ordinária, isto é, Para determi-
lnar a sua fração geratriz, colocaremos o período ( 4 ) como
>numerador de uma fração que terá para denominador, tantos
16) Converter em fração ordinária o decimal l, 4
algarismos 9 quantos -fôrem os algarismos do período (um,
O decimal representa: um inteiro e quatro décimos. 1t10 caso).
Depois da regra dada no exemplo anterior teremos: Teremos então:
4 4 o 4
l, 4 = 1 + - - - = 1 ---- O, 4 · O, ( 4 ) = O, 4 = O, 444 ...
10 10 9i
,_
10 X l +4 14 7 2
19) Converter em fmçã o ordiná ria irredutível, o deci-
- - - :==1 - - - 1--
10 10 5 mal O, ( 431).
5
2.30 Pa.uio Pessoa P robl t..·mus Uc !v1ati..n:n á ti ca . - ""''>31
o A d mrnoao
De acôrdo com o que foi dito no exemplo anterior, tere-
mos: 22) Calcular a fração geratriz do decimal 2, 864
O, (431) = O, 431 L....-! O,··~
431 O, 431 431. ..
431 Como de outras vêzes podemos escrever:
-----
999
20) O mesmo problema com relação ao número 3, (12)
2, 864 =
2 + O, 864 e como a segunda parcela
~ndica uma dízima periódica composta, podemos escrever:
Pelo que foi feito no problema 16 podemos escrever:
864 - 8 856 482
3, (12) =: 3, 12 =
••
3, 12 = 3, 12 12 . . . = 2 + - - -- 1= 2+ 2+---
12 99 X 3 .+ '12 990 990 495
= 3 + O, 12' 12. . . = 3 + depois de simplificar a fração.
99 99
297 + 12 309 103 4 23) Calcular a fração geratriz do decimal 1, O (13)
=3--
99 99 33 33
Convém salientar que o processo empregado na resa- Como no exemplo anterior, teremos:
] ução do exercício 17 não pode ser empregado quando se
trata de dízima periódica com a mesma simplicidade. 013 - o 13
1 + 0,0(13) 1 + - -- l+ - - -
21) Calcular a fração geratriz do decimal O, 3 (47). 990 990
Trata-se de uma dízima periódica composta, cuja parte
não periódica (anti-período) é 3 e cujo período é 47 . 24) Resolver a expressão:
Sua fração geratriz tem para numerador a parte· não
periódica acompanhada do período, e do ncmero assim for- 2 72 5
mado subtraída a parle não periódica. ( 0,1 +- -) X O, 333 ... X
Para denominador, tantos algarismos 9 quantos fôrem \ 10 8 10
os algarismos do período, seguidos de tantos zeros quantos
fôrem os algarismos da Parte não periódica. Transformemos os decimais em frações ordinárias .

Sendo assim: 1
ce O, 1
0,3(47) == 0,347 :=1 0.347 O, 3 474747 .. . = 10
347 - 3 344 172
!==' - - -- - - -- , depois de 3 1
990 990 495 O, 333 . .. =
tornada irredutível a fração. 9 3

232 Pa.nlo Pessoa ProblemRs de MatemáticR pao·a o Admissão 233


Então a expressão será:
Teremos então a expressão dad asob o seguinte: aspecto:
7 6 17
(
_1__, + _2_) X _1_ X ~ - _s_ ou --- x - - ---5
10 10 3 8 ilO 9 5 5

3 1 5 9 14 36
+ 2 9
ou

X X 9 - - - = - - ·- X
10 3 10 10 9 25 3 18
5 4
42 17 1
- - - = - - = 0.4
~- x
10 10
4.5 5 5
ou
25) Calcular o valor da expressão:
504
+ 2 18
O, 77 ... X 1. 2 3,4 5 X
+ - - - - - -- 225 3 9
1, 55.. . X 1. 44 2 9
14 17
3 18
15 25 14 ,56
Procedendo-se como da vez anterior: +----=
25
+
56 4 15
7
0.77 . .. '- - - 25 3
9
12 6 17 4 14 25 17
1. 2 - - -- +--~--=--X--+ X
10 s 25 3 15 56 25
3 5 51 125 + 153
s 14 X =--+
L SS .. . t:=, 1
+ 9 9
4 12 100 300
278 139
144 36
1.44 ---- 300 150
100 25
26) Um aluno enganou-se na escolha dos têrmos de uma
34 17
~ivisão. Considerando o dividendo como divisor, achou para
3,4 - --- =--- o ,q uociente O, 658. Qual era o verdadeiro quociente?
10 s
Problema·s de Matemática pa.ra o Admissão 235
ü ê11CJtttlo do aluno levou-t> u etchcu- Uni q uociente inv ê1·~ De acôrdo com a regra dada a fração se transforma em
so do que deveria ter achado. Por conseguinte para repa- uma dízima periódica composta com dois algarismos no
rar o seu engano êle deverá achar o inverso do quociente nti-período .
1
~ncontrado, ou seja A fração:
0,658
15 3 3
Assim: =~e também, de acôrdo
40 8
rom o que foi dito anteriormente ela gerará uma dízima
1000 O, 658 fi ni ta ou decimal exato, com três casas decimais .

3420 l, 519.7 A fração:


1300 8 8
6420 e também já foi explicado que
21 3 X 7
4980 t>m tais casos trata-se de uma dízima periódica simples.
374 28) Os O, 5 da idade de um homem são 20 anos. Seu
1ilho tem O, 10 da sua idade . Quantos anos tem o filho?
O quociente exato é então 1, 519 . 7
Basta transformarmos os números decimais em frações
0 1 clinárias Para recairmos em problemas de frações, resolvi-
27) Dizer sem converter, a natureza das dízimas: 1 los em outra ocasião .

7 15 8 Assim :
- - -; - - - e ---
12 40 21 5 1
o.s
Como já foi dito é bastante tornar as frações irredutíveis 10 2
e em seguida decompor em fatôres primos seus denomina-
dores . Então teremos:
l
Assim a primeira: - -·- da idade do pai - - - ---) 20 a nos
2
7 7 2
r idade dp pai - - - -~--...- ~ 40 anos .
12 2 2
X 3 2

236 Paulo Pessoa


EXERCíCIOS P ARA RESOLVER
10 l
== - - - =-
Sé seu filho têm O, 10

1
1
lOQí 10
da

a)
b)
-
Efetuar as operações:
7,'8 :
3,04
+5, 004
+0,6 +
+
18, 3
8,06 6,4 +
Made do pai, basta calcular - - da idade do pai, e) 35,0045 +0,985.4 +
0,010.1
10
(40 anos) para se ter: Resposta: a) 31,104; b) 18,l; e 36
1 Efetuar as operações:
- - X 40 = 4 anos (idade do filho).
10 a) 17, 2 - 5, 47
b) 9 - 2, 005
29) Q uem gastar O, 125 de uma fortuna; depois gastar e) 0,06 + 0, 012 - 0,0075
O, 45; tornar a gastar O, 2 e ainda ficar com NCr$ 9,00, -d) 48,01 - 8,04 - 9,89
quanto tem? x e) 715,004 - 306,45 + 2,006 - 410.5
)< f) (43, 81 - 18, 715) - (15, 08 - 9, 32)
Os gastos somados foram de: '<. g) (38 - 17,3) - (47, 83 - 28)

o. 125 + o. 45 + o, 2 = o, 775
~ h) 50, 005 + 20, 05 - º·
8

ResPosta: a) 11. 75; b) 6, 995; e) O, 0645; d) 30, 08; e)


Considérando que seu dinhiero era 1, ainda firam com O, 06; f) 19, 335; g) O, 87; h) 69, 255.
1- 0.775 = 0,225

Então:
) 3, 5 X 4, 136
0.225 --~ NCr$ 9, 00 ou - b) 5, 0007 X 3, 2
225
- e) 15, 32 X 6, 8 +l. 25
.)(d) (23, 4 - 22, 006} X (32, 16 - 31, 22)
--~ NCr$ 9, 00 ou ...: e) 3 (1, 732 - l, 414.2) - O, 87
1000
f) 23, 1 (3, 481 . - 3, 381) 15, 4 +
1 9,00 g) 5, 45 X 10 - (38, 3 - 27, 18) X 2
--~ ou 2
1000 225 h) (5 - l, '5 X - - ) X 0,2
3
1000 9 000,00 i) 14, 7 X O, 5
-·-~ ·= NCr$ 40,00 Ginasial - 19p8
1000 225
Resposta: a) 14, 476; b) 16, 002.54; e) 130, 22; dl
Poderíamos chegar ao mes~o resuJt9do se dividíssemo~ l, 310 . 36; e) O, 083. 4; f) 17, 71; g) 32, 26; h) O, 8; i) 7, 35
NCr$ 9, 00 por O, 225 , - · ,

Problen111•s de ;Matemática para o Admis siío 239


(}•tum: 7) Preencha a lacuna:
24, 8
a)439-:-- 0,0025
b) 5 -:-- 4, 95
6, 003.6 + - -- - = 3,523.ê
lQ
e) 34, 27 -:-- 23 Resposta: O
d) 73, 89 -:-- 45 Colég i o Militar - 19ti'l
e) O, 384 -:-- 4
- f) 7 49, 65 -:-- 38 8) Qual o resultado da expressão:
-g) 8, 447 -:-- o, 24
h ) O, 14 -:-- O, 5 7 - O, 5 7
- i ) 4, 7 -:-- 78, 543 ?
j) o. 001 . 26 -:-- o, 003 4 + 2, 5 3
k) 54, 3 -:-- O, 03
3
Ginasial - 1968 Resposta: - -
7
Ginasial - 1967
Resposta: a) 175, 600; b) l , 010; c) L 49; d) L 642;
p) O, 096; f) 19, 72; g ) 35, 19; h) O, 28; i) O, 059; j) O, 42; 9) Calcule o valor da expressão seguinte, escrevendo
k) l, 810 u seu resultado sob a forma ae número decimal:

5) Efetuar: 9
3, 07 - O, 07 X 2 + -- - O, 387
2
a) 12, 005 - 0,-006 -:-- 4 + O, 5 X O, 3
b) O, 05 X O. 04 + 8 -:-- O, 002 - O, 9 -:-- ·30 Resposta: 7, 043 , 1.E. e C. Dutra - 1968
c) 17,4 X 0,25 - 0,016 -:-- 8 +
0,000 . 5 X 4 -
- O, 7 X O, 5 10) Calcule:
d ) 3, 2 - O, 04 -:-- O, 2 +
3 X O, 5

Resposta: a) 12, 153. 5; b) 3, 999. 972; c) 4; d) 4, 5 a) 10 - .


(
0"2727 ... + - .18- - )
- 3 X 2
11
2 9 l
6) Dividir 8, 04 por 7 prolongando a divisã o a té
quatro casas decimais no quociente. :t:m seguida dê o valor (
- - +- -)-:-- 7 -:-- O, 45 s+ - -xz
15 20 3
Ido resto .
1
Resposta: a) O (zero); b) 2 - - -
Resposta: L 148 . 5 - quociente
3
O, 000.5 - resto Colégio Pedro II - 1968

240 Pp,ulo Peoo11. Matemática para o Admissão 241


i Í) Preencha as iacuna8:
a ) 4, 503 + * = 13, 04
14) Convertér em déeimal as fra~õês:
b) * - 5, 879 =1,111 9 7 5 8 71 7
e) 4 X O, 01 - O, 003 + * = O, 01
d) 30, 007 --' * = 19, 5 - : - -; - - ; - -; - - e --
e) * -7- O, 008 = 4 20 IS 12 21 80 9
f) 96 -7- * ·= 600
2 Resposta: 0,45; O, 46?6 . . . ; O, 41 (6); O, (380952); O, 0875
g) - - + O, 5 ~ * = O, 77 I 0, 7
5
h) 2934 -7- * = 2, 934 l 5) Conver ter em decimal os números mixtos:
i) 1, 967 X * + 1.967
3, 06 -7- * 7 5 5 7
j) - - - - = 17 2 - - ; 5 -- ; 1 -- e 3 - --
3 15 6 12 9
Resposta: a) 8, 537; b ) 6, 99; c) O, l; d) 10, 50'/ ; e)
O, 03~; l) O, 64; g) U, 03; h) (), 001; i) 1.000; j) O, 06 Resposta: 2, 4 (6); 5, 83; l, 41666 , . . ; 3, 777, ..

12) Complete as igualdades escrevendo no lugar das 16) Converter em fra ção ordiná ria o decimal O, 4 7575 ...
letras o nú,mero que satlsraz cada uma respectivamente:
471
a) 1.964 = 360, 8 X a + 160 Resposta :
b) 1.126,80 = b X 187,8+O,10
C) b X 2, 16 +
UH6 =
2, 68 990 Liceu Nilo Peçanha - 1967
d) a X 4~ 1 =
42, lU 17) Converter em fração ordinária os decimais:
e) o, 648 = 26 X o, 024 + b
a -7- 0, 4
f)
o, l
=
102, 7
7, 3; 9, 47; 19, 211 e O, 4739
73 947 19211 4739
Resposta: - - ; - - - - - - e _ __
Resposta: a) 5; b) 6; c) O, 4;, d) O, 10; e) O, 024; · f)
10 100 1000 10.000
4, 108
13) Converter em decimal as frações ordniárias: 18) Converter em frações ordinárias irredutíveis os
ele cimais:
14 46 15 15 2, 12; 0, 418; 7, 36 e 34,4512
a) ; b) - - ; c) - - e d) - -
30 24 18 8
3 209 9 282
Resposta: a) O, 46; b) l, 91 (6); c) O, 83; d) 1, 875 Resposta; 2- - : 7~e34
25 500 25 625
24Z P&ul9 Peijsoa
rr~~lem~ 4e .. 11tom•Hlllll p$l'~ ~ ~1!mj9P4o 24~
O, 4 (6); 1, 916; 0.8333 ... ; O, 6 (3); 3, 2 (7)1); :ZZ) esol ver ti expressa ó
19) Converter em frações ordinárias irredutíveis os
O, 75 O, 3636... 1
decimais;
0.06
X - 4,-2424
-- ...
----
14
1, 5; O, 7: O, (12); 1, 873
2, (27); Resposta: 1
3 5 7 4 97
Resposta: 2 - - ; 1 - -; - - ; - - ; 1 23) Resolyer:
11 9 9 33 111 a) l, 75 --;- O, 833 . . . +
O, 09 X 10
20) Converter em frações ordinárias irredutíveis os,
decimais:
b) 3, 411...
2, 222 ... .
+2, 333... 7 X O, 333 ... -

Resposta: a) 3; b) 1, 3
O, 4 (•6); 1, 916; 0,8333 ... ; O, 6 (3); 3, 2 (71);
. 24) Ditribuí~am urna garrafa de leite da seguinte ma-
9. 165; 2. 34 (5). n erra: Em uma x1cara O, 25; em outra menor O, 125 e o resto
7 23 5 19 269 m 25 canecas iguais . Que quantidade de leite foi posto
Resposta: - - -; - - - ; 3 - -; m cada caneca?
15 12 6 45 990 Resposta: O. 025.
82 2111 311
----e - - -- 2 - -- 25) Dois irmãos bebem diàriamente O, 75 e O, 8 de umcr
495 90ü 900 ga~rafa de ~eite, P,ºr dia. No fim de quantos dias o que bebe
21) Dizer sem efetuar, a natureza dos decimais gera- mms Por d10, tera bebido mais 10 garrafas do leitte que G
outro?
dos pelas frações:
25 9 21 5 8 8 Resposta: 200 dias.
a ) - ; b) - - ; c) - -;d) - ; e ) - ; f) - -;
147 20 15 12 14 21 . 26) Quem pagou 0,15 de sua dívid<:I· por mês, quanto
153 82 311 25 36 amda deverá na oitava prestação paga?
g) ; h) - - ; i) - -; j) - - ; k) - - Resposta: Não deverá nada. Terá pago O, 20 a mais.
240 495 900 15 144
Resposta: a) Periódica simples . 27) Qual a fração mais simples equivalente a O, 9166 ..•
b) Decimal exato ou dízima finita . cujos têrmos têm para m. d. e. 6?
e) Dízima periódica composta. 66
d) Dízima periódica composta. Resposta:
e) Dízima Periódica simples. 72 Colégio Pedro II - 1968
f) Dízima periódica simples.
g) Dízima finita. 28) A diferença de dois números é 19, 35. Se diminuís-
h) Dízima periódica composta. semos cada um dêles de 6 unidades o maior ficaria o quá-
i) Dízima periódica composta . druplo do menor. Qual o maior dos dois números dados
j) Dízima periódica simples , inicialmente?
k) Decimal exato . Resposta: 31, 80 l. E. e C. Dutra - 1968

Problemacs de Matemática para o Admissão 245


SISTEMA MÉTRICO DECIMAL

Chama-se sistema métrico decimal, um: conjunto de uni-


d ides de diversas espécies que são todos derivados de
111110 unidade principal de comprimento, chamado metro e
• 11jos múltiplos e submúltiplos se obtém multiplicando e divi-
d 111do a unidade principal pelas potências de 10.

llNlDADE DO COMPRIMENTO
A unidade principal de comprimento é o metro (m), que
i' igual à décima-milionésima parte do quarto do meridiano
lt rrestre, isto é, a distância do Pólo ao Equador, contada
1 • bre um meridiano.
PO LO NORT E
p

pi
P O LO SU L

Problemas de Mate mática pa ra o Admis são 247


MúLTIPLOS DO METRO um deslocamento da vírgula para a direita ou esquerda da:
posição em qus (a vírgula) se achar antes da conversão.
Decâmetro. O prefixo deca, quer dizer dez; por isso o
'decâmetro (dam) têm 10 metros. Assim, esquematizando o que foi dito:
Mm km hm dam m dm crn mm
Hectômetro. O prefixo hecto, quer dizer cem; por isso (desuso)
o hectôm'etr.o (hm) têm 100 metros. l-/--[-/-1--1-1-
Kilômetro. O prefixo kilo quer dizer mil; por isso o Se tivermos:
kilômetro (km) têm 1 . 000 metros. 19, 47 m para converter em milímetros, por exem-
plo, vemos que a vírgula indicativa, no caso, ds metros deve
Miriâmetro, (em desuso). . . O prefixo Mír.ia quer dizer andar três casas para à direita. Como depois dela só existem
dez mil; por isso o miriâmetro (Mm) têm 10 . 000 metros. duas casas•, teremos que criar outra, o que se faz por inter-
médio de acréscimo de zero e que será a casa dos milímetros.
SUBMÚLTIPLOS DO METRO
Então:
Decímetro. O prefixo deci quer dizer décima parte; 19, 47 m = 19 . 470 mm .
nor isso o decímetro (dm) é a décima parte do metro ou se:a
O, 1 do metro. Do mesmo modo, se desejássemos converter os 19, 47 m
m kilômetros, por exemplo, olhando o esquema acima veri-
Centímetro O prefixo centi quer dzier centésima parte; fica-se que basta apenas deslocar a vírgula três casas para
por isso o centímetro (cm) é a centésima parte do metro ou o esquerda e do mesmo modo s·e rá necessário criar uma
seja O, 01 do metro. terceira casa para o heetômetro e uma quarta para o kilô-
metro .
Milímetrc;>. O prefixo mili, quer dizer milésima parte;
por isso o milímetro (mm) é a milésima parte do metro ou Assim:
beja O, 001 do metro.
19,47 m == 0,019.47 km.

CONVERSÃO Quando temos vários comprimentos para sornar, subtrair


multiplicar ou dividir, é indispensável que todos estejam
Depois do que foi dito, concui-se fàcilmente que a con. referidos à mesma unidade.
versão de um múltiplo ou submúltiplo; em submúltiplo ou Se tal não acontecer, torna-se necessário convertê-los
múltiplo depende apenas de uma multiplicação ou divisão todos e depois somá-los, subtrai-los, multiplicá-los ou divi-
por uma potência de dez di-los.
Não obstante, se considerarmos escritos em uma linha Nos dois primeiros casos o resultado da operação terá
horizontal por exemplo, os múltiplos à esquerda do metro e o mesmo nome que o das diferentes medidas somadas ou
,os submútiplos à direita, tôdas as conversõea se reduzem a subtraídas. É o que acontece quando desejamos calcular

2.43 ranlo Pessoa Problemas de Matemática para o Admissão 24!9


Seu perímetro será representado por 2p e é igual a:
por exemplo perímetros ou semi-perím etro ou comparar, poi'
diferença, dois comprimentos, por exemplo .
2p = a +a +a +a = 4a e o
4a .. Perímetro de uma figura plana é a soma dos lados quo
Semi-pe ímetro: p = - - - = 2a a delini.itam .
.2 Semi-perímetro é a metade do perímetro .
Retângulo. Figura plana de quatro lados, iguais dois a Figura plana é tôda aquela qu e pode ser desenhada
dois; perpendiculares, como no quadrado, e tendo os lados cm urn plano .
apostos iguais e paralelos.
Plano é tôda superfície sôbre a qual pode assentar com-
Na figura: pletamente uma reta em tôdas as direções. É .gerada (a
·uperfície plana) por uma linha reta que se desloca parale-
lamente a si mesma.
A a B
,~~~~~~~~- !

Linha reta é o caminho mais curto entre dois pontos.

d' 1
Ih
1
Consideraremos apenas as seguintes figuras planas:

Quadrado. Figura plana de quatro lados iguais, per-


D e e pendiculares dois a dois e paralelas os opostos.
Os lados a e b; b e e; e e d e a e d, são perpe ndiculares,
Os lados a e e são opostos, iguais e paralelos, o mesmo Na figura:
acontecendo com os lados b e d.

Seu perímetro será representado por:

2p = a + b +c +d ou
A

dl
r- -,
[
a

lb
B

2p =1 a+ b +a +b ou
1 1
2p = 2a + 2b D
l
e
!
e
Seu semi-perímetro é:
Os lados a e b; b e e; e ·e d; a e d, são perpendiculares.
Os lados a e e e b e d, são opostos e paralelos.
Triângulo. Figura plana de três lados, que podem ser Os lados são todos iguais; por isso:
iguais ou diferentes ou ainda dois igua is e \JJJJ. d1.(erente.
a=b=c=d
2.5{) Pa ulo Pesso~
2.º caso: 2p t=. a + b + c e semi-perímetro:
a+b+c
TRlANGULOS
2
A A

3.0 caso: b = c e portanto: 2p =a+ b +c ou

b e a+ 2b ou a+ 2c
a +
2b a + 2c
p = -----
2 2
a
Vejamos alguns exemplos:

Calcular o perímetro de um quadrado que tem 3, 8 cm


de lado. Dar o resultado em metros .

Vimos qué o perímetro do quadrado é:


2p = 4a, sendo a o lado do quadrado.
e
Então o seu perímetro será:

temos em 1, um triângulo de lados iguais; na 2. um de 2p =4 X 3, 8 cm = 15, 2 cm


Jados diferentes e na 3, um com 2 lados iguais e um dife-
rente. O primeiro se chama equilátero; o segundo escaleno Como o resultado deve ser dado em metros, iremos con-
e o terceiro isósceles verter 15, 2 cm em metros e depois do que foi mostrado:

Em todos os casos seus perímetros são: 15, 2 cm = O. 152 m ·

Outro exemplo:
1.º caso: 2p = a + b + c, mas como a = b = c
teremos 2p =
a + a + a que dá: Calcular o perímetro e o semi-perímetro de um retân-
gulo de lados: 15, 3 dam e 149 km, dando a resposta em
2p = 3a e o semi-perímetro: hectômetros

Devemos converter as dimensões àadas à me.sma uni-


3a dade, que escolheremos o hectômetro, de vez que a resposta:
p= neve :;;er dada em hectômetros .
2

252 Pau lo Pesi::;oe


T remoi:
o que nos obrigou a criar com o auxilio do zero, a tM-
15,3 dam =·
1.53 hm é ceira casa .
149 km = 1.490 hm Calcular o semí-perímetro de um triô:ngulo da lados:
ia = 12 hm; b = 20 km e c = 2. 018 dam, dando a resposta
Vimos que o perímetro do retângulo é: em cen tímetros.
2P = 2a + 2b; a e b sendo os lados da figura . De início vamos converter as dimensões dos lados a
icentímetros.
Então:
Vem :
2p 2 X L 53 hm +
2 X l. 490 hm ou
2p 3, 06 hm + 2.980 hm ou a -= 12 hm = 120 .000 cm
2p = 2.983, 06 hm. b = 20 km= 2.000 . 000 cm
c = 2.018 dam = 2.018 . 000 cm
O semi-perímetro será:
Seu perímeh·o será.:
2a + 2b
p = - - -- - 2p = 120.000 cm+ 2.000 . 000 cm= 2.018 . 000 cm
2 ou
2p = 4.138.000 cm
Como já calculamos 2a + 2b = 2 . 983.06, teremos:
O semi-perímetro será:
2.983, 06 hm
p = - - -- - - 1.492, 53 hm 4. 138 . 000 cm
2 p= - -- - - -- = 2. 069. 000 cm
2
Finalmente consideremos o caso dos triângulos·
Calcular o perímetro de um triân<Jl!lo eqüilátero de 4, 5 Unidade da superfície
dm de lado, dando a resposta em decametros .

Vimos que o perímetro do triângulo eqiiilátero é; A unidade principal de superfície é o metro quadrado
(m 2) isto é, a área de um quadrado de um metro de lado .
2p = '3 a, a sendo o lado do triângulo .
Seus múltiplos, de acôrdo com a significação dos prefi-
Então: xos, dada quando tratamos da unidade de comprimento, são:

2p = 3 X 4, 5 dm = 13, 5 dm Decâmetro quadrado (dam~ - 100 rn~)


Hectôrnetro quadrado (hm~ - 10. 000 rn~)
Como desejamos o resultado em decâmetros devemo~
Kilôrnetro quadrado (km 2 - 1. 000. 000 rn2)
converter 13. 5 dm em decâmetros e pelas razões mostradas:
Miriâmetro quadrado (Mm~ - 100.000.000 m 2 em de-
pasta mover o; vírgula três ççrsc;tfi pqrc;t Q: E}sg;uerda, O, 135 d~ suso).

Probl em&s de Matemática pa.ra o Admissão 255


unida de agrária a re (a ), que tem pm a múltiplo o Hectare
Os submúltiplos do metro quadrado são: (ba ) e para s ubmúltipld o centíare (ca).
Decímetro quadrado (dm2 - O, 01 do m 2) Organizado um esquema nos moldes d o q ue foi estabe-
Centímetro quadrado (cm2 - O, 000. 1 do m 2 ) lecido para o metro e o metro quadrado, embora sendo
Milímetro quadrado (mm2 - O, 000.001 do m 2) uma unidade de superfície o deslocamento da vírgula para
a direita ou esquerda f]e fará como no caso do metro linear,
As conversões podem ser feitas usando-se o mesmo isto é, de uma casa em uma casa.
esquema utilizado para o metro, mas o deslocamento da
vírgula se fará para a direita ou esquerda, de duas em duas A equivalÉh1cia entre as medições de qualquer sup erfície
-casas. feita em m etros quadrados, seus múltiplos e submúltiplos,
Para as mesmas medições feitas em unidade agráâ a, ou
Mm2 Km2 dm2 dam2 m2 dm2 cm2 mm2 vice-versa, se faz através das relações:
1 1 1
1 1 1 1 1
1 hm2 =
1 ha
Se tivermos por exemplo: l dam2 = 1 a
1 m~ = 1 ca
1967, 341 m 2 para· converter em milímetros quadra-
Como se disse anteriormente, quando temos dois com-
dos, por exemplo, vemos que a vírgula indicativa, no caso,
primentos para muliiplicar, é necessário que as unidades
de metros quadrados, deve andar tres casas duplas ou sejam
Jlejam as mesmas e no caso de não serem, é indispensável
seis casas, para a direita. Como depois dela so existem três
convertê-las à mesma unidade e o produto delas represen-
:casas, o que se faz por intermédio do acréscimo de zeros
tará uma superfície.
(3 zeros) e que irão: o primeiro Para completar a dos cen-
tíme1ros quadrados e os dois últimos a dos milímetros qua- Assim pàra calcularmos a área de um quadrado é bas-
drados. tante elevar ao quadrado o comprimento do lado.
Por isso a área àe um quadrado de l, 5 m de lado é:
Assim:
1967,341 m 2 = 1.967.341.000 mm2 S = (1, 5 m) 2 = 1, Sm X l, Sm3
Se quisermos calcular a área de um retângulo de 15 cm
Do mesmo modo se desejassemos converter 1967, 341
de largura por 3, 8 dam de comprimento, teremos que redu-
m 2 em quilômetros quadrados, por exemplo, olhando-se o
zir as dimensões dadas à uma mesma e efetuar o produto
es,quema acima verifica-se que é bastante oes1ocar a vírgula delas, uma vez que a área do retângulo é igual ao produto
da posição em que se encontra, seis casas para a esquerda, dos números que medem a base e a altura, isto é:
para o que é bastante p reencher as casas dos hectóme-
tros quadrados, colocando-se dois zeros à esquerda do núme- S = (1, 5 m)2 = 1, 5 X 1, Sm = 2, 25m 2

:ro e mais a casa dos quilômetros quadrados, que será cons-


tituída do zero seguido da vírgula. vamente (comprimento e largura).

1967, 341 m 2 = O, 001. 967. 341 km2 Então, depois do que foi dito teremos inicialmente, se
q uisermos obter a área em centímetros quadrados:
As superfícies medidas pelo metro quadrado, seus múl-
tiplos e submúltiplos também podem ser avaliados Pela
3, 8 dam = 3800 cm

256 P1wlo ressoa


I'roblemas de Ma t emáti ca pa.ra o Admi ssão 257
F.nt - o a á rea se rá:
1 o mesmo modo se desejassemos converter 28953.41 m 3
S = 15 cm X 3800 m u 111 q111lômetros cúbicos, por exemplo, olhando-se o esquema
S = 57 . 000 cm~
" ' 1111u , verifica-se que é bastante deslocar a vírgula da posi-
11 n c·m que se encontra, para a esquerda nove casas, para
Unidade do volume •, , 111 é bastante completar a casa dos decâmetros cúbicos,
'•> 11 1 um zero; preencher a casa dos hectômetros cúbicos
A unidade principal de volume é o metro cúbico (m~). ' 'll ll tr ês zeros e mais a casa dos quilômetros cúbicos, que
is to é, o volume de um cubo que tem um metro de aresta. , 'I <'t ·ons tituída de um zero seguido da vírgula.

Os múltiplos do me iro cúbico são: /\:;sim:


Decâmetro cúbico (dam:: - 1000 m:1) 28 . 953. 41 m 3 = O, 000. 028. 953 . 41 kmª
He:::tômetro cúbico (hm~ - 1. 000. 000 m 'l)
Quilômetro cúbico (km' 1 - l . 000 . 000 . 000 m~) Os volumes medidos pelo metro cúbico, seus múltiplos
Miriâmetro cúbico (des.) (Mmª - 1. 000. 000. 000. 000 m :•) ,ubmúltiplos também podem ser avaliados pelo litro (1),
O s submúitiplos do metro cúbico são: • 11 11 múltiPlos e submútiplos, que também é unidade de
Decímetro cúbico (dm~ - O, 001 do m 3 ) volt1mc e cujos múltiplos são:
Centímetro cúbico (cm:: - 0,000.001 dom~)
Milímetro cúbico (mm'1 - O, 000. 000. 001 do m~ ) Decalitro (dal - 10 litros)
1 [cctolitro(hl - 100 litros)
As conversões podem ser .feitas usando-se o mesmo C) uilolitro (kl - l . 000 litros - em desuso)
esquema utilizado para o m etro, mas o deslocamento da vír- Miria litro (Ml - 10. 000 litros - em desuso)
gula se fará para a direita ou esquerda de três em trés
casas. • < ltJOS submúltiplos são:
Mm ': l<m '1 dm'l dom~ m:) dm3 cm::
Decilitro (dl - O, 1 do litro)
-'-"l~__,_l~~~
l ~__,_l~___,l~~~
I ~- 1 Centilitro (cl - O, 01 do litro)
Se tivermos por exemplo 28953, 41 m~ para convertemos Mililitro (ml - 0,.001 do litro)
em milímetros cúbicos, por exemplo, vemos que a vírgula
~ndicativa, no caso, de metros cúbicos, deve andar três casas
Organizado um esquema nos moldes do que foi esta-
l 11 lt cicl o para o merto; metro quadrado; metro cúbico, embo-
triplas, ou sejam nove casas para a direita. Como depois
dela só existem duas casas, teremos que criar mais sele 111 11endo unidade de volume, o deslocamento da vírqula
1 • 11 u a direita ou .e squerda, se fará como no caso do me'lro
casas, o que faremos por meio de acréscimo de zeros (7 zeros)
/111l'CTI', isto é, de uma i:::asa em uma casa.
e que irão : o primeiro completar a casa dos decímetros cúbi-
cos; os três seguintes a dos centímetros cúbicos e os três
A. equivalência entre as medições de qualquer volume
últimos, a dos milímetros cúbicos .
11 11<1 e m metros cúbicos, seus múltiplos e s ubmúltip los, para
Assim: •1 Ili .>mas medições feitas em litros ou vice- versa, se faz
11'111vf.s da relação:
28953, 41 m '1 28 .953 . 410.000 . 000 mm'i
1 litro = 1 dm3
2,53 P a.ulo P ess on
Calculemos agora o voiume de um para1elepípedo re.
Multiplicando-se três comprimentos, expressos na mes· twgulo, que é um sólido com a forma de uma caixa de
ma unidade, o produto dêles representará um volume em Jósfor.os, por exemplo .
qualquer dos múltiplos ou submúltiplos do metro cúbico.
PARALELEPIPEDO RETANGULAR
Assim para calcularmos o volume de um cubo que é
um sólido com a forma de um dado, é bastante elevar ao
VERTI CE DO
cubo o comprimento da aresta do cubo, que são iguais. PA RALELEPIPEDO

CUBO ARESTA DO
'PARALELEPIPEDO
YERTICE DO Cl!00

ARESTA DO
PARAL ELE PIPEDO

AREST.•~ C•O CU80

O parelepípedo tem seis faces iguais; duas a duas, isto


' . os opostos são iguais. Tem doze arestas, iguais quatro a
quatro, isto á, as arestas opostas são iguais.

Tem oito vértices.

Seu volume é dado pelo produto, de suas arestas, a, b e


que são: comprimento, largura e altura do parale~epípedo,
~ia figura e por isso, o volume do paralelep1pedo e:
O cubo tem seis faces iguais; tem doze arestas iguais e
tem oito vértices. V= abc
Se chamarmos V o se u volume, êle será igual a Posto isto, calculemos ~ volume de um paralelepípedo
V = a 3
,, sendo a a sua .arestç: . ujas arestas são: a = 3, 5 cm; b =
O, 4 me c = 30, 5 mm,
dando a resposta em litros .
Seja, depois disso calcular o volume de um cubo de
2, 5 cm de aresta, dando a resposta em litros.
Precisamos, como já foi dito, reduzir ou converter as
<l1mensões das arestas à mesma unidade, que poderá ser
Teremos: · ntímetro (cm), por exemplo.

V = (2, 5 cm)3 = 15, 625 cm3 ou Teremos:


O, 015.625 dn1=~ ::.:: O, 000.015.625 in;J 0, 015.625 a = 3, 5 cm; b = 40 cm e c 3, 05 cm
litros.
Probl em"'s de Matemáti ca pitra o Admissão 261
2·60
Depois disso vem:
As conversões podem ser feitas usando-se o m esm o
V = abc ou 11 qucma empregado para o metro e o de3loca mento da vír-
V = 3, S cm /, 40 <
' JJl 3, 05 cm ou qt ilCT s e fará para a direita ou esquerda, d 2 casa em ca sa .
V :_ 427 c m :: '. O. 427 dm " = O, 427 litros .
1111 o n1 Mg kg hg daj o dg cg ing
Unidades de massa ' 1 1 1 1 l_~
-1-~l-~1.__I
Usua lmente emprega.-se indistintamente as palavras Se tivermos:
pêso e massa, quando se quer designar a massa de um
corpo. Convém, entretanto, diferençar uma caixa da outra. 2567, 8 hg para converter em miligramas, por exem-
Péso d e um corpo é a fôrça que o atrai para o centro plo, vemos que a vírgula dave se deslocar para a direita,
.tla terra . Depende do ponto da terra em que se encon,ra e 1·1nco casas e como já existe uma; constituída pelo algarismo
,de si mesmo. · H. ó bastante acrescentar ao número dado, sem a vírgula,
Iv1assa ae um corpo, inàepende do local da terra em que qua tro zeros.
se encontra. Quando se usa a balança mede-se a mas!;a
uo corpo. Então:
A operação de medir a massa do corpo tem o nome d.- 2567, 8 hg 256780000 mg
pesada do corpo, dai a razão do uso indistinto das palavras
peso e massa, muitas vêzes erradamente. Do mesmo modo, se desejáS'semos converter 2567, 8 hg
11m toneladas, o esquema nos diria que é bastante deslocar
Os múltiplos da unidade de massa que é o quilograma o vírgula para a esquerda da posição em que se encontra,
(k9) são: 1 111 co casas.

Quilograma (k9 - l . 000 9ramas) Como o número dado só tem quatro algarismos antes
'1 1 vírgula, será necessário colocar-se um zero à esquerda
Miriagroma (Mg - 10. 000 gramas - em desuso) d número, precedido da vírgula .
Quintal métrico (Qm - 100. 000 .gramas)
Assim:
Tonelada métrica (tm · 1. 000 . 000 gram as)
2567, 8 hg = O, 25678 toneladas
Os submúltiplos são:
lJm grama (g) é o volume de um ce ntímetro cúbico de
Hectograma (hg - 100 gramas) < rq 1ia destila da a 4º centígrados e à pressão atmosférica de
Decagrama (dag - 10 gramas) '/()() mm da coluna do mercúrio.

Grama (g - 1 grama) ta 4° centígrados que a água destilada tem o se u menor


vo lume, o que significa que, em tais condições, cabe a maior
Decigrama dg - O, 1 do grama) 'p11rn lidade possível de água dentro do volume de um centí-
Centigrama (cg - O, 01 do grama) 111<>11o cúbico.
Miligrama (mg - O, 001 do grama) Trntan<ld-se pois, de água destilada, n:a s condições
111 1mrr, podemos estabelecer a s seguintes relações, que ser-
')62
a.' Pa u lo Pessoa
virão para medir o pêso de diferentes volumes de água Unidacle ele tempo
destilada:
A Unidade de tempo é o segundo (s).
1 cm3 = 1g
Seus múltiplos são:
1 dm3 = 1 litro = 1 kg
1 m3 = 1 tonelada O minuto; a hora e o dia.
Não tem subdivisões com designação própria e por isso
Se o pêso a determinar fôr de qualquer outra substân- 1se tivermos, por exemplo, 0, 2 segundos ou O, 03 segundos,
da homogênea que não a água destilada, torna-se necessá- diremos: dois décimos de segundos ou três centésimos de
rio conhecer a massa específica ou densidade da substância segundo .
homogênea e que é a relação entre a sua massa e a unidade Não faz parte do sistema métrico decimal pois a trans-
de volume da substância. formacão de dias em horas; horas em minutos e minutos
A sua unidade legal é o grama Por centímetro cúbico, em segundos não é feito pelo simples deslocamento da vír-
que se representa por g / cm3 . gula, mas sim, através da relação que liga uma unidade à
outra.
Assim:
Medidas efetivas dia = 24 horas
hora = 60 minutos
Unidades efetivas são instrumentos que servem efetiva- minuto = 60 segundos
mente para a operação de medir.
É o que se chama um número complexo.
Assim é que, para medir comprimentos encontramos o Se q°i.:isermos saber, por exemplo, quantos segundot.
metro; o meio metro; o duplo-decímetro; o decímetro; o tem um dia; empregaríamos o seguinte método:
duplo-decâmetro; o decâmetr.o e o meio decâmetro, consti- Um dia tendo 24 horas e cada hora 60 minutos, segue-sb
tuindo os três últimos as correntes de agrimensor. que um dia terá 24 X 60 = 1440 minutos .
Um minuto tendo 60 segundos, os 1440 minutos existentes
Para medir capacidades encontramos: o hectolitro; o
meio bectoiitro; o duplo-decalitr.o; o decalitro; o meio-decali-
ei."!1 um dia, corresponderão à 1440 X 60 = 86 . 400 segundos,
que é o total de segundos contidos em um dia.
tro; o duplo-litro; o litro; o meio litro; o duplo decilitro; o
decilitro; o meio decilitro; o duplo~centili tro e o centilitro. Poderíamos dispor as operações feitas como se segue:
l dia
Para medir massas, uma série de pêsos de metal (ferro
fundido, latão, prata ou platina) com as forr.ílas de troncos X :l4 horns
de pirâmide; cilindros ou simples lâminas de massas dife- 24 horas
rentes, variando do meio quintal ao miligrama. (n.b> de minutos por hora) X 60 minutos
Os protótipos do metro e do grama, unidades fundamen- 1 . 440. minutos
tais de comprimento e massa, são de platina iridiada e se (n." de segundos por minuto) X 60 segundos
acham guardados no Pavilhão de Pêsos e Medidas de Bre-
teuil, na França. 86 . 400 segundos

264 !'!Lulu Pe.s sva 265


?.) Efetuar as seguintes mudanças de unidades:
Se desejarmos saber quantos segundos existem em
3 h. 15 min. 24 s., procederíamos como procedemos acima, a) 3547,213 m~
is to é: b) O, 395 mm2
c) 37 km~
3 h. d) 1958, 3 dm~
X 160 m
Como vimos na ocasião em que ensinamos conversão,
180 m e como tem 15 m. vem:
somamos + 15 m
/1espostas:
195 m
X 60 a) 3 . 547, 213 m 2 =
354 . 721, 3 dm2 = 35.472.130 cmº
-= 3.547 -213 . 000 mm~ ou
11. 700 s . e como tem 24 36, 472.18 dom"= O, 354.721.3 hm 2 = O, 003 . 547.213 km ~
+ 24 s. b) O, 395 mm~ = O, 003. 95 cm2 = O, 000. 0395 dm' =
-= O, 000. 000 . 395 m " = O, 00 000 000 395 dam2 = ......... .
11. 724 s. ....::: O, 000.000.000.039 . 5 hm" =
O, 000 .000 . 000 . 000 .395 km:.!
e) 37 km~ = 3. 700 hm2 =
370 . 000 dam' = 37. 000 . 000
Asim, em 3 h. 15 m. e 24 s., existem 11 . 724 segundos . m~ ~ 3.700.000.000 dm" = 370-000.000.000 cm' = ..... .
= 37. 000.000.000 . 000 mm~
d) l.958, 3 dm2 = 195 .830 cm'1
=19 . 583 .000 mm2 =
,_ 19, 583 m ' ·= O, 195.83 dam2 = O, 001.958.3 hm~ =I ....
EXERCíCIOS RESOLVIDOS = O, 000.0, 19.583 km2

1) Efetuar as seguintes mudanças de unidades:


@ Efetuar as seguintes mudanças de unidades:
a) 347 km 2 em ca
a ) 19, 67 m b) 19, 375 dm" em ha
b) 29.873 mm e) 3. 051 m~ em ha
c) 153. 42 1 dam d) O, 007 dm" em a
X e) O, 003 cm' em CC'l
De acôrdo com o que foi ensinado, vem: .< fl 39 m 2 em~ bc 4
g) . 14589, 3 mm ~ em a
Resposta: a) 19. 57 =· 196, 7 dm = 1967 cm = l., 967
dam = O, 1967 hm = O. 01967 km As relações entre o metro quadrado, seus múltiplos e
s ubmúltiplos, e o are com seus múltiplos e submúltiplos são,
b) 29.873 mm = 2.987, 3 cm = 298, 83 dm = 29, 873
como já dissemos:
m = 2, 987. 3 dom = O, 298. 73 hm = O, 029 . 873 km
e) 153, 421 dam :=1 1.534 • .21 m = 15 .342.1 dm =
ha = hm~
a = dam~
= 153 .42 1 cm = 1.534.210 mm e 15, 3421 hm = 1, 534.21 ca = m "
km

'J66
1"!'111!9 f' illllU · l ' t::J:::iUB
P i ' o bl t~ l n ns
Entoo vetn:
e) 1.347, 391 -2 hm~ = L 347.391.2 km3 _ 1.347.391, 2
R.espostas: dam:J = 1. 347 . 391.200 m:~ =134 . 739.120.000 dm'1 = . . ... .
=
-= 134. 739 . 120 . 000 . 000 cm3 134 . 739 .120 . 000 . 000 . 000 m m ·1
a) 347 km 2 = 347.000 . 000 rn 2 = 347 . 000.000 ca
1 .835,891 mm~ = 1,835 . 891 cm 3 =
b ) 19, 375 dm 2 = O, 000 .019.375 hm 2 = O, 000 .019 . 375 lia
- O, 00 1.835.891 dm3 = O, 000 .001.835 . 891 m'1 =
e) 3, 051 m " =· 3, 051 ca = O, 000 . 305.1 ha
d) O, 007 dm 2 = O, 000 .000 . 7 dam2 = O, 000. 000 . 7 a = O, 000.000.001.835 . 891 dam8 =
- O, 000 .000 .000.001.835.891 hmR =
e) O, 03 cm" = O, 000.003 m~ = O, 000.003 ca
= O, 000.000.000.000 . 001.835 -891 km3
f) 39 m 2 = 39 ca = 13, 9 dca
g) 14.589, 3 mm" = O, 000 . 145 .893 dam~ = g) 1-235 dm3 = 1 1.235.000 cm3 = 1.235.000.000 mm:1 =
O. DOO. 145 . 893a. = L 235 m 3 = · O. 001 . 235 dam 3 = O, 000 . 001. 235 hmx =
= 0,000 . 000 . 001.235 km3
4) Efetue as seguintes mudanças de unidades:
5) Converter nos diferentes múltiplos e submúltiplos:
a) 13 m 3
b) O, 519 cm3 a) 1937 1
c) 3. 845 dam3 b) 270 dl
d) 18. 354, 9 hm~ e) 1, 437 cl
e) l. 347, 3912 hm3 d) 153, 98 ml
f) 1.835, 891 mm3 e) 2 . 349, 57 dal
g) 1.235 dm3 f ) 58. 324. 9 hl
g ) 29, 347 kl
De acôrdo com o que foi ensinado:
Como já foi dito:
a ) 13 m 3 ,.......:. 13 -000 dm3 = 13.000.000 cm3 .. . .. .
= 13 .000.000.000 mm 3 = O, 013 darn3 =O, 000 . 013 hm': = Hcspo .-: tas:
= O, 000.000.013 km 3
a) U3 37 1 = 19.370 dl = 193 . 700 cl = UJ 37 .000 ml
b) O, 519 cm3 = 519 mmª =
O, 000. 519 dm 3 =
= =
:=: O, 000 000 519 m = O, 000 . 000 . 000.519 dam 3
3 = : :: : : ::: b) 270 d l = 2 . 700 cl = 27. 000 ml
=
271 2, 7 dal =-=
= O, 000. 000 . 000. 000 . 519 hm3 =
O, 000.000.000. 000 . 000 . 519 O, 27 hl O, 027 kl
km 3 e) L 437 cl = 14, 37 ml = O, 143. 7 dl = O, 014 . 371 = · . ..

e) 3, 845 dam3 = 3.845 m ª = 3.845.000 dm 3 = ..... .. .


-= 0,00I . 437dal = 0,000 . 143.7hl = 0,000 . 014 .37kl
3 1 d) 153, 98 ml = 1, 539.8 cl = O, 153 . 98 dl = O, 015 . 398 l
= 3.845.000.000 cm = 3.845 . 000 .000.000 mm' =
= 0, 001.539 .8dal = o.000.153.98hl = 0,000 . 015 .398 kl
O, 003.845 hm3 O, 000 . 003 . 384. 5 km3
e) 2. 349, 57 dal = 234, 957 hl = 23, 495. 7 kl = 23. 495, 7
d) 18 . 354, 9 hm6 = 18, 3549 km 3 = 18. 354 900 da m': = l = 234. 957 dl = 2. 349. 570 cl = 23 . 495 . 700 ml
e== 18.354.900.000 In3 =
18 .354 . 900.000.000 dm~ = . .... .
f) 58. 324, 9 hl = 5. 8324, 49kl = 583. 832, 249 dal=
= 18.354.900.000.000.000 cm3 = 18. 354. 900. 000. GOO. 000 . 000 5.832.490 1 = 5 . 832.490.0 dl = 583.249 .COO cl =
mm 8
5.832 . 490 . 000 ml

263 P a ulo P ewsoa


;

./
b) 37.491, 183 dcg = 374.911. 83 cg -= 3.749.11 8, 3
6) Reduza: mg ~ 3 .749, 118.3 g = 374, 911.83 dag -= 37, 491.183 hg
a) 1937 1 a cm ~ · 3, 749 . 118.3 kg = O, 374 . 911.83 Qm = O, 037 . 491 . 183
b) 270 dl a dm3 toneladas.
c) 1, 437 cl a dam3 c) O, OD3. 58 kg = O, 035 . 8 hg = O, 358 dag = 3, 58
d) 153, 98 ml a m ª g = 35, 8 dag = 358 cg = 3.580 mg = O, 000 .035.8 Qm
e) 2. 349, 57 dal a hm:1 .....::: O, 000 . 003 . 58 toneladas.
f) 58 . 324, 9 hl a km::
g) 29, 347 kl a mm'J
d) 1.823.591 mg = 182.359, 1 cg = 18 .23S, 91 dcg = ..
= 1.823, 591g = 182,359 . l dag = 18, 235.91 hg = ... .... .
Vimos que 1 dm 1 = 1 litro. = l, 823. 591 kg = O, 001. 823 . 591 toneladas
Então: e) 134, 871 cg =
13498, 17 mg = 134, 9871 dcg = ..... .
= 13, 498.71 g = l.349.81dag = 0,134.981 hg = ....... .
a) 1 . 937 1 = 1. 937 dm 3 = 1. 937. 000 cmº = O, 013.498. l kg = O, 000 . 134 . 981 Qm = O, 000 . 0.13.498. l
b) 270 dl = 2 71 = 27 dm 3 toneladas
c) 1, 437 cl = O, 014.37 dm'J = O, 000.000 .014.37 dam';
f) O, 000.379 t = O, 003.79 Qm = O, 379 kg = 3, 79 hg
d) 153, 98 ml = : O, 153.98 1 = O, 153.98 dm''> = .. ..... .
0,000.153.98 m 3 = 37, 9 dag = 379 g = 3790 dcg1 = 37. 900 cg =
e) 2. 349, 57 dal = 23495, 7 1 = 23495, 7 dm'; = = 379.000 mg
= 23, 495 . 7 m 3 =O, 023.495.7 dam8 = g) 37, 29 hg = 3, 729 kg = _O, 037. 29 Qm = O, 003. 729
= O, 000. 023 . 495. l hm 2 toneladas e;=! 372, 9 dag = 3. 729 g = 37 · 290 dcg = 372. 900
f) 58. 324, 9 hl = 5832490 1 = 5832490 dm:: = . . , ... . . cg = 3 . 729. ODO mg
= 0,000.005.832.490 km ~
h) 425, 897 . 6 Qm = 42, 589. 76 toneladas = 42. 589,76 kg
g) 29, 347 kl = 29.347 1 = 29 . 347 dm~ = ......... ... . .
= 29. 347. 000. 000 mm3
= 425.897, 6 hg =· 4.258, 976 dag = 42. 589 .760 g = ..... .
= 425.897 .600 dcg = ' 4 . 258. 976 . 000 cg = 42 . 589. 760. 000
7) Converter nos diferentes múltiplos e submúltiplos: mg
a) l . 378, 43 g 8) Determinar o pêso dos seguintes volumes de água
b) 37. 491, 183 dog
destilada:
c) O, 003.58 kg
d) 1.823.591 mg a) l. 583, 7dm 3 em gramas
e) l. 349, 871 cg b) 37, 49 cm~ em quilogramas
f) O, 000. 379 toneladas c) O, 001 dm~ em toneladas
g) 37, 29 hg d) 147, 283 m:J em decagramas
h) 425, 897. 6 Qm e) 34, 81 1 em quilogramas
Respostas: f) O, 03 1 . 58 hl em hectogramas
1

a) l . 378, 43 g = l . 3784, 3 dcg = 137. 843 cg 1 . 378. 430 Sabemos que 1 cm'1 = 1 g e 1 m~ = 1 ton .
mg = 137, 843 dag = 13, 784.3 hg l, 378.43 kg = .. . . E 1 litro = 1 quilograma .
O, 137.843 Qm = O, 013.784.3 t

•)70 .. P1·obl en1a-s de Mutem{t.ticn pa.ra o Admissão 271


._ l'u1ilo l'<.::rn ua
Então: Cntão:
Respostas: 9 dam~ - 1. 6384 dam 2 -1- 10, 500 dam· : ...... .
17, 86 1.6 dam 2
a) 1. 583, 7 dm 3 =
1. 583 . 700 cm~ =
1. 583. 700 g
b) 37, 49 cmª =
37, 49 g = O, 037 49 kg 11) Efetuar:
c) O, 001 dm~ = 1 m~ = 1 tonelada.
d) 147, 283 m 3 =
147, 283 toneladas =
14'7 283 00 dag 45m:: ...;.- 160 m~ X 4 m X 200 m
e) 34, 81 l = 34, 81 kg
f) O, 031.59 hl = 3, l!:i9 l = 3, 159 kg = 31, 59 hg Podemos escrever:

9) Efetuar a soma:
X 4 m X 200 m. Como m 0 ...;.- m2 m,
O, 305. 04 km +
20. 005 dm + 23, 007 hm, dando a 160 m "
resposta em decâmetros. V m:
45 m 45m X 4m X 200m
Temos: X 4 m X 200 m = -----
160 H30
O, 305. 04 km = 30, 504 dam 225 m:1 depois de feitas as operações
20 . 005 dm = 200, OS dam
12) Dividir:
23, 007 hm = 230, 07 dam
7, 25 m" ' ...;.- 5 dam ~ .
Foi necessário como se disse anteriormente, efe tuar a
conversão. Depois converter:

Posto isto, a soma é 460, 624 dam.


7, 25 m:: ...;.- 5000 m :; = O, 145
13) Dividir 480 m :: por 80 m
10) Efetuar e dar o resultado em decâmetros quadrados:
9a - 3 20 cm X 51, 2 m + 125 dm X 8.400cm Então:
480m:1 480 m 2
Sabemos que: ---- = 6 m2
9 a 9 dam~ 80 m 80
320 cm = ü,32 dam 1 14) Um terreno tem 15 m de largura e 40 m de com-
X ~­ I,6384 dam ~ p1imento . Para cercá-lo com uma tela de arame que custa
51,2 m 5,12 dam j NCr$ 3, 20 o metro, qual será a despêsa?
O perímetro do terreno é:
125 dm l,25 dam 1
X r 10,500 dam2 2p =2a 2b + ou
8400 cm 8,04 dam J 2p =: 2 X 15 +2 X 40 = 110 m

,272, Pa.ulo Pessoa Problemas de Matemá ti ca pa.ra o Adn1issão 273


O metro de tela custando NCr$ 3, 20, a despêscr om (1
sua aquisição será: Sendo assim, cada lote terá:

110 X NCr$ 3, 20 = NCr$ 352, 00 6 km -;- 2 = 3 km de comprimento e


3 km -;- 2 = 1, 5 km de largura
15) Um terreno retangular foi dividido em quatro lotes
re tangulares iguais. O perímetro do terreno é ele 18 km e o . Sendo lotes retangulares, seu perímetro, como foi
comprimento é o dôbro da largura . Pede-se o perímetro de • nsmado, 2p = 2 X comprimento +
2 X largura e então:
cada um dos quatro lotes em que o terreno foi dividido em
dam.
2p = 2 X 3 km +2 X 1, 5 km = 9 km

Como a resposta deve ser dada em decâmetros é bas-


De início vamos calcular as dimensões do terreno, que 1un te transformar 9 km em decâmetros e vem: '
1depois de dividido em quatro partes (lotes) iguais, terá o
aspecto da figura: 9 km= 900 dam

comprimento 16) O_ pe~ím~tro ~e um triângulo isósceles, cuja soma


1- - --- - -, d.os lados iguais e o tnplo do lado diferente, é igual à 16m.
1 1 Calcular os lados do triângulo.
. 1 1
largura ~--1 -- -----1 O p erímetro de um triângulo isósceles, vimos ser:
1
1 1
1 1
2p = a + 2b
i- - - - ---1
O 2b representa a soma dos lados iguais e a o lado
O comprimento sendo o dôbro da largura, conclui-se diferente.
que a largura corresponde a duas partes iguais do compri-
Como a soma dos lados iguais é o triplo do lado dife-
mento e então o perímetro do terreno terá 6 partes iguais à
' ·nle, é como se tivessemos somado aos seus lados um
Jargura. Como o perímetro tem 18 km, cada uma das 6 par-
11 úmero igual a 4 vêzes o lado diferente.
tes terá:
Considerando que a soma dos lados é o perímetro e o
18 km -;- 6 = 3 km . p10blema diz que seu valor é 16 m.
Então a largura do terreno é 3 km e porque o problema Conclui-se que o seu lado diferente é:
diz que o comprimento tem o dôbro da largura, êle ser6
igual a: 16 m -;- 4 =4m
2 X 3 km = · 6 km Ficam sobrando para os lados iguais:
Para que o terreno tenha sido dividido em 4 lotes r etan- 16 m - 4 m = 12 m. e então cada lado igual va-
gulares iguais é preciso que tenha sido dividido ao meio l1•1á:
no sentido do comprimento e também no da largura.
12 m -:-- 2 = 6 m.
274 Paulo Pessoa
17) Um terreno tem 150 m de comprimento. A largura
2p = 2 X 450 m -!- 2 ,_ SGO m = 2 . 020 m
2
é - - - do comprimento. Calcular a área do terreno em Como pretendemos cercá.-lo com 3 fios de aram , <l v -
5 mos gastar o triplo do perímetro, isto é:
ares.
2 arame necessário. = 3 X 2 . 020 m = 6 . 060 m.
A largura sendo - - - do comprimento que tem 150 m,
5 19) Calcular em hectares a área de um terreno cujos
será: 7
2 medem 68.600 m 2 .
X 150 m = 60 m 11
5
Como no caso de vários problemas de frações :
A área: do terreno é dada por:
7 corresponde
S =a X b ou --:) 68.600 m'.!
S =
150 m X 60 m = 9. 000 m 2 11
Como o resultado deve ser dado em ares e o are é igual 1 corresPonde
ao dam~, vem: --:) 9.800 m 2
9.000 m~ = 90 dam~ = 90 a 11
18) Um terreno retangular tem 252. 000 m~ de área e 11 corresponde
5, 6 hm de largura . Se quisermos cercá-lo com 3 fios de ~ 89.800 m 2
arame farpado, quantos rolos de 80 m serão necessário? 11
Calculamos de início o comprimento do terreno, para o
que devemos dividir a sua área pela sua largura, depois de Como 1 m 2 1 ca (centiare), vem:
r eduzir a largura a metros, isto porque, sendo a áree o
produto do comprimento pela largura, conclui-se que o
89. 800 m 2 = 89. 800 ca = 8, 980 hét
comprimento será a área dividida pela largura, assim co- 19) Um reservatório contém 90 litros de água . Quantas
mo a largura será a área dividida pelo comprimento. lulas de 75 cl serão necessárias para conter a água do
1 servatório?
Então:
252 . 000 m 2
comprimento 450 m
Sabemos que 90 litros =
9.000 cl. Basta então dividir
'l . 000 cl por 75 cl para saber o número d e latas.
560 m
A seguir calcularemos o perímetrq do retângulo, que( Então:
como vimos, é:
2p = 2a + 2b 9. ººº cl 75 cl 120 latas

276 Problemas de Mntc múli ca pa,r:l o Admi ss ::ío 277


2
?.O) Um reservatório conté m água nt é - - - de sen Pod emos escrever:
3
volume. Suas dimensões são: 4 m; 3 m e l, 8 m . Quantos 2
liiros de água contém o reservatório? - - - --7 3, 10 hl
5
O volume do reservatório, que tem a forma de um para- 1
lelepípedo, vimos que é: - - - --) l, 70 hl
V = a X b X c 5

Então:
5
(a caixa tôda) ---- ~ 8, 50 hl
V = 4m
X 3 m X L8m 21, 6 m :) = 5
2
onsiderando que ·s ó - - - estão cheios , v .m: A metade da caixa conterá:
3 8, 50 hl -7- 2 4, 25 hl ou
Volume d'água contido no reservatóri o: 425 litros

2 23) As dimensões de um terreno retangular são: 38, 70


- - X 21,6 m :; = 14,4 m~ 111 e 28, 90 m. Ao seu redor constrói-se um muro de O, 30 m
3 d largura. Pede-se a superfície do terreno, em ares depois
lc murado .
21) Um terreno quadrado tem 250 m de lado. Custando
0 are do terreno NCr$ 30, 00, quanto custará a metade do As dimensões do terreno, depois de murado passou a
terreno? :;cr:
A á rea do terreno, que é quadrado é : 38,70 m ~ 0,30 m - 0,30m = 38, 10 m e
S = a~ = (250m)~ ·- 62. 500 m ' = 625 a 28 .90 m - 0,30 m - 0,30 m = 28,30 m
O seu valor total é: A área do terreno murado será:
625 X NCr$ 30. 00 NCr$ 18. 750, 00 S = 38. 10 m X 28, 30 m = 1. 078, 23 m 2 =
A metade custará então: 10, 782.3 dam 3 =
10,782 .3 ares ,

NCr$ 18 . 750 GO -7- 2 = NCr$ 9.375, 00 24)


Um reservatório de 2 m de comprimento; S d m d e
3
22) Quantos litros de água conterá a metade de uma lm gura, contém óleo até os - - - da altura. Êsse óleo foi
2 s
caixa d'água, se seus - - - contém 3, 40 hl? ~ k; trib uído
Por 75 latas cúbicas de 20 cm de aresta . Deter-
5 a:u inar a altura do reservatório.

278 P aulo I'esso o


l 'rublcrna :-.í de Ma l L•má Li ca i1a·ra u Admi ooáu 279
Se as !citas são cúbicas com 20 cm de aresta, seu · volu- 25) Um tanque é abastecido por três torneiras A pri-
me será: meira na razão de 900 dl por hora; a segunda 625 dl por
V = d3 = (O, 20 cm)º = O, 008 m 3 h ora e a terceira 285 dl por hora. Abertas ao mesmo tem-
po enchem o tanque em 3, S horas . Qual a capacidade do
Como são em número de 75, o óleo contido em tôdas tanque em hectolitros?
elas, será:
Em uma hora crs três torneiras deixam cair no tanque:
75 X O, 008 = O, 6 m 3
90 dl + 625 dl + 825 dl = 2-350 dl
A área da base do reservatório é:
O tanque ficará cheio, diz o problema, no fim de 3,5
S = 2 X O, S m = 1 m2 horas. Então sua capacidade será:
O volume do óleo que é de O, 6 m 3 , está contido em 2. 350 dl X 3, 5 = 8 . 225 dl = 8, 225 hl
um reservatório de 1 m 2 da base.
2ô) Um compo retangular com 25 hm de comprimento
O volume do reservatório sendo: e 1,6 km de largura está plantado cereal. Calcula ndo que
se devem colher 2 . 000 dal por 100 creres e que 10 dal do
V = a X b X c ou
cereal pesam 620 gramas, pede-se o valor total da colheita,
O, 6 = 2 m X O, S m X c (c é a altura do reser- supondo-se que uma tonelada custa NCr$ 30, 00 .
vatório) . Seu valor será:
A área do campo é:
O, 6 m 3
c = 0,6 m. S = 25 hm X 16 hm = 400 hm 2 = 400 ha
2 m X O, S m Em 100 ares ou 1 ha colhendo-se 2 . 000 dal, no terreno
3 todo a colheita será:
Como êle (óleo) a ünge só - - - da altura, segue-se 400 ha X 2 . 000 dal = 800. 000 dal.
5
que: Como 10 dal pesam 620 gramas, o pêso da colheita será:
3
-i> O, 6 m
800.000 dal X 62 g = 49.600 . 000 g = 49, 6 ton.

5 O custo da tonelada do cereal sendo NCr$ 30,.00, o valor


total da colheita será:
- -- --i> O, 2 m e 49, 6 X NCr$ 30, 00 = NCr$ 1 . 488 . 00
s
27) Uma caixa retangular tem 1, 8 m de comprimento;
5 8, S dm de largura e 75 cm de altura. Está cheia de gasolina
- -- --i> 1 m (altura total do reser- 3
5 até - - - da altura . Calcular o custo da quantidade de
vatório) 5

2.30 l'<tulo I'cs aua ·


Problemas de Matemática para o Admissão 281
Depois disso, cada reservatório teria~
gasolina contida nessa caixa, sabendo-se que o prêço do
decilitro é NCr$ O, 022. 135 hl --;- 3 - 45 bl, qile é o volume conlido no
3 pi iiaeiro reservatório.
Como a caixa só está cheia até os da sua .Assim sendo, conclui-se que o segundo terá:
5
,altura, que é 75 cm, segue-se que a altma da gasolina na 45 hl + 10 hl = 55 hl e o terceiro:
caixa é:
55 hl + 25 hl = 80 hl
3
X 75 cm ou 45 cm 29) A altura de um paralelepípedo retângulo mede
5 8 cm. A soma das dimensóes da base é igual a 24 cm,
sendo uma o triplo da outra. Calcular o volume do parale-
O vou.me de gasolina contido na caixa será então: lepípedo.
V = 8, S dm X 18 dm X 4, S dm = 688, 5 dm:: = A ba~e do paralelepípedo é constituída pelo compri-
688, 5 1 = 6. 885 dl mento e a largura que somados, diz o problema, v aiem
24 cm. Sendo uma aas dimensões o triplo da outra pode-
Como um decilitro custa NCr$ O, 022, o custo da gaso- mos dizer que as duas somadas valem quatro vêzes a menor
lina será: aimensão.
6 . 885 dl X NCr$ O, 022 = NCr$ 151, 47 Nessas condições, a menor dimensão será:

28) 180 hl de óleo deverão ser distribuídos por três 24 cm --;- 4 = 6 cm e a maior:
reservatórios, de modo que o segundo receba mais 10 hl
que o primeiro, e o terceiro mais 25 hl que o segundo. 3 X 6 cm = 18 cm
Quantos hl receberá cada reservatório? Depois disso acharemos o volume da paralelepípedo
Se tirarmos 10 hl de gasolina do segundo reservatór'o, que é:
e segundo e o primeiro ficarão com o mesmo número de hl V = a X b X e ou
de óleo. V -= o cm X ti cm X 18 cm = 864 cm"
O terceiro tem mais 25 hl do que o segundo tinha antes
de retirarmos dêle 10 hl Para igualá-lo ao primeiro. Se ;.;0) Um Paralelepípedo tem um volume igual u
retirarmos, portanto 25 hl +
10 hl do terceiro reservatório, 11. H:iU dm"; a altura mede 5 dm. Calcular a área du ba e
os três reservatórios ficarão com a mesma quantidade de do paralelepípedo.
óleo e correspondendo ao que tinha o primeiro.
Sendo o volume do paralelepípedo o produto da ár-.:u
da base pela altura, teremos:
Nessas condições não eram necessários 180 hl, bastando
que fôssem: 11. i60 dm~ =- área da base X altura ou
ll .16C dm't = án::a da base X 5 dm
180 hl - 10 hl - (25 +10 hl) ou á.rea da base =
11 . 160 dm:: --;- 5 dm 2. 232. dm" =
180 hl - 10 hl - 35 hl = 135 hl

Prolii t'rfl u·:i d!.! Mulemú.Ucn pa.l'a o Admi~~ão 283


~l) A soma das arestas de um cubo é igual a ~. 4 m..
Calcula r o volume do cubo em hectolitros. Assim :
Vimos que o volume do cubo é: 30 . 000 m 2 - 125 m 2 29 .875 m 2
V = a3 Depois:
Torna-se necessário calcular o valor da aresta, que são
oito, em um cubo e que somados valem 2, 4 m
29.875 m 2 ...;- 2 = 14.937, 5 m2
Ca da uma valerá então: E concluir que um terreno terá:
.2, 4 m ...;- 8 = O, 3 m 14 . 937, 5 m 2 e o outro:
O volume será pois: 14. 937, 5 m 2 + 125 m 2 = 15 . 062,5 m 2
V = (O, 3 m) 3 = O, 027 m = 27 dm
3 3
= 27 litros = 34) Uma sala de a ula tem 7, 50 m de comprimento
o. 27 hl 6, 80 m de largura e 3, 40 m de altura . Q ual 0 volume de
32) Um terreno retangular de 8, 4 dam de compri- ar reserva do a cada aluno se a sala contém 20 alunos?
llllento e 3, 2 hm de largura custou NCr$ 94, 00 . Por quanto O volume ·da sala é:
~eve ser vendido para se obter um lucro de NCr$ O, 20 por
centiare? V = 7, 50 m X 6, 80 m X 3, 40 m = 173, 4 m ª
A área do terreno é: Ca da aluno disporá então de:
S = 8, 4 dam X 32 dam = 268, 8 dam 2 ;:=: 268, 8 a 173, 4 m 3 ...;- 20 = 8, 67 m 3
= 26 880 ca
35) Uma sala de aula retangular tem 9,78 m de com-
Ganhar na venda NCr$ O, 20 por ca. equivale à ganhar: primento, 5, 36 m de largura e 3, 45 m de altura . De quanto
se deve levantar o teto para que os 52 alunos e o pro-
26. 880 X NCr$ O, 20 = NCr$ 53, 76 fessor tenham 4 m 3 de ar cada um?
! \ \!. ...

De ve ser vendido Por: O volume da sa la de aula necessário para os 52 al unos,


NCr$ 94, 000 + 53, 76 = NCr$ 147, 76 mais o professor, isto é, 53. pessoas, à razão de 4 m 3 por
p essoa, deve ser:
32) Dois campos têm juntos ·3 ha. Um tem 125 m" mais
do que o outro . Qual é a superfície de cada um?
53 X 4 i:n,
3
= 212 i:n, 3

Sabemos que 3 ha = 3 hm 2 = 30 . .QOO m 2. Se os dois A superfície da sala de aula (área da base) é:


terrenos íôssem iguais, e ra bastante dividir a superfície S = 9, 78 m X 5, 36 m = 52, 420 . 8 m •
total do terreno por dois .
Considerando que um tem 125 m 2 ma is do que o outro, O volume necessário, isto é, 212 m 3 , será obtido multi-
é bastante s ubtrair do total, a superfície de 125 m~. dividir plicando-se a área da sala pela altura .
o resto por dois e depois somar a um das partes os 125 m 2 Teremos então:.
retirados.
212 m 3 = 52, 420 . 8 m' X altura.
234 P aulo Pessoa
J;'rc;> l,Jl e m a s el e Ma t e má tica pa.ra 0 Ad mis s ã o 235
Como são três janelas, seus volumes totalizwu
Então: 3 X O, 585 m 3 -= 1, 755 m " que d v rn n 1 ui>
altura= 212 m
3 ...;- 52, 4280 m 2
= 4, 044 m 1r oído do volume total 21· m =:.

O problema diz que a sala tem 3, 45 m de altura. Então Então teremos:


precisará ser alterada de: 21 m=: - L 755 m '1 = 19, 245 m~
4, 044 m - 3, 45 m = O, 594 m
38) Uma sala: mede 12 m de comprimento por 80 dm
36) Durante um temporal recolheu-se numa cuba posta de largura . Quantos tacos quadrados de 8 dm de períme:ro
ao ar livre, uma quantidade de água que atingiu 6 cm. serão necessários para o revestimento do piso?
Que quantidade de água caiu sôbre um terreno de 20 da
>e 5 ca.? A área da sala é :
Para termos a superfície do terreno devemos somar
20 da. e 5 ca., para o que torna -se necessário converter tudo S = 12 m X 8 m = 96 m~
à mesma unidade O perímetro dos tacos quadrados que vão revestir o
Assim: piso sendo 8 dm ou O, 8 m, indica que o lado do taco é de
O, 8 m ...;- 4 ou seja O, 2 m .
20 da = 20.000 ca
5 ca := 5 ca e a superfície do terreno será: Sua área será, como vimos, a área do quadrado, isto é:
20 .005 ca =20. 005 m2
S = (0, 2)2 = O, 04 m"
A altura da água na cuba sendo de 6 cm, no terreno
todo também foi de 6 cm e o volume da água que caiu Serão necessários, então:
1s ôbre o terreno foi de 96 m~ --:- O, 04 m 2 2 400 tacos.
.20 . 005 m2 X 0,.06 m 1.200, 3 m 3
39) iPara constrnir um muro de 1620 cm de compri-
37) Uma parede de 28 m de comprimento, 2,50 m de mento, 45 dm de altura e 30 cm de espessura. Quantos tijo-
alttura e O. ·30 m de espessura tem três janelas de 1 m de los de 15 cm de comprimento, O, 25 m de largura e 6 cm
lar:gura por 1.95 m de altura. de espessura serão necessários para construi-lo?
Calcular o volume da parede .
O volume do muro será:
Se não houvessem çis janelas, o volume da parede seria :
seria: V = 16, 20 m X 4, 5 m X O, 30 m - 218, 7 m: 1
V =2, 50 m X 28 m X O, 30 m = 21 m 3
O volume de cada tijolo é:
Êsse volume ficou reduzido em virtude da abertura das
janelas, que ocupariam se não existissem, um volume V = 15 cm X 25 cm X 6 cm = 2. 250 cm3
igual a:
Dividindo-se o volume do muro pelo de cada tijolo,
V = 1 m X 1, 95 m X O, 30 m = O. 585 m 3 , cada teremos o número necessário de tijolos.
uma.
Probl e mas rlc Mntemúticn p:tm o Admiss ão 237
2.36
Então: Verifica-se, então, que um pe ~a mais 20 kg do que o outro.
V. do muro -218, 7 mª = 218.700 .000 cm 3 Qual a capacidade em dal do reservatório?
V. de cada tijolo 2. 250 cm3 Se pesarmos um reservatório com 1 hl de óleo e o outro
com 1 hl de água destilada, cujo hl pesa 100 kg; conside-
Então, serão necessários de: rando que os reservatórios vazios têm o mesmo pêso, a dife-
r ença encontrada seria:
218. 7000. 000 cm8 + 2. 250 cmn = 9. 720 tijolos
100 kg - 95 kg = 5 kg por hectolitro.
40) Um vaso vazio pesa 1.25 kg e cheio de água des-
tilada pesa 7,5 kg. Qual o volume do vaso? Como êles foram Pesados cheios, a diferença encontrada
foi d e 20 kg.
A diferença de pêso entre o vaso cheio de água e o
vaso vazio indica o pêso da água nêle contida que no Isto significa que no reservatório existiam:
caso será: 20 kg -+- 5 kg = 4 hl
7,5 kg - 1.25 kg =
6,25 kg.
Como a resposta deve ser dada em dal, teremos:
6,25 kg de água destilada, como já foi dito correspon- 4 hal =
40 dal
de a 6,25 litros ou 6.25 dm3 , que é o volume do vaso.
43) Na fabricação de pregos são perdidos 2 mm de
41) rm tonel vazio pesa 72 kg; contendo água pura arame em cada prego fabricado . Calcular o número de pre-
2 gos de 6 cm fabricados com um rôlo de arame próprio para
até os seus - -- pesa 48 kg. Quantos decímetros cúbi- a fabricação de 146, 072 m.
5
cos de água poderá conter o tonel? Transformemos 146, 072 m 1em centímetros:
2 pesam 146, 072 m = 14. 607, 2 cm
Se ~ 48 kg
5 Perder 2 mm em cada prego de 6 cm; é como se os
1 pesa pregos tivessem depois de fabricados, 6 cm 2 mm +
---~ 24 kg -+- 6 cm + O, 2 cm = 6, 2 cm .
5 Para sabermos o número de pregos a fabricar com o
---~ 120 kg rôlo de a rame, basta dividir o seu comprimento de 146, 072 m
pesam ou 146 07, 2 cm por 6, 2 cm.
5 Feita a divisão, enco traremos 2 356 Pregos .

Como no exemplo anterior, o Pêso de água nêle con- EXERCíCIOS PARA RESOLVER
tida, fica:
120 kg - 72 kg = 48 litros, que c~responde 1) Efetuar as seguintes mudanças de unidades:
a 48 dm 3 , que é o volume do tonel. a) 371 mm a dam e m
b) 41 cm a hm e dm
42) Tem.se dois reservatórios com a mesma capaci- c) 8 dam a mm e km
dade e pêso. Enche-se o primeiro de óleo cujo hectolitro d) 10, 54 km a dam e mm
pesa 95 kg e o segundo de água destilada (a 4° centígrados). e) 1341, 29 cm a m e km

288 Pa.ulo P~s•oa P r oblema s el e Malcmúli ca 11arn o J\drnla • fio 2~~9


Resposta!!: e) 2, 03 dam 3 em km 3 e mm 3
d) 183, 491. 5 hm~ e m m 3 e cm3
a ) O. 037 .1 dam e O, 371 m
e) O, 003. 94 km3 em dam3 e mm 3
b) O, 004 . 1 hm e 4, l dm
e) 80.000 mm e O, 08 km Respostas: a) 15 . 000 dm 3 e O, 015 dam 3
d) 1. 054 dam e 10. 540. 000 mm b) 779 mm 3 e O, 000 . 779 dm3
e) 13,412.9 me 0,013 . 412 . 9 km e) O, 000.002 .03 km 3 e 2 .030 .Q00 . 000. 000 mm 3
d) 1.834. 915 m 3 e 1.834 . 915.000 . 000 cm 3
2) Efetuar as seguintes mudanças de unidades: e) 39 . 400 dam 3 e 39. 400 . DOO . 000. 000 . 000 mm 3
a) 45, 7 dam2 a km 2 e mm 2
b) 65. 3 hm 2 a m 2 e cm 2 5) Converter:
e) 327, 4 mm 2 a hm 2 e m 2 a) 204 1 em hl e ml
d) 8.474 dm 2 a km 2 e mm 2 b) 400 dl em hl e dal
e) O, 005 . 6 hm 2 a dm 2 e dam 2 e) 13. 49 cl em ml e litros
d) O, 001 hl em 1 e ml
Respostas: e) 13, 47 ml em hl e dl
a) 0,004. 57 km2 e 4 . 570. 000. 000 mm 2
b) 16 53. 000 m • e 6. 530. 000 . 000 cm 2 Respostas: a) 2, 04 hl e 204 000 ml
e) 0,000.000.032.74 hm 2 e 0,000 .'3 27.4 m 2 b) O, 4 hl e 40 dal
e) 134, 9 ml e O, 134 . 9 litros
d) 0,000 . 084.74 km2 e 84.740.000 m 2
d) O, 1 1 e 100 ml
e) 5 600 dm 2 e O, 56 dam 2
e) 0,000 . 134.7 hl e O, 134 . 7 dl
Complete as igualdades:
6) Reduza:
a ) 32 a vT .. . ... hm 2 = . ... . ... dm 2 a) 215 1 a cm 3 e dm3
b) O,Aiha = . . .. .. ca = ..... ... km 2
b) 12, 41 hl a hm3 m 3
O, 513 . 0 m 2
e) = .... ...... = .. · ·. · · · · · · · ha
a
e) O, 015 dal a m 3 e mm 3
X <l) 1967 da = . . . . . . . . . . m 2 = .. .. .. . . .. · . ca
d) 1895, 2 dl a dam 3 e cm3
e) O, 198 ca = · .. . .. cm 2 = ....... ... mm 2
e) 10, 451 hl a m 3 e dm3
Respostas: a) O, 32 hm 2 e 320. 000 dm 2
b) 4 . 700 ca e O, 004. 7 km2 Respostas: a) 215.000 cm3 e 215 dm3
e) O, 005.13 a e O, 000 . 051.3 h o
b) 0,000.001 . 241 hm 3 e 1,241 m 3
e) O, 000 . 15 m 3 e 150.000 mm3
d ) 1.967.000 m 2 e 1 . 967 . 000 ca
d) O, 000 . 189. 52 darn 3 e là9 . 520 cm 3
e) 1 . 980 cm' e 198 . 000 mm 2
e) l , 045. l m 3 e 1.045, 1 dm3
4) Converter:
7) Reduza:
a ) 15 m ~ em dm~ e dam 3
a ) 1 327, 5 g a cg e mg
b) O, 779 cm3 em mm 3 e dm 3
b) 30, 19 hg a toneladas e g

2.90 P aulo P essoa Problemas de Matemáti ca p ara o Ad missão 291


c) 18 345, 3 dag a dg e mg
d) 3, 198 toneladas a dag e cg
@ Efetuar, dando a respos'a em ca:
e) 194 053 mg a hg e kg 5 a + 4 dam + 5, 8 ha
2

Resposta: a ) 132 750 cg e 1 327 500 mg Resposta: 58. 900 ca


b ) O. 003.019 ton e 3019 g
c) 1. 834. 530 dg e 183 . 453. 000 mg 13) Dados: A =0,05 km; B = 2.4 hm e C = 150 m,
d) 319 800 dag e 319.800 .000cg diga em dm . quanto íalta à soma dessas três medidas para
e) 1, 940.53 hg e O, 194 .053 kg completar 50 dam .
d) O, 000. 382 toneladas.
8) Determinar o pêso dos seguintes volumes de água Resposta: 600 dm Colégio Pedro II - 1967
destilada:
14) Reduza
a) 3.82, 7 dm 3 em g
b) 49, 73 cm 3 em kg a) 5, 79 hm a mm
c) 1 dm 3 em toneladas b) 3. 2 ca a m 2
d) 10, 17 m 3 em dag c) 640 cm3 a hl
e) O, 35 hl em hg d) 38, 2 dag a toneladas
f) 3, 19 1 em kg Resposta: a) 579. 000 mm; b) 320 m 2 ; c) O, 006 . 4 hl;
d) O, 000 382 toneladas
Respostas: a) 382. 700 g; b) O, 049. 73 kg; c) O, 001 ton.; Colégio Mili tar - 1967
d) L O. 17,00 dag e) 350 hg; f) 3 19 kg.
15) Efetue dando o resultado em dm 3 :
9) De uma salina foram retirados vários carregamen-
tos de sal. O primeiro de 58, 75 m 3 ; o segundo de 60, 038 m 3 5
10 m 2 X ( -- dal -:- 500 cm 2 )
e o terceiro de 118, 788 m 3 • Qual a quantidade de sal reti-
4
rada?
Resposta: 2. 500 dm 3 Colégio Militar - 1967
3
Resposta: 237, 576 m
16) Quantos decímetros cúbicos há em 7 meios litros?
@) Efetue a soma e dê o resultado em ha: Resposta: 3, 5 dm3 GinaS'ial - 1967

325, 78 m2 +r849 dm + .j7. 21 dam


2 2

63,
17) Qual a diferença entre a metade de 8, 58 dag e
g em gramas?
Resposta: O, 505. 527 ha
Resposta: 36, 56 g Ginasial - 1967
)\__ 0:) Efetue a soma e dê o resultado em dam2 :
18) Um caminhão que pode transportar a té 8 tonela-
das está com 5 . 200 kg de carga. Q uantos quilos poderia
72 a + 5c.!82.9 ha + ca ainda tra nsPortar?
Resposta: 585, 97 dam2 Resposta: 2 . 800 kg Ginasial - 1967

:292. Paulo Pessoa P r ob!cma·s de Matemática para o Admissão 293


Quantos gramas tem um pacote de biscoito de dois 25) Um terreno re'.angular tem 18 m de largura e 250
quilogromas e meio? dm comprimento . Quem percorrer o seu perímet: o 5
vêzes, quantos quilômetros andará?
Resposta: 2. 500 g Ginasial - 1967
Resposta: O, 43 km
20 Emílio andou durante duas horas e meia com a
velocidade de 4 km por hora. Qual a distância percorrida
)\ @ Calcular o perímetro de um ret~ngulo que tem
por êle em dam? 20 m de comprimento e cuja largura é - - - dêle.
4
Resposta: 1 . 000 dam Ginasial - 1967
Resposta: 90 m.
@ Efetue e dê o resultado em dam2 : · Gi} Calcular em metros quadrados a área de um retân-
gulo que tem 2, 6 dam de perímetro, sabendo-se que a base
x a) 87 hm 2 -
450. 000 m 2
+
O, 421 m2
,.( b) 307 a +
9 da - 300 dca tem 8 m.
><_ e) 32 m X 2 dm +O, 48 hm 2 ...;-- 25 dam X 40 d m Resposta: 40 m 2
Resposta: a) 4 . 242 dam2; b) 367 dam 2 ; c) O, 832 .00 da m 2 28 Um terreno retangular de 40 m de largura e 6 dam
de mprimento, que valor terá, se o metro q-nadrado custa
@ Efetuar dando o resultado em m~: NCr$ 4, 20?
>(. a) 10 dm X 10 dm X 10 dm NCr$ 10. 080, 00
>( b) 3 drn X O, 4 mm X 10 hrn 9) Um terreno tem 300 m de comPrimento . A largura
>(_ e) 1 m X O, 625 dam2 ...;-- 125
Resposta: a) 1 m 3 ; b) O, 120 m 3 ; c) O, 500 mª é - - - do comprimento. Calcular sua área em hectares.
5
X.. 2 Um dos lados de uma rua de meio quilômetro d '.l Resposta 7, 2 ha
extensão foi urbanizada de uma extremidade à outra, conser-
vando-se entre as árvores a distância de 20 m. Sabendo-se 30) Considerando A = O, 05 km; B = 2, 4 hm e C =:.
que o prêço de cada árvore é de NCr$ 1, 50 e que a mã o 150 m, as dimensões de um reservatório (forma de para-
P,e obra importa em NCr$ 130, 00, qual será a despêsa tota l? lelepípedo), calcular a sua capacidade em dam~ .
A..
Resposta: 800 dam3 Colégio Pedro II - 1967
Resposta: NCr$ 69,00~ Ginasial - 1867
31) Uma caixa de forma cúbica contém água a lé O ,2 m .
@ 2
Um terreno de 41, 60 dam de área e 520 dm de
Jargura, tem a forma retangular. Quantos rolos de arame
Sabendo-se que a aresta do cubo mede 6 dm, calcular a
~apacidade da caixa em dl e o volume da parte vazia em
farPado de 42 m são necessários para cercá-lo com 7 ordens çm3.
de fios? Resposta: Capacidade da caixa: 2 . 100 dl; vo:ume da
parte vazia: 144. 000 cm3
Resposta: 44 rolos Colégio Militar - 1967

2.94 Paulo Pessoa Problema:s de Matemática pa.ra o Admissão 295


8 39) O comprimento do lado de um terreno quadrado é
32)Calcular em ares a área de um terreno cujos - - - de 30 m. Ao seu redor fêz-se um canteiro de L 5 m d e
9 largo . Pede-se a superfície ocupada pelo canteiro em ha.
m edem 19, 44 km 2 •
Resposta : O. 17 . 1 ha
Resposta: 218 . 700 ares.
40) Um reservatório cuja capacidade é de 12 .075 dm 3
33) Em uma sal a re tangular medindo 6 por 4 m foi é alimentado por uma torneira que fornece 900 l de á gua
colocado um tapête também retangular, distando um metro por hora . Uma outra torneira esva zia-o na razão d e 4, 37 hl
rle cada parede. Calcular a área do tapête. por hora . Estando o reservatório vazio, em quanto tempo
Resposta: 8 m " Ginasial - 1967 ficará cheio se as duas torneiras fôrem abertas juntas? Q ual
o pêso da água existente no reservatório no fim de 2 horas?
34) Uma caixa d'água de base quadrada de 5 m de Resposta 25 horas e 966 kg
lado e de 2 m de profundidade está cheia. Quantos litros
d'água há nessa caixa? 41) Um salão de 12 m de comprimento e O, 8 dam de
Resposta: 50. 000 litros Ginasial - 1967 largura, para ser ladrilhado, quantos ladrilhos necessita se
1ôrem quadrados e tiverem 8 dm de perímetro?
35) Um tanque tem O, 64 hl de volume. Quantas latas
de forma cúbica, de O, 4 dm de aresta são necessários para
Resposta: 2. 400 ladrilhos
tonterem o óleo nêle guardado?
42) Um pátio mede 18 m de comprimento e 25.000 cm
Resposta: 1 . 000 latas pe largura . Quero forrá-:lo com ladrilhos quadrados de
200 mm de lado, exceto uma barra de 10 dm de largura
36) Um terreno quadrado de 30 dam de lado foi ven- Quantos ladrilhos gastarei?
dido por NCr$ 1. 800, 00. Quanto custou o are?
Resposta: Gastarei 99 . 200 ladrilhos
Resposta: NCr$ 2,00
Liceu Nilo Peça nha - 1967
4
37) Os - - - do volume de um reserva tório contém 43) Um reserva '. ório mede 3 dam de comprimento; 4 m
5 de largura e 90 dm de altura . Considerando-se o reserva-
400 hl de água. Quanto conterá a metade do reservatório tório cheio de refrigerante que custa NCr$ O, 18 o litro, qual
em m 3 ? volume do reservatório e qual o valor do refrigerante?
Resposta: 25 m 3
Respos~a: Volume : 1080. 000 litros; - Valor do refri-
38) Um terreno quadrado de 1. 500 dm de lado para gerante : NCr$ 1 . 944, 00
Liceu Nilo Peçanha - 1967
ser cercado por uma tela de arame que custa NCr$ 0,50,
que despêsa acarretará?
44) Um reservatório com a forma de um paralelepípedo
Resposta: NCr$ 300, 00 retângulo, tem 1 m de comprimento, 9 dm de la rgura e

2·96 P aulo Pessoa P r obl em as de Mutcmálica para o Admissão 297


80 cm de altura . Estando vazio, colocam-se 48 dal d' água . que o leite pesa l, 030 hg por litro, determinar a capacidade
Quantos litros d' água ainda caberiam no reservatório? da bilha em hectolitros.
Resposta: 1 hl
Resposta: 240 litros 1. E. e C. Dutra - 1968 ..
45) Copiando as dimensões de um retângulo medindo 50) A altura de um paralelepípedo retângulo é de
10, 5 dam de comprimento por 12, 4 dam de largura, escre- 6 cm. O perímetro da base é igual a 54 cm, sendo uma
veu-se dm em lugar de dam . Qual foi a diferença em me- de suas dimensões o dôbro da outra. Calcular o volume do
tros quadrados encontrada no cálculo de sua área? paralelepípedo em decalitros .

Resposta: 13.018, 6980 m 2 Resposta: O, O97 . 4 dal


Colégio Pedro II - 1968
51) A área da base de um Paralelepípedo retângulo é
72 dm 2 • Calcular a altura do sólido, sabendo-se que o seu
46) Um campo quadrado com 80 km de perímetro, está volume é de 504 litros.
plantado de trigo. Presume-se que sejam colhidos 100 hl
por 2 . 000 da. O pêso do trigo é de meio quilo por litro e o Resposta 7 dm
quintal métrico é vendido ao prêço de NCr$ 20, 00.
52) Um terreno retangular cuja largura é a metade do
Qual o valor da colheita? comprimento, foi adquirido a razão de NCr$ 15, 00 o are.
Resposta: NCr$ 2 . 000, 00 O perímetro do terreno é de 504 m. Calcular o lucro obtido
na venda dêsse terreno a NCr$ 2. 500, 00.
47) Uma caixa com a forma de um paralelepípedo tem
2 m de comprimento, 16, 5 dm de largura e 144 cm de altura. Resposta: NCr$ 383, 20
2
Es'. á cheia de óleo até - - - da altura. 53) Comprei dois sítios medindo, respectivamente 285
3 km 2 e 151.500 dam 2 • Tendo pago, ao todo NCr$ 1.206,00
quanto custou cada km 2 ?
Calcular o custo do óleo nêle contido sabendo-se que
o prêço do litro da substância é de NCr$ L 00. Resposta: NCr$ 80, 40 Colégio Pedro II - 1968

Resposta: NCr$1. 168, 00


@ Um terreno foi comprado por NCr$ 280. 000, 00 e
48) Distribuir 10. DOO litros d'água por três tanques, de em seguida dividido em dois lotes, tendo um dêles 3 ha
;modo que o primeiro tenha 100 litros mais do que o segundo mais do que o outro. Calcular em ha, a área de cada lole,
e o segundo 1 . 200 litros mais do que o terceiro. J;abendo-se que o m 2 foi pago a NCr$ 4, 00.

Resposta: 1.0 3.800 litros; 2. 0 3.700 litros e o 3.0 2.500 Resposta: 2 ha e 5 ha


litros
55) Uma sala de aula deve receber 50 alunos e tem
49) Uma bilha cheia de leite pesa 116 kg . Cheia de
46, 90 m 2 de superfície e 6, 50 m de largura. De quanto deve
água destilada (a 4° centígrados) pesa 113 kg . Sabendo-se

;z93 Paulo Pessoa


P roblema·s de Matemática para o Admissão 299
61 ) Um reservatório cujas dimensoes são iguais a 5 dm,
s er aumentado o comprimento para que cada a luno dispo-
10 dm e 15 dm, es'. á cheio de um líquido do qual cada litro
nha d e L 25 m 2 ?
p esa 1, 15 kg e é vendido a NCr$ 50,00 o hec logrcnna. Cal-
Resposta : 2, 40 m. cular o valor do líquido contido no reservatório .

56) Um salã o de cinema deve ter capacidade para Resposta: NCr$ 431 . 250 00
·350 espectadores. Q ual deve ser a sua altura , para que 62) Na fabricação d e a gulhas de 'lilJ eção perde-se
cada espectador possa dispor de 4 m 3 de a r, sabendo que mm de fio de platina em cada agulha de 5 cm . Q ua ntas
o seu comprimento é d e 20 m e sua largura de 15 m? a gulhas p odem ser feitas com uma peça de O. 108 km de
comprimento?
Resposta: 4, 667 m
Respos'. a : 18 agulhas
57) Calculàr o prêço da construção de um muro de
5 m de comprimento, 20 dm de altura e 60 cm de espessura , 63) A figura abaixo representa um terreno retang,ular
isabendo-se que o m 3 custou NCr$ 151, 00 . ABC D dividido em 4 lotes triangulares iguais . O Pen me-
tro do terreno mede 2 . 400 m; sua maior dimensão é o triplo
Resposta: NCr$ 900,00 ela menor . Pede-se: l.º) As dimensões do terreno em decâ-
metro . 2.º) A área d e um dos lotes em que o terreno foi
58) Uma muralha de 5 m de comprimento, 20 dm de à ividido, em ares.
altura e 30 cm de espessura . Na sua construção foram em-
pregados tijolos de 20 cm de comprimento, L 5 dm de altura

:N/21:
e 10 cm de espessura. Calcular o número de tijolos empre-
3
gados, sabendo-se que a a rgamassa ocupa - - - do volu-
20
lne total da muralha.

Resposta: 850 tijolos Resp osta: l.º) 90 dam e 30 dam; 2.0 ) 675 a res

59) Um reservatório de água tem as dimensões : 2, 4 m, Colégio Militar - 1967


5 m e 1 m. Quantos dal de água podemos depositar no refe-
rido reservatório?

Resposta: 1 . 200 dal

60) Um arrozal de 2, 5 ha, produz 5 litro Por m 2 • Valen-


do NCr$ 1. 000, 00 o saco de 50 kg, pede-se o va lor da pro-
duçã o .

Resposta: NCr$ 200 . 000, 00

30·0 Paulo Pessoa


P rnblcmacs tle Ma t emáti ca pa.ra o Admi ssão 301
--~------ -- . . '
índice

Prefácio 9

Numeração ....................... ........... 11

Operações fundamentais ........................ 21

Potência 67

Divisibilidade . . . . . . . . . . . . . . . . . . . . . . . . . . . . . . . . . . 75

Números primos . . . . . . . . . . . . . . . . . . . . . . . . . . . . . . . . 91

Maior divisor comum . . . . . . . . . . . . . . . . . . . . . . . . . . 107

Mínimo múltiplo comum . . . . . . . . . . . . . . . . . . . . . . . . 127

Frações 145

Números decimais . . . . . . . . . . . . . . . . . . . . . . . . . . . . . 213

Sist ema m étrico decimal . . . . . . . . . . . . . . . . . . . . . . . . 247

Você também pode gostar